NU374 Exam 4

¡Supera tus tareas y exámenes ahora con Quizwiz!

A client with urinary retention needs to undergo a procedure to insert an indwelling catheter. What should the nurse discuss with the health care provider before catheterization? type and size of the catheter to be used placement of the catheter procedure for insertion of the catheter administration of cleansing enemas

type and size of the catheter to be used Before catheterization, the nurse should inquire about the type and size of the catheter to be used and if the catheter should be removed or retained in place after the bladder is empty.

In a diagnosis of an upper urinary tract infection, which structures could be affected? Select all that apply. urethra ureter kidney bladder

ureter kidney The upper urinary tract is composed of the kidneys, renal pelvis, and ureters.

A client with Crohn's disease is to receive prednisone as part of the treatment plan. Which of the following instructions would be appropriate? "Avoid contact with other people who might have an infection." "Make sure to increase your salt intake to compensate for the loss of fluid." "Once your symptoms improve, you can stop taking the drug." "Take the drug on an empty stomach to avoid upsetting your stomach."

"Avoid contact with other people who might have an infection." Clients taking corticosteroids may not experience a normal immune response to infection. The client needs to monitor himself or herself for signs and symptoms of infection and to avoid situations where they may be exposed to infection, such as others who might be ill. The drug should be taken with meals to decrease gastrointestinal irritation and should be withdrawn or tapered slowly to prevent Addisonian crisis. Clients also need to limit their sodium intake or follow a low-sodium diet to minimize water retention associated with this drug.

A nurse is providing discharge instruction for a client who is postoperative bariatric surgery. What statement will the nurse include when providing teaching aimed at decreasing the risk of gastric ulcers? "Sit in a semi-recumbent position while eating." "Avoid taking antacid drugs." "Keep the head of your bed propped on blocks at night." "Avoid taking non-steroidal anti-inflammatory drugs."

"Avoid taking non-steroidal anti-inflammatory drugs." The only statement that aids in avoiding gastric ulcers is the statement instructing the client to avoid taking non-steroidal anti-inflammatory (NSAID) drugs. Sitting in a semi-recumbent of low Fowler's position aids in digestion but does not aid in the prevention of gastric ulcers. Propping the head of the bed would be beneficial for a client report GERD or acid reflux. antacid drugs do not increase the risk of gastric ulcers.

A client is prescribed tetracycline to treat peptic ulcer disease. Which instruction would the nurse give the client? "Expect a metallic taste when taking this medicine, which is normal." "Do not drive when taking this medication." "Take the medication with milk." "Be sure to wear sunscreen while taking this medicine."

"Be sure to wear sunscreen while taking this medicine." Tetracycline may cause a photosensitivity reaction in clients. The nurse should caution the client to use sunscreen when taking this drug. Dairy products can reduce the effectiveness of tetracycline, so the nurse should not advise him or her to take the medication with milk. A metallic taste accompanies administration of metronidazole (Flagyl). Administration of tetracycline does not necessitate driving restrictions.

A nurse is teaching a client with gastritis about the need to avoid the intake of caffeinated beverages. The client asks why this is so important. Which explanation from the nurse would be most accurate? "Caffeine intake can cause tears in your esophagus and intestines, which can lead to hemorrhage." "Caffeine can interfere with absorption of vitamin B12, which leads to anemia and further digestive problems." "Caffeine stimulates the central nervous system and thus gastric activity and secretio

"Caffeine stimulates the central nervous system and thus gastric activity and secretions, which need to be minimized to promote recovery." Caffeine is a central nervous system stimulant that increases gastric activity and pepsin secretion. Caffeine is a diuretic that causes decreased fluid volume and potential dehydration. It does not lead to hemorrhage and does not interfere with absorption of vitamin B12.

A client asks the nurse why a creatinine clearance test is accurate. The nurse is most correct to reply which of the following? "Creatinine is metabolized in the liver and excreted by the kidney at a regular rate." "Creatinine is a stress-related response that is excreted by the kidney." "Creatinine is broken down at a constant rate, and the total amount is excreted by the kidney." "Creatinine is found in the urine to make the urine acidic and can be measured."

"Creatinine is broken down at a constant rate, and the total amount is excreted by the kidney." A creatinine clearance test is used to determine kidney function and creatinine excretion. Creatinine results from a breakdown of phosphocreatine. It is filtered by the glomeruli and excreted at a consistent rate by the kidney.

A nurse cares for an obese client taking phentermine for weight loss. What client teaching will the nurse include when discussing precautions about the medication? "Take the medication at night before bedtime." "Take the medication with a full glass of water." "Do not drink alcohol while taking this medication." "Do not drive while taking this medication."

"Do not drink alcohol while taking this medication." The nurse should tell the client to avoid drinking alcohol while taking this medication. The other answer choices do not pertain to education specific to this medication.

A client is scheduled for magnetic resonance imaging (MRI). During the client teaching, what will the nurse discuss? "You must remove all jewelry but can wear your wedding ring." "You must be NPO for the day before the examination." "Do you experience any claustrophobia?" "The examination will take only 15 minutes."

"Do you experience any claustrophobia?" Explanation: MRI is a noninvasive technique that uses magnetic fields and radio waves to produce images of the area being studied. Clients must be NPO for 6 to 8 hours before the study and remove all jewelry and other metals. The examination takes 60 to 90 minutes and can induce feelings of claustrophobia, because the scanner is close fitting.

While reviewing a client's chart, the nurse notes the client has been experiencing enuresis. To assess whether this remains an ongoing problem for the client, the nurse asks which question? "Is it painful when you urinate?" "Does it burn when you urinate?" "Do you have a strong desire to void?" "Do you urinate while sleeping?"

"Do you urinate while sleeping?" Enuresis is defined as involuntary voiding during sleep. The remaining questions do not relate to this problem associated with changes in the client's voiding pattern.

A nurse is educating a client who will undergo bariatric surgery on methods to prevent dysphagia. What teaching will the nurse include? Select all that apply. "Chew your food thoroughly." "Avoid eating tough foods." "Eat slowly." "Eat bland foods such as doughy bread." "Avoid eating overcooked meats."

"Eat slowly." "Chew your food thoroughly." "Avoid eating tough foods." "Avoid eating overcooked meats." Dysphagia means "difficulty swallowing." This complication may occur after restrictive bariatric surgery and tends to be most severe 4 to 6 weeks after surgery and persists for up to 6 months. The nurse should instruct the client to chew thoroughly and eat slowly. Advise the client to avoid eating tough foods, doughy breads, and overcooked meats.

nurse cares for a client who wants to know more information about bariatric surgery. The client asks the nurse, "What weight loss can I expect?" What is the nurse's best response? "Expect to lose 45-50% of total body weight 2 to 3 years postoperatively." "Expect to lose 50 pounds in the first month after surgery." "Expect to lose 10-35% of total body weight 2 to 3 years postoperatively." "Expect to lose 10-35 pounds in the first month after surgery."

"Expect to lose 10-35% of total body weight 2 to 3 years postoperatively." When discussing weight loss expectations with the client, the nurse should let the client know to expect to lose 10-35% of total body weight 2 to 3 years postoperatively. The client may lose a large amount of weight the first month after surgery; however, this is not generally quantified with exact numbers or ranges.

The nurse is caring for a man who has experienced a spinal cord injury. Throughout his recovery, the client expects to gain control of his bowels. The nurse's best response to this client would be which of the following? "Having a bowel movement is a spinal reflex requiring intact nerve fibers. Yours are not intact." "It is not going to happen. Your nerve cells are too damaged." "Wearing an undergarment will become more comfortable over time." "Over time, the nerve fibers will regrow new tracts

"Having a bowel movement is a spinal reflex requiring intact nerve fibers. Yours are not intact." Explanation: The act of defecation is a spinal reflex involving the parasympathetic nerve fibers. Normally, the external anal sphincter is maintained in a state of tonic contraction. With a spinal cord injury, the client no longer has this nervous system control and is often incontinent.

The nurse is caring for a client following gastrointestinal diagnostic testing. The client verbalizes being ashamed because he is having frequent gas. Which nursing suggestion is best? "Do not be ashamed. Everyone has gas following the procedure." "The nursing staff is used to having clients with gas due to the procedure completed." "Having gas following the procedure is normal. Expel the gas to decrease discomfort." "Nurses anticipate that client will have gas following the procedure and prov

"Having gas following the procedure is normal. Expel the gas to decrease discomfort." Explanation: The nurse is correct to tell the client that what he is experiencing is normal and encourage the client to release the gas to decrease pain and discomfort. Proving information relieving the embarrassment and stating the benefit of the action is most helpful.

A patient is scheduled for a test with contrast to determine kidney function. What statement made by the patient should the nurse inform the physician about prior to testing? "I don't like needles." "I take medication to help me sleep at night." "I am allergic to shrimp." "I have had a test similar to this one in the past."

"I am allergic to shrimp." The nurse should obtain the patient's allergy history with emphasis on allergy to iodine, shellfish, and other seafood, because many contrast agents contain iodine.

The nurse is giving discharge instructions to the client following a bladder ultrasound. Which statement by the client indicates the client understands the instructions? "I should increase my fluid intake for the rest of the day." "If I have difficulty urinating, I should contact my physician." "It is normal for my urine to be blood-tinged." "I can resume my usual activities without restriction."

"I can resume my usual activities without restriction." A bladder ultrasound is a non-invasive procedure. The client can resume usual activities without restriction.

A client is readmitted with an exacerbation of celiac disease 2 weeks after discharge. Which statement by the client indicates the need for a dietary consult? "I don't understand why this happened again; I didn't travel out of the country." "I don't understand this; I took the medication the doctor ordered and followed the diet." "I didn't eat anything I shouldn't have; I just ate roast beef on rye bread." "I don't like oatmeal, so it doesn't matter that I can't have it."

"I didn't eat anything I shouldn't have; I just ate roast beef on rye bread." The client stating that he ate roast beef on rye bread indicates the need for a dietary consult because rye bread contains gluten, which must be eliminated from the client's diet. The client stating that he's followed the ordered medication regimen and diet doesn't suggest that the client needs a dietary consult; a treatment regimen consisting of medications to improve symptoms and dietary modification is necessary to treat celiac disease. The client stating that he hasn't traveled outside of the country doesn't suggest that dietary concerns exist. The client saying that he can't have oatmeal shows an understanding of the dietary restrictions necessary with celiac disease.

The nurse determines that teaching for the client with peptic ulcer disease has been effective when the client makes which statement? "I should stop all my medications if I develop any side effects." "I have learned some relaxation strategies that decrease my stress." "I should continue my treatment regimen as long as I have pain." "I can buy whatever antacids are on sale because they all have the same effect."

"I have learned some relaxation strategies that decrease my stress." The nurse assists the client to identify stressful or exhausting situations. A hectic lifestyle and an irregular schedule may aggravate symptoms and interfere with regular meals taken in relaxed settings along with the regular administration of medications. The client may benefit from regular rest periods during the day, at least during the acute phase of the disease. Biofeedback, hypnosis, behavior modification, massage, or acupuncture may be helpful.

A nurse is teaching an older adult client about good bowel habits. Which statement by the client indicates to the nurse that additional teaching is required? "I need to use laxatives regularly to prevent constipation." "I should exercise four times per week." "I need to drink 2 to 3 liters of fluids every day." "I should eat a fiber-rich diet with raw, leafy vegetables, unpeeled fruit, and whole grain bread."

"I need to use laxatives regularly to prevent constipation." The client requires more teaching if he states that he'll use laxatives regularly to prevent constipation. The nurse should teach this client to gradually eliminate the use of laxatives because using laxatives to promote regular bowel movements may have the opposite effect. A high-fiber diet, ample amounts of fluids, and regular exercise promote good bowel health.

A nurse is teaching a female client with a history of multiple urinary tract infections (UTIs). Which statement indicates the client understands the teaching about preventing UTIs? "I should wipe from back to front." "I should take a tub bath at least 3 times per week." "I should take at least 1,000 mg of vitamin C each day." "I should limit my fluid intake to limit my trips to the bathroom."

"I should take at least 1,000 mg of vitamin C each day." The client demonstrates understanding of teaching when she states that she should take vitamin C each day. Increasing vitamin C intake to at least 1,000 mg per day helps acidify the urine, decreasing the amount of bacteria that can grow. The client should wipe from front to back to avoid introducing bacteria from the anal area into the urethra. The client should shower, not bathe, to minimize the amount of bacteria that can enter the urethra. The client should increase her fluid intake, and void every 2 to 3 hours and completely empty her bladder. Holding urine in the bladder can cause the bladder to become distended, which places the client at further risk for UTI.

The nurse is preparing the client for magnetic resonance imaging (MRI) of the kidney. Which statement by the client requires action by the nurse? "I had my last cigarette 3 hours ago with my morning coffee." "I took my blood pressure medication with my morning coffee an hour ago." "I did not take my multivitamin this morning." "I do not have a pacemaker, artificial heart valve, or artificial joints."

"I took my blood pressure medication with my morning coffee an hour ago." The client should not eat for at least 1 hour before an MRI. Alcohol, caffeine-containing beverages, and smoking should be avoided for at least 2 hours before an MRI. The client can take his or her usual medications except for iron supplements prior to the procedure.

The nurse is providing instructions to the client prior to an intravenous pyelogram. Which statement by the client indicates teaching was effective? "I should remove all jewelry before the test." "I will need to drink all of the dye as quickly as possible." "I should let the staff know if I feel claustrophobic." "I will feel a warm sensation as the dye is injected."

"I will feel a warm sensation as the dye is injected." A contrast agent is injected into the client for an intravenous pyelogram. The client may experience a feeling of warmth, flushing of the face, or taste a seafood flavor as the contrast infuses. Jewelry does not need to be removed before the procedure. Claustrophobia is not expected.

A home care nurse is caring for a client with reports of epigastric discomfort who is scheduled for a barium swallow. Which statement by the client indicates an understanding of the test? "I'll drink full liquids the day before the test." "I'll take a laxative to clear my bowels before the test." "There is no need for special preparation before the test." "I'll avoid eating or drinking anything 6 to 8 hours before the test."

"I'll avoid eating or drinking anything 6 to 8 hours before the test." Explanation: The client demonstrates understanding of a barium swallow when stating he or she must refrain from eating or drinking for 6 to 8 hours before the test. No other preparation is needed. Before a lower GI series, the client should eat a low-residue or clear liquid diet for 2 days and take a potent laxative and an oral liquid preparation.

The nurse is explaining the steps for collecting a clean catch urine specimen to a client. Which statement by the client indicates effective teaching? "I need to use one antiseptic wipe to clean the sides and down the middle." "After I clean the area, I can let go of my labia to hold the container." "I'll start to urinate for a few seconds and then start to collect the specimen." "I need to collect at least 100 mL of urine for the specimen."

"I'll start to urinate for a few seconds and then start to collect the specimen." When collecting a clean catch urine specimen, the client would begin voiding for a few seconds and then collect 30 to 50 mL of the midstream urine into the container. The client would use one antiseptic towelette to clean the one side of the urethra, one to clean the other side of the urethra, and then a third to clean the center. The labia are held apart during the cleaning process and throughout the voiding until after the specimen is collected.

A client undergoes extracorporeal shock wave lithotripsy. Before discharge, the nurse should provide which instruction? "Increase your fluid intake to 2 to 3 L per day." "Be aware that your urine will be cherry-red for 5 to 7 days." "Take your temperature every 4 hours." "Apply an antibacterial dressing to the incision daily."

"Increase your fluid intake to 2 to 3 L per day." The nurse should instruct the client to increase his fluid intake. Increasing fluid intake flushes the renal calculi fragments through — and prevents obstruction of — the urinary system. Measuring temperature every 4 hours isn't needed. Lithotripsy doesn't require an incision. Hematuria may occur for a few hours after lithotripsy but should then disappear.

A nurse is caring for a client who will undergo bariatric surgery. Which nutritional recommendation will the nurse include in the client teaching? "Increase your intake of fluids at meals." "Increase your intake of plant-based proteins." "Increase your intake of complex carbohydrates." "Increase your intake of monounsaturated fats."

"Increase your intake of plant-based proteins." The client should be advised to increase protein intake, particularly plant-based protein because animal-based protein may not be tolerated well. The client should be advised to decrease fat intake, regardless of the source. Additionally, the client should be advised to decrease fluid intake at meals, not increase intake.

A nurse cares for a client who is obese. The health care provider prescribes orlistat in an effort to help the client lose weight, along with diet and exercise. When teaching the client about this medication, what will the nurse include? "It binds with enzymes to decrease carbohydrate absorption." "It decreases your appetite." "It works to make you feel full." "It binds with enzymes to help prevent digestion of fat."

"It binds with enzymes to help prevent digestion of fat." Orlistat (Xenical) works to bind to gastric and pancreatic lipase to prevent the digestion of 30% of ingested fat, thereby decreasing caloric intake.

A patient has been diagnosed with acute gastritis and asks the nurse what could have caused it. What is the best response by the nurse? (Select all that apply.) "It is a hereditary disease." "It can be caused by ingestion of strong acids." "You may have ingested some irritating foods." "It is probably your nerves." "Is it possible that you are overusing aspirin."

"It can be caused by ingestion of strong acids." "You may have ingested some irritating foods." "Is it possible that you are overusing aspirin." Acute gastritis is often caused by dietary indiscretion—the person eats food that is irritating, too highly seasoned, or contaminated with disease-causing microorganisms. Other causes of acute gastritis include overuse of aspirin and other nonsteroidal anti-inflammatory drugs (NSAIDs), excessive alcohol intake, bile reflux, and radiation therapy. A more severe form of acute gastritis is caused by the ingestion of strong acid or alkali, which may cause the mucosa to become gangrenous or to perforate.

A nurse reviews with the client the various types of medications used to treat diabetes. Which statement will the nurse use when teaching the client about liraglutide's mechanism of action? "It stimulates central noradrenergic receptors." "It diminishes intestinal absorption and metabolism of fats." "It stimulates central 5-HT2C receptors." "It causes delayed gastric emptying."

"It causes delayed gastric emptying." Liraglutide (Saxenda), a GLP-1 receptor agonist, delays gastric emptying, curbing appetite. Gastrointestinal lipase inhibitors (orlistat/Xenical), diminishes intestinal absorption and metabolism of fats. The selective serotonergic 5-HT2C receptor agonist stimulates central 5-HT2C receptors, causing appetite suppression. Sympathomimetic amines stimulate central noradrenergic receptors, causing appetite suppression.

A client asks the nurse why the physician ordered the blood test carcinoembryonic antigen (CEA). The nurse answers: "It detects a protein normally found in the blood." "It indicates if a cancer is present." "It determines functionality of the liver." "It tells the physician what type of cancer is present."

"It indicates if a cancer is present." Explanation: The carcinoembryonic antigen (CEA) blood test detects the presence of cancer by identifying the presence of a protein not normally detected in the blood of a healthy person. However, it does not indicate what type of cancer is present nor does it detect the functionality of the liver.

The nurse is conducting a health instruction program on oral cancer. The nurse determines that the participants understand the instructions when they state "A typical lesion is soft and craterlike." "Most oral cancers are painful at the outset." "Many oral cancers produce no symptoms in the early stages." "Blood testing is used to diagnose oral cancer."

"Many oral cancers produce no symptoms in the early stages." The most frequent symptom of oral cancer is a painless sore that does not heal. The client may complain of tenderness and difficulty chewing, swallowing, or speaking as the cancer progresses. Biopsy is used to diagnose oral cancer. A typical lesion in oral cancer is a painless, hardened ulcer with raised edges.

A nurse is teaching a client who has experienced an episode of acute gastritis. The nurse knows further education is necessary when the client makes which statement? "My appetite should come back tomorrow." "Once I can eat again, I should stick with bland foods." "I should limit alcohol intake, at least until symptoms subside." "I should feel better in about 24 to 36 hours."

"My appetite should come back tomorrow." The gastric mucosa is capable of repairing itself after an episode of gastritis. As a rule, the client recovers in about 1 day, although the appetite may be diminished for an additional 2 or 3 days. Acute gastritis is also managed by instructing the client to refrain from alcohol and food until symptoms subside. When the client can take nourishment by mouth, a nonirritating diet is recommended.

The nurse is caring for a client who is scheduled for the creation of an ileal conduit. Which statement by the client provides evidence that client teaching was effective? "A catheter will drain urine directly from my kidney." "My urine will be eliminated with my feces." "I will not need to worry about being incontinent of urine." "My urine will be eliminated through a stoma."

"My urine will be eliminated through a stoma." An ileal conduit is a non-continent urinary diversion whereby the ureters drain into an isolated section of ileum. A stoma is created at one end of the ileum, exiting through the abdominal wall.

A nurse prepares community teaching on healthy lifestyle modifications to a group of older adults. When discussing obesity rates of older adults in comparison with the rest of the population, what will the nurse include? "Older adults have a slightly higher prevalence of obesity in comparison to the general population." "Older adults have the same prevalence of obesity in comparison to the general population." "Older adults have a greatly reduced prevalence of obesity in comparison to the gener

"Older adults have a slightly higher prevalence of obesity in comparison to the general population." Older adults have a slightly higher prevalence of obesity when compared to the general population.

A client with obesity is prescribed orlistat for weight loss. The client asks the nurse, "I understand the medication prevents digestion of fat, but what happens if I eat fat?" What is the nurse's best response? "The fat is absorbed in your intestines." "The fat is excreted in your urine." "The fat remains undigested in your stomach." "The fat is passed in your stools."

"The fat is passed in your stools." Orlistat (Xenical) prevents the absorption of 30% of fat, decreasing caloric intake. Undigested fat is passed in the stools. The undigested fat is not excreted in the urine, absorbed in the intestines, or left undigested in the stomach.

A client with a peptic ulcer is diagnosed with Helicobacter pylori infection. The nurse is teaching the client about the medications prescribed, including metronidazole, omeprazole, and clarithromycin. Which statement by the client indicates the best understanding of the medication regimen? "These medications will coat the ulcer and decrease the acid production in my stomach." "The medications will kill the bacteria and stop the acid production." "I should take these medications only when I hav

"The medications will kill the bacteria and stop the acid production." Currently, the most commonly used therapy for peptic ulcers is a combination of antibiotics, proton-pump inhibitors, and bismuth salts that suppress or eradicate H. pylori. Recommended therapy for 10 to 14 days includes triple therapy with two antibiotics (e.g., metronidazole [Flagyl] or amoxicillin [Amoxil] and clarithromycin [Biaxin]) plus a proton-pump inhibitor (e.g., lansoprazole [Prevacid], omeprazole [Prilosec], or rabeprazole [Aciphex]), or quadruple therapy with two antibiotics (metronidazole and tetracycline) plus a proton-pump inhibitor and bismuth salts (Pepto-Bismol). Research is being conducted to develop a vaccine against H. pylori.

A client is scheduled for a Roux-en-Y bariatric surgery. When teaching the client about the surgical procedure, which statement will the nurse use? "The stomach is stapled to a very small pouch and the entire small intestine is rerouted. "The stomach is stapled to create a very small pouch and part of the small intestine is rerouted." "A prosthetic device binds the stomach and creates a very small pouch and restricts oral intake." "85% of the stomach is removed surgically, leaving a much smalle

"The stomach is stapled to create a very small pouch and part of the small intestine is rerouted." In Roux-en-Y bariatric surgery, a horizontal row of staples across the fundus of the stomach creates a pouch with a capacity of 20 to 30 mL. The jejunum is divided distal to the ligament of Treitz, and the distal end is anastomosed to the new pouch. The proximal segment is anastomosed to the jejunum.

A client with obesity is prescribed lorcaserin for weight loss. The client reports dry mouth. What is the nurse's best response? "This is an expected finding with this medication." "Your dose may need to be adjusted." "How much water are drinking?" "Taking this medication with meals decreases this symptom."

"This is an expected finding with this medication." Lorcaserin (Belviq), a selective serotonergic 5-HT2C receptor agonist, causes dry mouth. This is an expected and normal finding. Increasing fluid intake does not make this symptom go away. The other answer choices are incorrect.

A client is prescribed flavoxate (Urispas) following cystoscopy. Which of the following instructions would the nurse give the client? "This medication will treat the blood in your urine." "This medication prevents urinary incontinence." "This medication prevents infection in your urinary tract" "This medication will relieve your pain."

"This medication will relieve your pain." Flavoxate (Urispas) is a antispasmodic agent used for the treatment of burning and pain of the urinary tract.

A client with urinary tract infection is prescribed phenazopyridine (Pyridium). Which of the following instructions would the nurse give the client? "This medication will prevent re-infection." "This medication will relieve your pain." "This medication should be taken at bedtime." "This will kill the organism causing the infection."

"This medication will relieve your pain." Phenazopyridine (Pyridium) is a urinary analgesic agent used for the treatment of burning and pain associated with UTIs.

A nurse cares for a client with a BMI of 36 kg/m2 and nonalcoholic fatty liver disease. The client asks the nurse if he is a candidate for bariatric surgery. How should the nurse respond to the client? "No, you do not have any qualifying criteria for bariatric surgery." "Yes, your BMI and chronic condition meets the criteria for bariatric surgery." "No, you have one qualifying condition but not the other; this excludes you from bariatric surgery." "Yes, your chronic condition meets the criteria

"Yes, your BMI and chronic condition meets the criteria for bariatric surgery." The client's BMI of > 35 kg/m2 and a more severe obesity-associated comorbid condition, makes the client a candidate for bariatric surgery.

A client is scheduled for a renal ultrasound. Which of the following would the nurse include when explaining this procedure to the client? "An x-ray will be done to view your kidneys, ureters, and bladder." "A contrast medium will be used to help see the structures better." "You don't need to do any fasting before this noninvasive test." "You'll have a pressure dressing on your groin after the test."

"You don't need to do any fasting before this noninvasive test." Renal ultrasonography identifies the kidney's shape, size, location, collecting systems, and adjacent tissues. It is not invasive, does not require the injection of a radiopaque dye, and does not require fasting or bowel preparation. An x-ray of the abdomen to view the kidneys, ureters, and bladder is called a KUB. A contrast medium is used for computed tomography of the abdomen and pelvis. A pressure dressing is applied to the groin after a renal arteriogram.

Upon hearing that the small intestine lining has thinned, an elderly client asks, "What can this lead to?" What is the best response by the nurse? "At times you may see mucus in your stool." "You may frequently have diarrhea." "You may frequently experience constipation." "It is the aging process."

"You may frequently experience constipation." Explanation: As a person ages, the epithelial cells and villi thin in the small intestine. Implications of this consequence include decreased intestinal motility and transit time, which can lead to constipation. This would lead the nurse to discuss and advise the client on ways to prevent constipation.

A nurse cares for a female client of childbearing age who will undergo bariatric surgery. When teaching the client about precautions after surgery, which teaching will the nurse include that is specific to this population? "After surgery, your ability to conceive is decreased considerably." "After surgery, contraceptives have much less efficacy." "You should avoid pregnancy for at least 18 months after surgery." "You should avoid pregnancy for at least 9 months after surgery"

"You should avoid pregnancy for at least 18 months after surgery." When teaching a female of childbearing age regarding precautions after bariatric surgery, the nurse should instruct the client to avoid pregnancy for at least 18 months after surgery. The ability to conceive after weight loss surgery may improve more often than worsen. Contraceptives are no less effective after surgery than before.

A client presented with gastrointestinal bleeding 2 days ago and continues to have problems. The health care provider has ordered a visualization of the small intestine via a capsule endoscopy. What will the nurse include in the client education about this procedure? "A capsule will be inserted into your rectum." "An x-ray machine will use a capsule ray to follow your intestinal tract." "The health care provider will use a scope called a capsule to view your intestine." "You will need to swallo

"You will need to swallow a capsule." Explanation: A capsule endoscopy allows for noninvasive visualization of the small intestinal mucosa. The technique consists of the client swallowing a capsule that is embedded with a wireless miniature camera, which is propelled through the intestine by peristalsis. The capsule passes from the rectum in 1 to 2 days.

A nurse researches the cost and financial impact of obesity in America. What is the annual health care cost tied to obesity? $118 billion $147 billion $1 trillion $3 trillion

$147 billion The estimated annual health care costs in America tied to obesity is $147 billion.

Renal function results may be within normal limits until the GFR is reduced to less than which percentage of normal? 20 30 40 50

50 Renal function test results may be within normal limits until the GFR is reduced to less than 50% of normal.

When fluid intake is normal, the specific gravity of urine should be: 1.000 Less than 1.010 Greater than 1.025 1.010 to 1.025

1.010 to 1.025 Urine-specific gravity is a measurement of the kidneys' ability to concentrate urine. The specific gravity of water is 1.000. A urine-specific gravity less than 1.010 may indicate inadequate fluid intake. A urine-specific gravity greater than 1.025 may indicate overhydration.

The nurse is reviewing the results of a client's renal function study. The nurse understands that which value represent a normal BUN-to-creatinine ratio? 4:1 6:1 8:1 10:1

10:1 A normal BUN-to-creatinine ratio is about 10:1. The other values are incorrect.

A nurse is assisting the physician conducting a cystogram. The client has an intravenous (IV) infusion of D5W at 40 ml/hr. The physician inserts a urinary catheter into the bladder and instills a total of 350 ml of a contrast agent. The nurse empties 500 ml from the urinary catheter drainage bag at the conclusion of the procedure. How many milliliters does the nurse record as urine?

150 The urinary drainage bag contains both the contrast agent and urine at the conclusion of the procedure. Total contents (500 ml) in the drainage bag consist of 350 ml of contrast agent and 150 ml of urine.

A group of students is reviewing the process of urine elimination. The students demonstrate understanding of the process when they identify which amount of urine as triggering the reflex? 150 mL 350 mL 50 mL 250 mL

150 mL The desire to urinate comes from the feeling of bladder fullness. A nerve reflex is triggered when approximately 150 to 200 mL of urine accumulates.

Approximately what percentage of blood passing through the glomeruli is filtered into the nephron? 10 20 30 40

20 Under normal conditions, about 20% of the blood passing through the glomeruli is filtered into the nephron, amounting to about 180 L/day of filtrate.

Calculate the BMI of a client who is 6 feet 1 inch tall and weighs 200 pounds. Round to one decimal.

26.4 To calculate BMI, multiply weight in pounds by 703 and then divide that by height in inches squared

A client weighs 215 lbs and is 5' 8" tall. The nurse calculate this client's body mass index (BMI) as what? 32.7 44.9 24.8 19.5

32.7 Using the formula for BMI, the client's weight in pounds (215) is divided by the height in inches squared (68 inches squared) and then multiplied by 703. The result would be 32.7.

Calculate the BMI of a client who is 180 pounds and is 5 feet 2 inches tall. Round to one decimal point.

32.9 To calculate BMI, multiply weight in pounds by 703 and then divide that by height in inches squared.

A nurse caring for adults with obesity recognizes that obesity is classified based on BMI. Which BMI does the nurse recognize as Class II obesity? 40 kg/m2 29 kg/m2 34 kg/m2 35 kg/m2

35 kg/m2 Class I obesity is defined as 30-34.9 kg/m2. Class II obesity is defined as a BMI of 35-39.9 kg/m2. A BMI of 40 kg/m2 or greater defines Class III obesity.

When assessing a client during a routine checkup, the nurse reviews the history and notes that the client had aphthous stomatitis at the time of the last visit. How is aphthous stomatitis best described by the nurse? A canker sore of the oral soft tissues An acute stomach infection Acid indigestion An early sign of peptic ulcer disease

A canker sore of the oral soft tissues Aphthous stomatitis refers to a canker sore of the oral soft tissues, including the lips, tongue, and inside of the cheeks. Aphthous stomatitis isn't an acute stomach infection, acid indigestion, or early sign of peptic ulcer disease.

A nurse is preparing a presentation for a local community group of older adults about colon cancer. What would the nurse include as the primary characteristic associated with this disorder? Abdominal distention Abdominal pain Frank blood in the stool A change in bowel habits

A change in bowel habits Although abdominal distention and blood in the stool (frank or occult) may be present, the chief characteristic of cancer of the colon is a change in bowel habits, such as alternating constipation and diarrhea. Abdominal pain is a late sign.

A client has developed an anorectal abscess. Which client is at most risk for the development of this type of abscess? A client with diverticulosis A client with hemorrhoids A client with Crohn's disease A client with colon cancer

A client with Crohn's disease An anorectal abscess is common in clients with Crohn's disease. The other disorders do not predispose the client to risk for anorectal abscess.

A community health nurse is performing a home visit to a 53-year-old patient who requires twice-weekly wound care on her foot. The patient mentions that she is currently having hemorrhoids, a problem that she has not previously experienced. What treatment measure should the nurse recommend to this patient? Daily application of topical antibiotics Decreased fluid intake A high-fiber diet with increased fruit intake Bathing, rather than showering, once per day

A high-fiber diet with increased fruit intake Hemorrhoid symptoms and discomfort can be relieved by good personal hygiene and by avoiding excessive straining during defecation. A high-residue diet that contains fruit and bran along with an increased fluid intake may be all the treatment necessary to promote the passage of soft, bulky stools to prevent straining. It is unnecessary to avoid showering, and antibiotics are not an effective treatment.

After assessing a client with peritonitis, how would the nurse most likely document the client's bowel sounds? Mild High-pitched Hyperactive Absent

Absent Since lack of bowel motility typically accompanies peritonitis, bowel sounds are absent. Therefore, the nurse will not observe mild, high-pitched, or hyperactive bowel sounds.

Which of the following is the primary function of the small intestine? Peristalsis Digestion Absorption Secretion

Absorption Explanation: Absorption is the primary function of the small intestine. Digestion occurs in the stomach. Peristalsis occurs in the colon. The duodenum secretes enzymes.

A patient is scheduled for a Billroth I procedure for ulcer management. What does the nurse understand will occur when this procedure is performed? A partial gastrectomy is performed with anastomosis of the stomach segment to the duodenum. A sectioned portion of the stomach is joined to the jejunum. The antral portion of the stomach is removed and a vagotomy is performed. The vagus nerve is cut and gastric drainage is established.

A partial gastrectomy is performed with anastomosis of the stomach segment to the duodenum. A Billroth I procedure involves removal of the lower portion of the antrum of the stomach (which contains the cells that secrete gastrin) as well as a small portion of the duodenum and pylorus. The remaining segment is anastomosed to the duodenum.

A patient is scheduled for a Billroth I procedure for ulcer management. What does the nurse understand will occur when this procedure is performed? A sectioned portion of the stomach is joined to the jejunum. The vagus nerve is cut and gastric drainage is established. The antral portion of the stomach is removed and a vagotomy is performed. A partial gastrectomy is performed with anastomosis of the stomach segment to the duodenum.

A partial gastrectomy is performed with anastomosis of the stomach segment to the duodenum. A Billroth I procedure involves removal of the lower portion of the antrum of the stomach (which contains the cells that secrete gastrin) as well as a small portion of the duodenum and pylorus. The remaining segment is anastomosed to the duodenum.

The nurse is instructing the client on frequent sensations experienced when a contrast agent is injected into the body during diagnostic studies. Which sensation is most common? A warm sensation Chills Light-headedness Heart palpitations

A warm sensation Explanation: The nurse informs the client that he or she may experience a warm sensation and nausea when the contrast agent is instilled. The client is instructed to take a couple of deep breaths, and, many times, the sensation will go away. The other options are not frequently encountered.

The nurse is caring for a client recovering from an esophagogastroduodenoscopy (EGD). Which of the following client symptoms would require further nursing assessment? Thirst Drowsiness Sore throat Abdominal distention

Abdominal distention Explanation: The nurse is correct to fully assess the client experiencing abdominal distention following an esophagogastroduodenoscopy (EGD). Abdominal distention could indicate complications such as perforation and bleeding. The client experiences drowsiness from the sedative during the early recovery process and a sore throat from passage of the scope. The client may also experience thirst because the client has not had liquids for a period of time.

A client comes to the clinic after developing a headache, abdominal pain, nausea, hiccupping, and fatigue about 2 hours ago. The client tells the nurse that the last food was buffalo chicken wings and beer. Which medical condition does the nurse find to be most consistent with the client's presenting problems? Gastric cancer Duodenal ulcer Acute gastritis Gastric ulcer

Acute gastritis A client with acute gastritis may have a rapid onset of symptoms, including abdominal discomfort, headache, lassitude, nausea, anorexia, vomiting, and hiccupping, which can last from a few hours to a few days. Acute gastritis is often caused by dietary indiscretion-a person eats food that is irritating, too highly seasoned, or contaminated with disease-causing microorganisms. A client with a duodenal ulcer will present with heartburn, nausea, excessive gas and vomiting. A client with gastric cancer will have persistent symptoms of nausea and vomiting, not sudden symptoms. A client with a gastric ulcer will have bloating, nausea, and vomiting, but not necessarily hiccups.

A client comes to the emergency department complaining of severe pain in the right flank, nausea, and vomiting. The physician tentatively diagnoses right ureterolithiasis (renal calculi). When planning this client's care, the nurse should assign the highest priority to which nursing diagnosis? Impaired urinary elimination Risk for infection Acute pain Imbalanced nutrition: Less than body requirements

Acute pain Ureterolithiasis typically causes such acute, severe pain that the client can't rest and becomes increasingly anxious. Therefore, the nursing diagnosis of Acute pain takes highest priority. Diagnoses of Risk for infection and Impaired urinary elimination are appropriate when the client's pain is controlled. A diagnosis of Imbalanced nutrition: Less than body requirements isn't pertinent at this time.

A 24-hour urine collection is scheduled to begin at 8:00 am. When should the nurse initiate the procedure? With the first specimen voided after 8:00 am At 8:00 am, with or without a specimen 6 hours after the urine is discarded After discarding the 8:00 am specimen

After discarding the 8:00 am specimen A 24-hour collection of urine is the primary test of renal clearance used to evaluate how well the kidney performs this important excretory function. The client is initially instructed to void and discard the urine. The collection bottle is marked with the time the client voided. Thereafter, all the urine is collected for the entire 24 hours. The last urine is voided at the same time the test originally began.

Which hormone causes the kidneys to reabsorb sodium? Prostaglandins Aldosterone Antidiuretic hormone Growth hormone

Aldosterone Aldosterone is a hormone synthesized and released by the adrenal cortex. Antidiuretic hormone is secreted by the posterior pituitary gland. Growth hormone and prostaglandins do not cause the kidneys to reabsorb sodium.

The nurse practitioner suspects that a patient may have a gastric ulcer after completing a history and physical exam. Select an indicator that can be used to help establish the distinction. Amount of hydrochloric acid (HCL) secretion in the stomach Patient's age Presence of H. pylori Sensitivity to the use of nonsteroidal anti-inflammatory drugs (NSAIDs)

Amount of hydrochloric acid (HCL) secretion in the stomach A duodenal ulcer is characterized by hypersecretion of stomach acid, whereas a gastric ulcer evidences hyposecretion of stomach acid. The other three choices have similar characteristics in both types of ulcers.

The nurse is caring for a patient who has been NPO for 2 days pending a diagnostic procedure that has been repeated cancelled. When evaluating this patient's urinalysis, what would the nurse anticipate? A fluctuating urine specific gravity A decreased urine specific gravity A fixed urine specific gravity An increased urine specific gravity

An increased urine specific gravity Urine specific gravity depends largely on hydration status. A decrease in fluid intake (such as a "nothing by mouth" status) will lead to an increase in the urine specific gravity. With high fluid intake, specific gravity decreases. In patients with kidney disease, urine specific gravity does not vary with fluid intake, and the patient's urine is said to have a fixed specific gravity.

The nurse is performing a rectal assessment and notices a longitudinal tear or ulceration in the lining of the anal canal. The nurse documents the finding as which condition? Anal fistula Anorectal abscess Hemorrhoid Anal fissure

Anal fissure Fissures are usually caused by the trauma of passing a large, firm stool or from persistent tightening of the anal canal secondary to stress or anxiety (leading to constipation). An anorectal abscess is an infection in the pararectal spaces. An anal fistula is a tiny, tubular, fibrous tract that extends into the anal canal from an opening located beside the anus. A hemorrhoid is a dilated portion of vein in the anal canal.

Which of the following diagnostic tests would the nurse expect to be ordered to determine the details of the arterial supply to the kidneys? Radiography Angiography Cystoscopy Computed tomography (CT scan)

Angiography Angiography provides the details of the arterial supply to the kidneys, specifically the number and location of renal arteries. Radiography shows the size and position of the kidneys, ureters, and bladder. A CT scan is useful in identifying calculi, congenital abnormalities, obstruction, infections, and polycystic diseases. Cystoscopy is used for providing a visual examination of the internal bladder.

The nurse is employed in a urologist's office. Which classification of medication is anticipated for clients having difficulty with urinary incontinence? Cholinergic Anticholinergic Diuretics Anticonvulsant

Anticholinergic Pharmacologic agents that can improve bladder retention, emptying, and control include anticholinergic drugs. In this classification are medications such as Detrol, Ditropan, and Urecholine. Diuretics eliminate fluid from the body but do not affect the muscles of urinary elimination. Anticonvulsant and cholinergic medications also do not directly help with control.

The nurse is assessing a patient upon admission to the hospital. What significant nursing assessment data is relevant to renal function? Select all that apply. Any voiding disorders The patient's financial status The ability of the patient to manage activities of daily living The patient's occupation The presence of hypertension or diabetes

Any voiding disorders The presence of hypertension or diabetes When obtaining the health history, the nurse should inquire about the following: dysuria (painful or difficult urination), as well as when during voiding (i.e., at initiation or at termination of voiding) this occurs; occupational, recreational, or environmental exposure to chemicals (plastics, pitch, tar, rubber); hypertension; or diabetes.

An ileal conduit is created for a client after a radical cystectomy. Which of the following would the nurse expect to include in the client's plan of care? Irrigating the urinary diversion Intermittent catheterizations Exercises to promote sphincter control Application of an ostomy pouch

Application of an ostomy pouch An ileal conduit involves care of a urinary stoma, much like that of a fecal stoma, including the application of an ostomy pouch, skin protection, and stoma care. Intermittent catheterizations and irrigations are appropriate for a continent urinary diverse such as a Kock or Indiana pouch. Exercises to promote sphincter control are appropriate for an ureterosigmoidoscopy.

A nurse is completing an assessment on a client with a postoperative neck dissection. The nurse notices excessive bleeding from the dressing site and suspects possible carotid artery rupture. What action should the nurse take first? Elevate the head of the patient's bed Apply pressure to the bleeding site Summon assistance Notify the surgeon to repair the vessel

Apply pressure to the bleeding site The first action for the nurse is to apply pressure to the bleeding site. The nurse will need to obtain assistance, elevate the head of the bed, and notify the surgeon, but client care is most important initially.

A nurse caring for a client who has had radical neck surgery notices an abnormal amount of serosanguineous secretions in the wound suction unit during the first postoperative day. What is an expected, normal amount of drainage? Greater than 160 mL Approximately 80 to 120 mL Between 40 and 80 mL Between 120 and 160 mL

Approximately 80 to 120 mL Between 80 to 120 mL may drain over the first 24 hours. Drainage of greater than 120 mL may be indicative of a chyle fistula or hemorrhage.

The nurse is preparing the client for an assessment of the abdomen. What should the nurse complete prior to this assessment? Ask the client to empty the bladder. Prepare for a prostate examination. Dim the lights for privacy. Assist the client to a Fowler's position.

Ask the client to empty the bladder. Explanation: The physical examination of the gastrointestinal system includes assessment of the mouth, abdomen, and rectum. It requires good light, full exposure of the abdomen, warm hands with short nails, and a relaxed client with an empty bladder. A full bladder will interfere with inspection and may elicit discomfort with palpation and percussion, thereby altering results.

A client with obesity is suspected of having nonalcoholic fatty liver disease. Which diagnostic labs does the nurse anticipate the client needing? Select all that apply. Triglycerides Glycosylated hemoglobin Aspartate aminotransferase Alanine aminotransferase Fasting glucose

Aspartate aminotransferase Alanine aminotransferase Aspartate aminotransferase (AST) and alanine aminotransferase (ALT) are two liver function tests that will be used in diagnosing nonalcoholic fatty liver disease, a complication of obesity. Triglycerides, glycosylated hemoglobin (HbgA1C), and fasting glucose are diagnostic tests; however, these are not generally required in diagnosing nonalcoholic fatty liver disease.

A client in the emergency department reports that a piece of meat became stuck in the throat while eating. The nurse notes the client is anxious with respirations at 30 breaths/min, frequent swallowing, and little saliva in the mouth. An esophagogastroscopy with removal of foreign body is scheduled for today. What would be the first activity performed by the nurse? Assess lung sounds bilaterally. Obtain consent for the esophagogastroscopy. Suction the oral cavity of the client. Administer presc

Assess lung sounds bilaterally. All these activities are things the nurse may do for a client with a foreign body in the esophagus. This client is at risk for esophageal perforation, and thus pneumothorax. By auscultating lung sounds the nurse will be able to assess if a pneumothorax is present. The client has little saliva in the oral cavity and does not need to be suctioned. A client may also report pain with a foreign body. However, ABCs (airway, breathing, circulation) take priority. The consent for the esophagogastroscopy may be obtained after the nurse has completed the client assessment.

After undergoing renal arteriogram, in which the left groin was accessed, a client complains of left calf pain. Which intervention should the nurse perform first? Assess peripheral pulses in the left leg. Exercise the leg and foot. Place cool compresses on the calf. Assess for anaphylaxis.

Assess peripheral pulses in the left leg. The nurse should begin by assessing peripheral pulses in the left leg to determine if blood flow was interrupted by the procedure. The client may also have thrombophlebitis. Cool compresses aren't used to relieve pain and inflammation in thrombophlebitis. The leg should remain straight after the procedure. Calf pain isn't a symptom of anaphylaxis.

A patient had a renal angiography and is being brought back to the hospital room. What nursing interventions should the nurse carry out after the procedure to detect complications? Select all that apply. Examine the puncture site for swelling and hematoma formation. Assess peripheral pulses. Apply warm compresses to the insertion site to decrease swelling. Increase the amount of IV fluids to prevent clot formation. Compare color and temperature between the involved and uninvolved extremities.

Assess peripheral pulses. Compare color and temperature between the involved and uninvolved extremities. Examine the puncture site for swelling and hematoma formation. After the procedure, vital signs are monitored until stable. If the axillary artery was the injection site, blood pressure measurements are taken on the opposite arm. The injection site is examined for swelling and hematoma. Peripheral pulses are palpated, and the color and temperature of the involved extremity are noted and compared with those of the uninvolved extremity. Cold compresses may be applied to the injection site to decrease edema and pain.

When caring for a client with an acute exacerbation of a peptic ulcer, the nurse finds the client doubled up in bed with severe pain in the right shoulder. What is the initial appropriate action by the nurse? Notify the health care provider. Irrigate the client's NG tube. Place the client in the high-Fowler's position. Assess the client's abdomen and vital signs.

Assess the client's abdomen and vital signs. Signs and symptoms of perforation includes sudden, severe upper abdominal pain (persisting and increasing in intensity); pain may be referred to the shoulders, especially the right shoulder, because of irritation of the phrenic nerve in the diaphragm. The nurse should assess the vital signs and abdomen prior to notifying the physician. Irrigation of the NG tube should not be performed because the additional fluid may be spilled into the peritoneal cavity, and the client should be placed in a position of comfort, usually on the side with the head slightly elevated.

When caring for a client with an acute exacerbation of a peptic ulcer, the nurse finds the client doubled up in bed with severe pain in the right shoulder. What is the initial appropriate action by the nurse? Place the client in the high-Fowler's position. Assess the client's abdomen and vital signs. Notify the health care provider. Irrigate the client's NG tube.

Assess the client's abdomen and vital signs. Signs and symptoms of perforation includes sudden, severe upper abdominal pain (persisting and increasing in intensity); pain may be referred to the shoulders, especially the right shoulder, because of irritation of the phrenic nerve in the diaphragm. The nurse should assess the vital signs and abdomen prior to notifying the physician. Irrigation of the NG tube should not be performed because the additional fluid may be spilled into the peritoneal cavity, and the client should be placed in a position of comfort, usually on the side with the head slightly elevated.

A client is postoperative following a graft reconstruction of the neck. What intervention is the most important for the nurse to complete with the client? Assess the graft for color and temperature. Administer prescribed intravenous vancomycin at the correct time. Cleanse around the drain using aseptic technique. Reinforce the neck dressing when blood is present on the dressing.

Assess the graft for color and temperature. Assessing the graft for color and temperature addresses circulation and is most important for the nurse to complete. Reinforcing the neck dressing is important, but not the priority. Administering medication and cleansing the drain site are not most important interventions with the client after graft reconstruction of the neck.

Following a renal biopsy, a client reports severe pain in the back, the arms, and the shoulders. Which intervention should be offered by the nurse? Assess the patient's back and shoulder areas for signs of internal bleeding. Distract the client's attention from the pain. Provide analgesics to the client. Enable the client to sit up and ambulate.

Assess the patient's back and shoulder areas for signs of internal bleeding. After a renal biopsy, the client should be on bed rest. The nurse observes the urine for signs of hematuria. It is important to assess the dressing frequently for signs of bleeding, monitor vital signs, and evaluate the type and severity of pain. Severe pain in the back, shoulder, or abdomen may indicate bleeding. In such a case, the nurse should notify the physician about these signs and symptoms. The nurse should also assess the client for difficulty voiding and encourage adequate fluid intake.

A client is frustrated and embarrassed by urinary incontinence. Which measure should the nurse include in a bladder retraining program? Encouraging the client to increase the time between voidings Assessing present voiding patterns Establishing a predetermined fluid intake pattern for the client Restricting fluid intake to reduce the need to void

Assessing present voiding patterns The guidelines for initiating bladder retraining include assessing the client's present intake patterns, voiding patterns, and reasons for each accidental voiding. Lowering the client's fluid intake won't reduce or prevent incontinence. The client should be encouraged to drink 1.5 to 2 L of water per day. A voiding schedule should be established after assessment.

A client with anorexia reports constipation. Which nursing measure would be most effective in helping the client reduce constipation? Assist client to increase dietary fiber. Obtain complete food history. Provide adequate quantity of food. Obtain medical and allergy history.

Assist client to increase dietary fiber. The nurse should assist the client to increase the dietary fiber in food because it helps reduce constipation. Providing an adequate quantity of food is necessary in maintaining sufficient nutrition and in sustaining normal body weight. Obtaining medical, allergy, and food history would provide valuable information, however, it would not help reduce constipation.

Following a cystoscopy, the client has a nursing diagnosis of acute pain related to the trauma of the procedure to the urinary tract. An appropriate nursing intervention is to: Administer prescribed antibiotics. Apply moist heat to the flank area. Monitor for urinary retention. Assist with warm sitz baths.

Assist with warm sitz baths. Acute pain can be relieved with warm sitz baths. The nurse should monitor the client for urinary retention, which can help detect a potential cause of pain, but this nursing action does not relieve pain. Antibiotics may be prescribed to prevent infection. The pain associated with cystoscopy tends to be confined to the perineal area and lower abdomen not the flank area.

An older adult client is admitted to an acute care facility for treatment of an acute flare-up of a chronic gastrointestinal condition. In addition to assessing the client for complications of the current illness, the nurse monitors for age-related changes in the gastrointestinal tract. Which age-related change increases the risk of anemia? Increase in bile secretion Decrease in intestinal flora Atrophy of the gastric mucosa Dulling of nerve impulses

Atrophy of the gastric mucosa Explanation: Atrophy of the gastric mucosa reduces hydrochloric acid secretion; this, in turn, impairs absorption of iron and vitamin B12, increasing the risk of anemia as a person ages. A decrease in hydrochloric acid increases, not decreases, intestinal flora; as a result, the client is at increased risk for infection, not anemia. A reduction, not increase, in bile secretion may lead to malabsorption of fats and fat-soluble vitamins. Dulling of nerve impulses associated with aging increases the risk of constipation, not anemia.

When assisting with preparing a client scheduled for a barium swallow, which of the following would be appropriate to include? Take vitamin K before the procedure. Take three cleansing enemas before the procedure. Avoid the intake of red meat before the procedure. Avoid smoking for at least 12 to 24 hours before the procedure.

Avoid smoking for at least 12 to 24 hours before the procedure. Explanation: The nurse should instruct the client to avoid smoking for at least a day before the procedure of barium swallow because smoking stimulates gastric motility. The client is advised to take vitamin K before a liver biopsy and instructed to take three cleansing enemas before a barium enema. Instruction to avoid red meat would be appropriate for a client who is having a Hemoccult test.

The nurse instructs the client with gastroesophageal reflux disease (GERD) regarding dietary measures. Which action by the client demonstrates that the client has understood the recommended dietary changes? Eliminating spicy foods. Avoiding chocolate and coffee. Eliminating cucumbers and other foods with seeds. Avoiding steamed foods.

Avoiding chocolate and coffee. Chocolate, tea, cola, and caffeine lower esophageal sphincter pressure, thereby increasing reflux. Clients do not need to eliminate spicy foods unless such foods bother them. Foods with seeds are restricted in diverticulosis. Steamed foods are encouraged to retain vitamins and decrease fat intake.

A nurse cares for clients with obesity. Which clinical measurements use quantified measurements to diagnose obesity? Select all that apply. BMI Weight Waist circumference Blood pressure Total cholesterol

BMI Weight Waist circumference Weight, BMI, and waist circumference are used to measure obesity. Blood pressure and cholesterol may be used in the client with obesity; however, these do not diagnose obesity.

The nurse is creating a plan of care for a client who is not able to tolerate brushing his teeth. The nurse includes which mouth irrigation in the plan of care? Mouthwash and water Full-strength peroxide Dextrose and water Baking soda and water

Baking soda and water When a client is unable to tolerate teeth brushing, the following irrigating solutions are recommended: 1 tsp baking soda in 8 oz warm water, half-strength hydrogen peroxide, or normal saline solution.

A client is scheduled for removal of the lower portion of the antrum of the stomach and a small portion of the duodenum and pylorus. What surgical procedure will the nurse prepare the client for? Pyloroplasty Vagotomy Billroth II Billroth I

Billroth I A Billroth I is the removal of the lower portion (antrum) of the stomach (which contains the cells that secrete gastrin) as well as a small portion of the duodenum and pylorus. A vagotomy is a surgical dissection of the vagus nerve to decrease gastric acid. A pyloroplasty is a procedure to widen the pylorus. A Billroth II is the removal of the lower portion (antrum) of stomach with anastomosis to the jejunum.

Which of the following is considered the gold standard for the diagnosis of liver disease? Cholecystography Paracentesis Biopsy Ultrasonography

Biopsy Explanation: Liver biopsy is considered the gold standard for the diagnosis of liver disease. Paracentesis is the removal of fluid (ascites) from the peritoneal cavity through a puncture or a small surgical incision through the abdominal wall under sterile conditions. Cholecystography and ultrasonography may be used to detect gallstones.

The nurse is to obtain a stool specimen from a client who reported that he is taking iron supplements. The nurse would expect the stool to be which color? Green Black Dark brown Red

Black Explanation: Ingestion of iron can cause the stool to turn black. Meat protein causes stool to appear dark brown. Ingestion of large amounts of spinach may turn stool green while ingestion of carrots and beets may cause stool to turn red.

A client is admitted to the health care facility with a diagnosis of a bleeding gastric ulcer. The nurse expects the client's stools to have which description? Coffee-ground-like Clay-colored Black and tarry Bright red

Black and tarry Black, tarry stools are a sign of bleeding high in the GI tract, as from a gastric ulcer, and result from the action of digestive enzymes on the blood. Vomitus associated with upper GI tract bleeding commonly is described as coffee-ground-like. Clay-colored stools are associated with biliary obstruction. Bright red stools indicate lower GI tract bleeding.

As women age, many experience an increased sense of urgency to void, as well as an increased risk of incontinence. This is usually the result of age-related changes in which part of the renal system? Kidney Nephron Tubule system Bladder

Bladder With increased age, bladder tone and capacity is decreased. In women, this is compounded by a decrease in estrogen, which causes changes to the urethral sphincter.

Behavioral interventions for urinary incontinence can be coordinated by a nurse. A comprehensive program that incorporates timed voiding and urinary urge inhibition is referred to as what? Prompted voiding Interval voiding Voiding at given intervals Bladder retraining

Bladder retraining Bladder retraining includes a timed voiding schedule and urinary urge inhibition exercises. These exercises involve delaying voiding to help the patient stay dry for a set period of time. When one time interval is reached, another is set. The time is usually increased by 10 to 15 minutes, until an acceptable voiding interval is achieved.

The nurse is providing care to a client who has had a kidney biopsy. The nurse would need to be alert for signs and symptoms of which of the following? Infection Allergic reaction Bleeding Dehydration

Bleeding Renal biopsy carries the risk of post procedure bleeding, because the kidneys receive up to 25% of the cardiac output each minute. Therefore, the nurse would need to be alert for signs and symptoms of bleeding. Although infection is also a risk, the risk for bleeding is greater. Dehydration and allergic reaction are not associated with a renal biopsy.

The nurse is providing care to a client who has had a renal biopsy. The nurse would need to be alert for signs and symptoms of which of the following? Dehydration Allergic reaction Bleeding Infection

Bleeding Renal biopsy carries the risk of postprocedure bleeding because the kidneys receive up to 25% of the cardiac output each minute. Therefore, the nurse would need to be alert for signs and symptoms of bleeding. Although infection is also a risk, the risk for bleeding is greater. Dehydration and allergic reaction are not associated with a renal biopsy.

Which of the following would a nurse expect to assess in a client with peritonitis? Hyperactive bowel sounds Board-like abdomen Decreased pulse rate Deep slow respirations

Board-like abdomen The client with peritonitis would typically exhibit a rigid, board-like abdomen, with absent bowel sounds, elevated pulse rate, and rapid, shallow respirations.

The nurse is performing an abdominal assessment for a patient with diarrhea and auscultates a loud rumbling sound in the left lower quadrant. What will the nurse document this sound as on the nurse's notes? Tenesmus Borborygmus Peristalsis Loud bowel sounds

Borborygmus Borborygmus is a rumbling noise caused by the movement of gas through the intestines, often associated with diarrhea.

The nurse is performing and documenting the findings of an abdominal assessment. When the nurse hears intestinal rumbling and the client then experiences diarrhea, the nurse documents the presence of which condition? Borborygmus Azotorrhea Diverticulitis Tenesmus

Borborygmus Borborygmus is the intestinal rumbling caused by the movement of gas through the intestines that accompanies diarrhea. Tenesmus refers to ineffectual straining at stool. Azotorrhea refers to excess of nitrogenous matter in the feces or urine. Diverticulitis refers to inflammation of a diverticulum from obstruction (by fecal matter) resulting in abscess formation.

A nurse researcher is reviewing data obtained from a developing nation on nutrition and metabolism issues facing that country. What is the nurse's understanding of the "double-burden" many developing nations now face? Both obesity and scare food sources Both undernutrition and sedentary lifestyles Both undernutrition and obesity Both low metabolism and high metabolism

Both undernutrition and obesity The WHO mentions that many developing nations now face a double-burden of both undernutrition and obesity. Both of these issues occur simultaneously and create a public health burden to developing nations.

During a colonoscopy with moderate sedation, the patient groans with obvious discomfort and begins bleeding from the rectum. The patient is diaphoretic and has an increase in abdominal girth from distention. What complication of this procedure is the nurse aware may be occurring? Rectal fissure Colonic polyp Bowel perforation Infection

Bowel perforation Explanation: Immediately after the test, the patient is monitored for signs and symptoms of bowel perforation (e.g., rectal bleeding, abdominal pain or distention, fever, focal peritoneal signs).

Which clinical manifestation is not associated with hemorrhage? Tachypnea Hypotension Bradycardia Tachycardia

Bradycardia Hemorrhage may occur from carotid artery rupture as a result of necrosis of the graft or damage to the artery itself from tumor or infection. Tachycardia, tachypnea, and hypotension may indicate hemorrhage and impending hypovolemic shock.

In women, which of the following types of cancer exceeds colorectal cancer? Breast Skin Liver Lung

Breast In women, only incidences of breast cancer exceed that of colorectal cancer. In men, only incidences of prostate cancer and lung cancer exceed that of colorectal cancer.

A patient with IBD would be encouraged to increase fluids, use vitamins and iron supplements, and follow a diet designed to reduce inflammation. Select the meal choice that would be recommended for a low-residue diet. A peanut butter sandwich and fruit cup A fruit salad with yogurt Broiled chicken with low-fiber pasta Salami on whole grain bread and V-8 juice

Broiled chicken with low-fiber pasta A low-residue, high-protein, and high-calorie diet is recommended to reduce the size and number of stools. Foods to avoid include yogurt, fruit, salami, and peanut butter.

A nurse is conducting a health history on a patient who is seeing her health care provider for symptoms consistent with a UTI. The nurse understands that the most common route of infection is which of the following? Due to a fistula (direct extension) Through the bloodstream (hematogenous spread) The result of urethra abrasion (sexual intercourse) By ascending infection (transurethral)

By ascending infection (transurethral) The most common route of infection is transurethral, in which bacteria colonize the periurethral area and enter the bladder by means of the urethra.

Which term describes a reddened, circumscribed lesion that ulcerates and becomes crusted and is a primary lesion of syphilis? Lichen planus Actinic cheilitis Chancre Leukoplakia

Chancre A chancre is a reddened circumscribed lesion that ulcerates and becomes crusted and is a primary lesion of syphilis. Lichen planus is a white papule at the intersection of a network of interlacing lesions. Actinic cheilitis is an irritation of the lips associated with a scaling, crusting fissure. Leukoplakias are white patches usually found in the buccal mucosa

Which objective symptom of a UTI is most common in older adults, especially those with dementia? Change in cognitive functioning Incontinence Back pain Hematuria

Change in cognitive functioning The most common objective finding is a change in cognitive functioning, especially in those with dementia, because these clients usually exhibit even more profound cognitive changes with the onset of a UTI. Incontinence, hematuria, and back pain are not the most common presenting objective symptoms.

The nurse working with a client after an ileal conduit notices that the pouching system is leaking small amounts of urine. What is the appropriate nursing intervention? Secure or patch it with tape. Empty the pouch. Secure or patch it with barrier paste. Change the wafer and pouch.

Change the wafer and pouch. Whenever a leaking pouching system is noted, the nurse should change the wafer and pouch. Attempting to secure or patch the leak with tape and/or barrier paste can trap urine under the barrier or faceplate, which will compromise peristomal skin integrity. Emptying the pouch will not rectify the leaking.

When examining the skin of a client who is dehydrated due to fluid losses from the gastrointestinal tract, which of the following would be most important? Checking if the mucous membranes are dry Examining the sclera if it is yellow Observing for distended abdominal veins Checking if the skin is discolored

Checking if the mucous membranes are dry Explanation: Mucous membranes may be dry, and skin turgor may be poor in clients suffering from dehydration as a result of fluid losses from the GI tract. Checking the skin for discoloration and inspecting the sclera if it is yellow is taken into consideration when the client could have symptoms of jaundice, not fluid losses. Distended abdominal veins are not associated with dehydration.

A client informs the nurse that he has been having abdominal pain that is relieved when having a bowel movement. The client states that the physician told him he has irritable bowel syndrome. What does the nurse recognize as characteristic of this disorder? Blood and mucus in the stool Client is awakened from sleep due to abdominal pain. Weight loss due to malabsorption Chronic constipation with sporadic bouts of diarrhea

Chronic constipation with sporadic bouts of diarrhea Most clients with irritable bowel syndrome (IBS) describe having chronic constipation with sporadic bouts of diarrhea. Some report the opposite pattern, although less commonly. Most clients experience various degrees of abdominal pain that defecation may relieve. Weight usually remains stable, indicating that when diarrhea occurs, malabsorption of nutrients does not accompany it. Stools may have mucus, but blood is not usually found because the bowel is not locally inflamed. The sleep is not disturbed from abdominal pain.

Which is an accurate statement regarding cancer of the esophagus? It is three times more common in women than men in the United States . It usually occurs in the fourth decade of life. Chronic irritation of the esophagus is a known risk factor. It is seen more frequently in Caucasian Americans than in African America

Chronic irritation of the esophagus is a known risk factor. In the United States, cancer of the esophagus has been associated with the ingestion of alcohol and the use of tobacco. In the United States, carcinoma of the esophagus occurs more than three times more often in men as in women. It is seen more frequently in African Americans than in Caucasian Americans. It usually occurs in the fifth decade of life.

Which is an accurate statement regarding cancer of the esophagus? It is three times more common in women than men in the United States. It usually occurs in the fourth decade of life. It is seen more frequently in Caucasian Americans than in African Americans. Chronic irritation of the esophagus is a known risk factor.

Chronic irritation of the esophagus is a known risk factor. In the United States, cancer of the esophagus has been associated with the ingestion of alcohol and the use of tobacco. In the United States, carcinoma of the esophagus occurs more than three times more often in men as in women. It is seen more frequently in African Americans than in Caucasian Americans. It usually occurs in the fifth decade of life.

The nurse is irrigating a colostomy when the patient says, "You will have to stop, I am cramping so badly." What is the priority action by the nurse? Clamp the tubing and give the patient a rest period. Inform the patient that it will only last a minute and continue with the procedure. Stop the irrigation and remove the tube. Replace the fluid with cooler water since it is probably too warm.

Clamp the tubing and give the patient a rest period. When irrigating a colostomy, the nurse should allow tepid fluid to enter the colon slowly. If cramping occurs, the nurse should clamp off the tubing and allow the patient to rest before progressing. Water should flow in over a 5- to 10-minute period.

A nurse cares for a client who is 5 feet 11 inches tall and weighs 225 pounds. What statement describes the client's BMI? Class I obesity Class II obesity Overweight Normal weight

Class I obesity To calculate BMI, multiply weight in pounds by 703 and then divide that by height in inches squared. The client's BMI is 31.4 kg/m2. This falls under the Class I obesity category. Normal weight BMI is 18.5-25 kg/m2. Overweight BMI is 25-30 kg/m2. Class II obesity is a BMI 35-40 kg/m2.

Which nursing assessment finding indicates the client with renal dysfunction has not met expected outcomes? Urine output is 100 ml/hr. Client rates pain at a 3 on a scale of 0 to 10. Client denies frequency and urgency. Client reports increasing fatigue.

Client reports increasing fatigue. Fatigue, shortness of breath, and exercise intolerance are consistent with unexplained anemia, which can be secondary to gradual renal dysfunction.

A female client who is diagnosed with a malignant tumor in her bladder is advised to undergo cystectomy followed by a urinary diversion procedure. Which of the following would be most important for the nurse to assess preoperatively? History of allergy to iodine and seafood Client's manual dexterity and vision Dietary habits involving cholesterol-laden food Menstrual history

Client's manual dexterity and vision It is essential to assess manual dexterity, vision, and level of understanding of a client who undergoes a urinary diversion procedure, because this information will determine the client's ability to manage stoma care and self-catheterization following the urinary diversion procedure. The client's history of allergy to iodine and seafood, dietary habits related to high cholesterol intake, and menstrual history are not important factors for this situation.

The nurse is caring for a client with chronic gastritis. The nurse monitors the client knowing that this client is at risk for which vitamin deficiency? Vitamin C Vitamin A Vitamin E Vitamin B12

Clients with chronic gastritis from vitamin deficiency usually have evidence of malabsorption of vitamin B12 caused by the production of antibodies that interfere with the binding of vitamin B12 to intrinsic factor. However, some clients with chronic gastritis have no symptoms. Vitamins A, C, and E are not affected by gastritis.

A patient comes to the clinic suspecting a possible UTI. What symptoms of a UTI would the nurse recognize from the assessment data gathered? Rebound tenderness at McBurney's point Urine with a specific gravity of 1.005-1.022 Cloudy urine An output of 200mL with each voiding

Cloudy urine The nurse should observe for signs and symptoms of UTI: cloudy malodorous urine, hematuria, fever, chills, anorexia, and malaise.

The nurse is encouraging the client with recurrent urinary tract infections to increase his fluid intake to 8 large glasses of fluids daily. The client states he frequently drinks water and all of the following. Which of the following would the nurse discourage for this client? Milk at lunch Coffee in the morning Fruit juice midmorning Ginger ale at dinner time

Coffee in the morning The nurse would discourage drinking coffee. Coffee, tea, alcohol, and colas are urinary tract irritants. Fruit juice, milk, and ginger ale are appropriate for drinking and countered toward the daily fluid total.

A creatinine clearance test has been ordered. The nurse prepares to: Collect the client's urine for 24 hours. Obtain a blood specimen. Obtain a clean catch urine. Insert a straight catheter for a specimen.

Collect the client's urine for 24 hours. A creatinine clearance test is a 24-hour urine test and is useful in evaluating renal disease.

A patient is suspected to have diverticulosis without symptoms of diverticulitis. What diagnostic test does the nurse anticipate educating the patient about prior to scheduling? Flexible sigmoidoscopy CT scan Barium enema Colonoscopy

Colonoscopy Diverticulosis is typically diagnosed by colonoscopy, which permits visualization of the extent of diverticular disease and biopsy of tissue to rule out other diseases. In the past, barium enema was the preferred diagnostic test, but it is now used less frequently than colonoscopy. CT with contrast agent is the diagnostic test of choice if the suspected diagnosis is diverticulitis; it can also reveal abscesses.

A client is scheduled for several diagnostic tests to evaluate gastrointestinal function. After teaching the client about these tests, the nurse determines that the client has understood the teaching when the client identifies which test as not requiring the use of a contrast medium? Computer tomography Upper GI series Colonoscopy Small bowel series

Colonoscopy Explanation: A colonoscopy is a direct visual examination of the entire large intestine. It does not involve the use of a contrast agent. Contrast medium may be used with a small bowel series, computed tomography, and upper GI series.

The nurse caring for a client is providing instructions for an upcoming renal angiography. Which nursing action, explained in the preoperative instructions, is essential in the postprocedure period? Assess cognitive status. Encourage voiding following the procedure. Assess renal blood work. Complete a pulse assessment of the legs and feet.

Complete a pulse assessment of the legs and feet. A renal angiography provides details about the arterial blood supply to the kidney. A catheter is passed up the femoral artery into the aorta in the area of the renal artery. After the procedure, a pressure dressing remains in place for several hours. It is essential that the nurse palpates pulses in the legs and feet at least every 1 to 2 hours for signs of arterial occlusion. Reviewing lab work is completed in the preoperative period. Voiding assesses renal status that provides additional data in the post procedural period. Assessing cognitive status is completed due to the sedative that is administered in the preprocedural period.

The nurse plans care for a client with obesity. What does the nurse recognize is the primary pathophysiological reason clients with obesity are at greater risk for developing thromboembolism? Increased blood viscosity Compromised peripheral blood flow Increased fat accumulation in the blood Impaired clotting

Compromised peripheral blood flow A client with obesity is at increased risk for developing thromboembolism due to compromised blood flow and resulting venous stasis. Although the client with obesity is at risk for high cholesterol levels, increased fat in the blood does not directly impact the risk for developing thromboembolism. Increased blood viscosity and impaired clotting do not typically occur in obesity and are not the reason a client with obesity would be at greater risk for developing thromboembolism.

The nurse is assessing a client at the diagnostic imaging center. For which diagnostic test would the client assess for an allergy to shellfish? Computed tomography with contrast Bladder ultrasonography Cystoscopy Radiography

Computed tomography with contrast The nurse is correct to assess for an allergy to shellfish most times when a contrast medium is ordered. The other options do not necessarily have a contrast medium.

A nurse is reviewing the history and physical of a client admitted for a hemorrhoidectomy. Which predisposing condition does the nurse expect to see? Constipation Hyperkalemia Lactic acidosis Hypoglycemia

Constipation Orthostatic hypertension and other conditions associated with persistently high intra-abdominal pressure (such as pregnancy) can lead to hemorrhoids. The passing of hard stools, not diarrhea, can aggravate hemorrhoids. Diverticulosis has no relationship to hemorrhoids. Rectal bleeding is a symptom of hemorrhoids, not a predisposing condition.

When describing the functions of the kidney to a client, which of the following would the nurse include? Secretion of enzymes Regulation of white blood cell production Control of water balance Synthesis of vitamin K

Control of water balance Functions of the kidneys include control of water balance and blood pressure, regulation of red blood cell production, synthesis of vitamin D to active form, and secretion of prostaglandins.

When describing the functions of the kidney to a client, which of the following would the nurse include? Select all that apply. Secretion of the enzyme renin Control of water balance Regulation of white blood cell production Synthesis of vitamin K

Control of water balance Secretion of the enzyme renin Functions of the kidneys include control of water balance and blood pressure, regulation of red blood cell production, synthesis of vitamin D to active form, and secretion of prostaglandins. They also produce the enzyme renin.

Which part of the kidney contains the nephrons? Pelvis Medulla Glomerulus Cortex

Cortex The cortex is located farthest from the center of the kidney and around the outermost edges. It contains the nephrons (the functional units of the kidney).

The client is admitted to the hospital with a diagnosis of acute pyelonephritis. Which clinical manifestations would the nurse expect to find? Costovertebral angle tenderness Perineal pain Suprapubic pain Pain after voiding

Costovertebral angle tenderness Acute pyelonephritis is characterized by costovertebral angle tenderness. Suprapubic pain is suggestive of bladder distention or infection. Urethral trauma and irritation of the bladder neck can cause pain after voiding. Perineal pain is experienced by male clients with prostate cancer or prostatitis.

A creatinine clearance test is ordered for a client with possible renal insufficiency. The nurse must collect which serum concentration midway through the 24-hour urine collection? Blood urea nitrogen Hemoglobin Creatinine Osmolality

Creatinine To calculate creatinine clearance, a 24-hour urine specimen is collected. The serum creatinine concentration is measured midway through the collection. The other concentrations are not measured during this test.

Which value does the nurse recognize as the best clinical measure of renal function? Urine-specific gravity Circulating ADH concentration Creatinine clearance Volume of urine output

Creatinine clearance Creatinine clearance is a good measure of the glomerular filtration rate (GFR), the amount of plasma filtered through the glomeruli per unit of time. Creatinine clearance is the best approximation of renal function. As renal function declines, both creatinine clearance and renal clearance (the ability to excrete solutes) decrease.

A client develops decreased renal function and requires a change in antibiotic dosage. On which factor should the physician base the dosage change? Liver function studies Therapeutic index Creatinine clearance GI absorption rate

Creatinine clearance The physician should base changes to antibiotic dosages on creatinine clearance test results, which gauge the kidney's glomerular filtration rate; this factor is important because most drugs are excreted at least partially by the kidneys. The GI absorption rate, therapeutic index, and liver function studies don't help determine dosage change in a client with decreased renal function.

A patient is being seen in the clinic for possible kidney disease. What major sensitive indicator of kidney disease does the nurse anticipate the patient will be tested for? Blood urea nitrogen level Creatinine clearance level Serum potassium level Uric acid level

Creatinine clearance level Creatinine is an endogenous waste product of skeletal muscle that is filtered at the glomerulus, passed through the tubules with minimal change, and excreted in the urine. Hence, creatinine clearance is a good measure of the glomerular filtration rate (GFR), the amount of plasma filtered through the glomeruli per unit of time. Creatinine clearance is the best approximation of renal function. As renal function declines, both creatinine clearance and renal clearance (the ability to excrete solutes) decrease.

The health care provider ordered four tests of renal function for a patient suspected of having renal disease. Which of the four is the most sensitive indicator? BUN to creatinine ratio Uric acid level Blood urea nitrogen (BUN) Creatinine clearance level

Creatinine clearance level The creatinine clearance measures the volume of blood cleared of endogenous creatinine in 1 minute. This serves as a measure of the glomerular filtration rate. Therefore the creatinine clearance test is a sensitive indicator of renal disease progression.

A client is having a diagnostic workup for reports of frequent diarrhea, right lower abdominal pain, and weight loss. The nurse is reviewing the results of the barium study and notes the presence of "string sign." What does the nurse understand that this is significant of? Crohn's disease Diverticulitis Ulcerative colitis Irritable bowel syndrome

Crohn's disease The most conclusive diagnostic aid for Crohn's disease has classically been a barium study of the upper GI tract that shows a "string sign" on an x-ray film of the terminal ileum, indicating the constriction of a segment of intestine.

Which ulcer is associated with extensive burn injury? Cushing ulcer Curling ulcer Peptic ulcer Duodenal ulcer

Curling ulcer Curling ulcer is frequently observed about 72 hours after extensive burns and involves the antrum of the stomach or the duodenum.

A client sustained second- and third-degree burns over 30% of the body surface area approximately 72 hours ago. What type of ulcer should the nurse be alert for while caring for this client? Esophageal ulcer Meckel's ulcer Curling's ulcer Peptic ulcer

Curling's ulcer Curling's ulcer is frequently observed about 72 hours after extensive burns and involves the antrum of the stomach or the duodenum. Peptic, esophageal, and Meckel's ulcers are not related to burn injuries.

A nurse cares for a client who has secondary obesity. Which condition is the most likely to result in secondary obesity? Cushing's disease Addison's disease Grave's disease Crohn's disease

Cushing's disease Cushing's disease, results from excess cortisol in the blood. This increases the risk of obesity. The other diseases or conditions listed most likely causes weight loss, not weight gain.

A client has undergone diagnostic testing that involved the insertion of a lighted tube with a telescopic lens. The nurse identifies this test as which of the following? Cystoscopy Intravenous pyelography Renal angiography Excretory urogram

Cystoscopy Cystoscopy is the visual examination of the inside of the bladder using an instrument called a cystoscope, a lighted tube with a telescopic lens. Renal angiography involves the passage of a catheter up the femoral artery into the aorta to the level of the renal vessels. Intravenous pyelography or excretory urography is a radiologic study that involves the use of a contrast medium to evaluate the kidneys' ability to excrete it.

Which of the following urine characteristics would the nurse anticipate when caring for a client whose lab work reveals a high urine specific gravity related to dehydration? Red urine Clear or light yellow urine Dark amber urine Turbid urine

Dark amber urine Concentrated urine (one with a high specific gravity) is a dark amber color due to the solutes in the urine. Clear or yellow urine indicates a flushing of the urinary system. Red urine indicates hematuria. A turbid urine may indicate bacteriuria.

An older adult client seeks help for chronic constipation. What factor related to aging can cause constipation in elderly clients? Decreased production of hydrochloric acid Increased intestinal motility Decreased abdominal strength Increased intestinal bacteria

Decreased abdominal strength Decreased abdominal strength, muscle tone of the intestinal wall, and motility all contribute to chronic constipation in the elderly. A decrease in hydrochloric acid causes a decrease in absorption of iron and vitamin B12, whereas an increase in intestinal bacteria actually causes diarrhea.

The nurse is completing a routine urinalysis using a dipstick. The test reveals an increased specific gravity. The nurse should suspect which condition? Increased fluid intake Diabetes insipidus Glomerulonephritis Decreased fluid intake

Decreased fluid intake When fluid intake decreases, specific gravity normally increases. With high fluid intake, specific gravity decreases. Disorders or conditions that cause decreased urine-specific gravity include diabetes insipidus, glomerulonephritis, and severe renal damage. Disorders that can cause increased specific gravity include diabetes, nephritis, and fluid deficit.

After teaching a group of students about the types of urinary incontinence and possible causes, the instructor determines that the students have understood the material when they identify which of the following as a cause of stress incontinence? Obstruction due to fecal impaction or enlarged prostate Bladder irritation related to urinary tract infections Increased urine production due to metabolic conditions Decreased pelvic muscle tone due to multiple pregnancies

Decreased pelvic muscle tone due to multiple pregnancies Stress incontinence is due to decreased pelvic muscle tone, which is associated with multiple pregnancies, obstetric injuries, obesity, menopause, or pelvic disease. Transient incontinence is due to increased urine production related to metabolic conditions. Urge incontinence is due to bladder irritation related to urinary tract infections, bladder tumors, radiation therapy, enlarged prostate, or neurologic dysfunction. Overflow incontinence is due to obstruction from fecal impaction or enlarged prostate.

After teaching a group of students about the types of urinary incontinence and possible causes, the instructor determines that the students have understood the material when they identify which of the following as a cause of stress incontinence? Obstruction due to fecal impaction or enlarged prostate Bladder irritation related to urinary tract infections Increased urine production due to metabolic conditions Decreased pelvic muscle tone due to multiple pregnancies

Deficient knowledge: management of urinary diversion Disturbed body image Risk for impaired skin integrity Deficient knowledge, disturbed body image, and risk for impaired skin integrity are expected problems for the client with a new ileal conduit. Urinary retention and chronic pain are not expected client problems.

The nurse is caring for a 77-year-old patient diagnosed with Crohn's disease. What would be especially important to monitor this patient for? Pain Dehydration Fluid overload Fatigue

Dehydration Elderly patients can become dehydrated quickly and develop low potassium levels (i.e., hypokalemia) as a result of diarrhea. The nurse observes for clinical manifestations of muscle weakness, dysrhythmias, or decreased peristaltic motility that may lead to paralytic ileus. All options would be important to monitor, but especially important is monitoring for dehydration.

The wall of the bladder is comprised of four layers. Which of the following is the layer responsible for micturition? Detrusor muscle Submucosal layer of connective tissue Inner layer of epithelium Adventitia (connective tissue)

Detrusor muscle The bladder wall contains four layers. The smooth muscle layer beneath the adventitia is known as the detrusor layer. When this muscle contracts, urine is released from the bladder. When the bladder is relaxed, the muscle fibers are closed and act as a sphincter.

A nurse who works in a clinic sees many patients with a variety of medical conditions. The nurse understands that a risk factor for UTIs is which of the following? Hyperuricemia Hyperparathyroidism Diabetes mellitus Pancreatitis

Diabetes mellitus Increased urinary glucose levels create an infection-prone environment in the urinary tract.

Which is one of the primary symptoms of irritable bowel syndrome (IBS)? Bloating Pain Diarrhea Abdominal distention

Diarrhea The primary symptoms of IBS include constipation, diarrhea, or a combination of both. Pain, bloating, and abdominal distention often accompany changes in bowel pattern.

Which of the following appears to be a significant factor in the development of gastric cancer? Diet Ethnicity Gender Age

Diet Diet seems to be a significant factor: a diet high in smoked, salted, or pickled foods and low in fruits and vegetables may increase the risk of gastric cancer. The typical patient with gastric cancer is between 50 and 70 years of age. Men have a higher incidence than women. Native Americans, Hispanic Americans, and African Americans are twice as likely as Caucasian Americans to develop gastric cancer.

A client with acute gastritis asks the nurse what might have caused the problem. What is a possible cause of acute gastritis? Excessive alcohol intake Overuse of allergy medicine Drinking fruit juices Dietary indiscretion Radiation therapy

Dietary indiscretion Excessive alcohol intake Radiation therapy Possible causes of gastritis include dietary indiscretion, overuse of aspirin and other nonsteroidal anti-inflammatory drugs, excessive alcohol intake, bile reflux, and radiation therapy. Allergy medicine and fruit juices are not causes of acute gastritis.

Which is the primary symptom of achalasia (absent or ineffective peristalsis (wavelike contraction) of the distal esophagus accompanied by failure of the esophageal sphincter to relax in response to swallowing)? Pulmonary symptoms Heartburn Difficulty swallowing Chest pain

Difficulty swallowing The primary symptom of achalasia is difficulty in swallowing both liquids and solids. The client may also report chest pain and heartburn that may or may not be associated with eating. Secondary pulmonary complications may result from aspiration of gastric contents.

A client is diagnosed with dumping syndrome after bariatric surgery. Which findings on the nursing assessment correlate with this diagnosis? Select all that apply. Hypertension Dizziness Fever Tachycardia Sweating

Dizziness Tachycardia Sweating Dumping syndrome is an unpleasant set of vasomotor and GI symptoms that is common among clients who have had bariatric surgery. Symptoms of dumping syndrome include (but are not limited to): sweating, tachycardia, nausea, vomiting, dizziness, and diarrhea. Fever and hypertension are not symptoms of dumping syndrome.

The nurse is teaching a client with an ostomy how to change the pouching system. Which information should the nurse include when teaching a client with no peristomal skin irritation? Dry skin thoroughly after washing Apply triamcinolone acetonide spray Apply barrier powder Dust with nystatin powder

Dry skin thoroughly after washing The nurse should teach the client without peristomal skin irritation to dry the skin thoroughly after washing. Barrier powder, triamcinolone acetonide spray, and nystatin powder are used when the client has peristomal skin irritation and/or fungal infection.

A client has a full bladder. Which sound would the nurse expect to hear on percussion? Resonance Flatness Dullness Tympany

Dullness Dullness on percussion indicates a full bladder; tympany indicates an empty bladder. Resonance is heard over areas that are part air and part solid, such as the lungs. Flatness is heard over very dense tissue, such as the bone or muscle.

A client who had a Roux-en-Y bypass procedure for morbid obesity ate a chocolate chip cookie after a meal. After ingestion of the cookie, the client reported cramping pains, dizziness, and palpitation. After having a bowel movement, the symptoms resolved. What should the nurse educate the client about regarding this event? Dumping syndrome Celiac disease Bile reflux Gastric outlet obstruction

Dumping syndrome Dumping syndrome is an unpleasant set of vasomotor and GI symptoms that occur in up to 76% of patients who have had bariatric surgery. Early symptoms include a sensation of fullness, weakness, faintness, dizziness, palpitations, diaphoresis, cramping pains, and diarrhea. These symptoms resolve once the intestine has been evacuated (i.e., with defecation).

A nurse is providing follow-up teaching at a clinic visit for a client recovering from gastric resection. The client reports sweating, diarrhea, nausea, palpitations, and the desire to lie down 15 to 30 minutes after meals. Based on the client's assessment, what will the nurse suspect? Peritonitis Dehiscence of the surgical wound A normal reaction to surgery Dumping syndrome

Dumping syndrome Early manifestations of dumping syndrome occur 15 to 30 minutes after eating. Signs and symptoms include vertigo, tachycardia, syncope, sweating, pallor, palpitations, diarrhea, nausea, and the desire to lie down. Dehiscence of the surgical wound is characterized by pain and a pulling or popping feeling at the surgical site. Peritonitis presents with a rigid, board-like abdomen, tenderness, and fever. The client's signs and symptoms aren't a normal reaction to surgery.

Clients with Type O blood are at higher risk for which of the following GI disorders? Duodenal ulcers Esophageal varices Diverticulitis Gastric cancer

Duodenal ulcers Familial tendency also may be a significant predisposing factor. People with blood type O are more susceptible to peptic ulcers than are those with blood type A, B, or AB. Blood type is not a predisposing factor for gastric cancer, esophageal varices, and diverticulitis.

The nurse is obtaining a history on a patient who comes to the clinic. What symptom described by the patient is one of the first symptoms associated with esophageal disease? Dysphagia Malnutrition Pain Regurgitation of food

Dysphagia Dysphagia (difficulty swallowing), the most common symptom of esophageal disease, may vary from an uncomfortable feeling that a bolus of food is caught in the upper esophagus to acute odynophagia (pain on swallowing).

A patient comes to the clinic complaining of pain in the epigastric region. The nurse suspects that the patient's pain is related to a peptic ulcer when the patient states the pain is relieved by what? Eating Having a bowel movement Suppressing emesis Drinking milk

Eating Taking antacids, eating, or vomiting often relieves the pain. Pain occurs about 2 hours after eating. Milk is contraindicated in relieving peptic ulcer pain.

An appropriate nursing intervention for the client following a nuclear scan of the kidney is to: Strain all urine for 48 hours. Monitor for hematuria. Encourage high fluid intake. Apply moist heat to the flank area.

Encourage high fluid intake. A nuclear scan of the kidney involves the IV administration of a radioisotope. Fluid intake is encouraged to flush the urinary tract to promote excretion of the isotope. Monitoring for hematuria, applying heat, and straining urine do not address the potential renal complications associated with the radioisotope.

Following a voiding cystogram, the client has a nursing diagnosis of risk for infection related to the introduction of bacterial following manipulation of the urinary tract. An appropriate nursing intervention for the client is to: Strain all urine for 48 hours. Encourage high fluid intake. Monitor for hematuria. Apply moist heat to the flank area.

Encourage high fluid intake. A voiding cystogram involves the insertion of a urinary catheter, which can result in the introduction of microorganism into the urinary tract. Fluid intake is encouraged to flush the urinary tract and promote removal of microorganisms. Monitoring for hematuria, applying heat, and straining urine do not address the nursing diagnosis of risk for infection.

A nurse is preparing a client with Crohn's disease for a barium enema. What should the nurse do the day before the test? Serve dairy products. Encourage plenty of fluids. Order a high-fiber diet. Serve the client his usual diet.

Encourage plenty of fluids. Explanation: The nurse should encourage plenty of fluids because adequate fluid intake is necessary to avoid dehydration that may be caused by the bowel preparation and to prevent fecal impaction after the procedure. The client may be placed on a low-residue diet 1 to 2 days before the procedure to reduce the contents in the GI tract. Fiber intake is limited in a low-residue diet. Because dairy products leave a residue, they aren't allowed the evening before the test. Clear liquids only are allowed the evening before the test.

A client has been recently diagnosed with an anorectal condition. The nurse is reviewing interventions that will assist the client with managing the therapeutic regimen. What would not be included? Encourage the client to follow diet and medication instructions. Encourage the client to avoid exercise. Instruct the client to cleanse perianal area with warm water. Teach the client how to do sitz baths at home using warm water three to four times each day.

Encourage the client to avoid exercise. Activity promotes healing and normal stool patterns. Proper cleansing prevents infection and irritation. Sitz baths promote healing, decrease skin irritation, and relieve rectal spasms. Encouragement promotes compliance with therapeutic regimen and prevents complications.

A health care provider suspects that a client has peptic ulcer disease. With which diagnostic procedure would the nurse most likely prepare to assist? Barium study of the upper gastrointestinal tract Gastric secretion study Stool antigen test Endoscopy

Endoscopy Barium study of the upper GI tract may show an ulcer; however, endoscopy is the preferred diagnostic procedure because it allows direct visualization of inflammatory changes, ulcers, and lesions. Through endoscopy, a biopsy of the gastric mucosa and of any suspicious lesions can be obtained. Endoscopy may reveal lesions that, because of their size or location, are not evident on x-ray studies. Less invasive diagnostic measures for detecting H. pylori include serologic testing for antibodies against the H. pylori antigen, stool antigen test, and urea breath test.

The nurse is instructing a 3-year-old's mother regarding abnormal findings within the urinary system. Which assessment finding would the nurse document as a normal finding for this age group? Hematuria Enuresis Dysuria Anuria

Enuresis The nurse would be most correct to document that enuresis, the involuntary voiding during sleep or commonly called "wetting the bed," is a normal finding in a pediatric client younger than 5 years old. Dysuria (pain on urination), hematuria (red blood cells in urine), and anuria (urine output less than 50 mL/day) are all abnormal findings needing further investigation.

Which substance stimulates the bone marrow to produce red blood cells? Prostaglandin E Erythropoietin Prostacyclin Renin

Erythropoietin Erythropoietin stimulates the bone marrow to produce red blood cells, thereby increasing the amount of hemoglobin available to carry oxygen. The kidneys produce prostaglandin E and prostacyclin, which have vasodilatory effect and are important in maintaining renal blood flow. Renin is involved in controlling arterial blood pressure.

An elderly client seeks medical attention for a vague complaint of difficulty swallowing. Which of the following assessment findings is most significant as related to this symptom? Gastroesophageal reflux disease Gastritis Hiatal hernia Esophageal tumor

Esophageal tumor Esophageal tumor is most significant and can result in advancing cancer. Esophageal cancer is a serious condition that presents with a symptom of difficulty swallowing as the tumor grows. Hiatal hernia, gastritis, and GERD can lead to serious associated complications but less likely to be as significant as esophageal tumor/cancer.

A client underwent a continent ileostomy. Within which time frame should the client expect to empty the reservoir? At least once every 2 days Every 4 to 6 hours Three or four times daily At least once a day

Every 4 to 6 hours The length of time between drainage periods is gradually increased until the reservoir needs to be drained only every 4 to 6 hours and irrigated once each day. This prevents the accumulating effluent from spilling or causing infection.

Which of the following is a cause of a calcium renal stone? Neurogenic bladder Excessive intake of vitamin D Gout Foreign bodies

Excessive intake of vitamin D Potential causes of calcium renal stones include excessive intake of vitamin D, hypercalcemia, hyperparathyroidism, excessive intake of milk and alkali, and renal tubular acidosis. Gout is associated with uric acid. Struvite stones are associated with neurogenic bladder and foreign bodies.

An elderly client states, "I don't understand why I have so many caries in my teeth." What assessment made by the nurse places the client at risk for dental caries? Eating fruits and cheese in diet Drinking fluoridated water Using a soft-bristled toothbrush Exhibiting hemoglobin A1C 8.2

Exhibiting hemoglobin A1C 8.2 Measures used to prevent and control dental caries include controlling diabetes. A hemoglobin A1C of 8.2 is not controlled. It is recommended for hemoglobin A1C to be less than 7 for people with diabetes. Other measures to prevent and control dental caries include drinking fluoridated water; eating foods that are less cariogenic, which include fruits, vegetables, nuts, cheese, or plain yogurt; and brushing teeth evenly with a soft-bristled toothbrush.

The nurse is performing a community screening for colorectal cancer. Which characteristic should the nurse include in the screening? Familial polyposis Low-fat, low-protein, high-fiber diet Age younger than 40 years History of skin cancer

Familial polyposis Family history of colon cancer or familial polyposis is a risk factor for colorectal cancer. Age older than 40 years and a high-fat, high-protein, low-fiber diet are risk factors for colorectal cancer. A history of skin cancer is not a recognized risk factor for colorectal cancer.

Which medication is classified as a histamine-2 receptor antagonist? Esomeprazole Famotidine Metronidazole Lansoprazole

Famotidine Famotidine is a histamine-2 receptor antagonist. Lansoprazole and esomeprazole are proton pump inhibitors (PPIs). Metronidazole is an antibiotic.

A client has been taking famotidine at home. What teaching should the nurse include with the client? Famotidine will inhibit gastric acid secretions. Famotidine will neutralize acid in the stomach. Famotidine will shorten the time required for digestion in the stomach. Famotidine will improve the mixing of foods and gastric secretions.

Famotidine will inhibit gastric acid secretions. Famotidine is useful for treating and preventing ulcers and managing gastroesophageal reflux disease. It functions by inhibiting the action of histamine at the H-2 receptor site located in the gastric parietal cells, thus inhibiting gastric acid secretion. Famotidine will not neutralize acid in the stomach, but inhibits acid secretion. Famotidine will not shorten digestion time and will not improve food mixing with gastric secretions.

A nurse cares for a client who is postoperative bariatric surgery and has experienced frequent episodes of dumping syndrome. The client now reports anorexia. What is the primary reason for the client's report of anorexia? Absorption of food Fear of eating Size of the stomach Taste of food

Fear of eating Dumping syndrome is an unpleasant set of GI and vasomotor symptoms that commonly occur in clients who have had bariatric surgery. The symptoms are so unpleasant that the client may develop a fear of eating, leading to anorexia.

A 60-year-old woman has begun a course of oral antibiotics for the treatment of a urinary tract infection (UTI). The patient's nurse should recognize that the causative microorganisms most likely originated from: Proliferation of normal microbiotic flora Colonization of the patient's urethra from bloodborne pathogens Fecal contamination from the patient's perineum Ingested microorganisms

Fecal contamination from the patient's perineum Most of the microorganisms the result in UTIs are a result of fecal contamination. UTIs do not usually result from proliferation of normal microbiota or ingested microorganisms.

The nurse teaches a client scheduled for a colonoscopy. Which instruction should be included as part of the preparation for the procedure? Spray or gargle with a local anesthetic. Do not void for at least 30 minutes before the test. Consume at least 3 quarts of water 30 minutes before the test. Follow the dietary and fluid restrictions and bowel preparation procedures.

Follow the dietary and fluid restrictions and bowel preparation procedures. Explanation: For a client due to have a colonoscopy, it is essential that the client follow the dietary and fluid restrictions and bowel preparation procedures. For the client having an esophagogastroduodenoscopy (EGD), it is necessary for the client to spray or gargle with a local anesthetic. The client is not advised to consume 3 quarts of water nor to void before the test. These interventions may be essential for tests that involve ultrasonographic procedures.

Postoperatively, a client with a radical neck dissection should be placed in which position? Prone Fowler Side-lying Supine

Fowler The client should be placed in the Fowler position to facilitate breathing and promote comfort. This position expands the lungs because the diaphragm is pulled downward and the abdominal viscera are pulled away from the lungs. The other positions are not the position of choice postoperatively.

A few hours after eating hot and spicy chicken wings, a client presents with lower chest pain. He wonders if he is having a heart attack. How should the nurse proceed first? Further investigate the initial complaint. Explain that fatty foods can mimic chest pain. Administer an over-the-counter antacid tablet. Call for an immediate electrocardiogram.

Further investigate the initial complaint. Explanation: While fatty foods can cause discomfort similar to chest pain, the nurse must fully assess all the client's symptoms. Investigation of chief complaint begins with a complete history. The underlying cause of pain influences the characteristics, duration, pattern, location, and distribution of pain.

A nurse cares for a client with obesity. Which medication that the client takes may be contributing to the client's obesity? Metformin Topiramate Gabapentin Bupropion

Gabapentin Gabapentin (Neurontin) is an anticonvulsant medication which promotes weight gain. The other answer choices are medications which promote weight loss, not gain.

A patient describes a burning sensation in the esophagus, pain when swallowing, and frequent indigestion. What does the nurse suspect that these clinical manifestations indicate? Peptic ulcer disease Esophageal cancer Gastroesophageal reflux disease Diverticulitis

Gastroesophageal reflux disease Symptoms may include pyrosis (burning sensation in the esophagus), dyspepsia (indigestion), regurgitation, dysphagia or odynophagia (pain on swallowing), hypersalivation, and esophagitis.

The nurse analyzes a urinalysis report. He is aware that the presence of this substance in the urine indicates a blood level that exceeds the kidney's reabsorption capacity. Select the substance. Creatinine Glucose Bicarbonate Sodium

Glucose Glucose is usually filtered at the level of the glomerulus. It does not normally appear in the urine. Renal glycosuria occurs if the glucose in the blood exceeds the amount that is able to be reabsorbed. The other substances are normally excreted in the urine.

The nurse is caring for a patient suspected of having renal dysfunction. When reviewing laboratory results for this patient, the nurse recalls that several substances are filtered from the blood by the glomerulus and these substances are then excreted in the urine. The nurse identifies the presence of which substances in the urine as abnormal findings? Bicarbonate and urea Glucose and protein Creatinine and chloride Potassium and sodium

Glucose and protein The various substances normally filtered by the glomerulus, reabsorbed by the tubules, and excreted in the urine include sodium, chloride, bicarbonate, potassium, glucose, urea, creatinine, and uric acid. Within the tubule, some of these substances are selectively reabsorbed into the blood. Glucose is completely reabsorbed in the tubule and normally does not appear in the urine. However, glucose is found in the urine if the amount of glucose in the blood and glomerular filtrate exceeds the amount that the tubules are able to reabsorb. Protein molecules are also generally not found in the urine because amino acids are also filtered at the level of the glomerulus and reabsorbed so that it is not excreted in the urine.

A client with human immunodeficiency virus (HIV) comes to the clinic and is experiencing white patches on the lateral border of the tongue. What type of lesions does the nurse document? Aphthous stomatitis Nicotine stomatitis Erythroplakia Hairy leukoplakia

Hairy leukoplakia Hairy leukoplakia is a condition often seen in people who are HIV positive in which white patches with rough, hairlike projections form, typically on lateral border of the tongue. Aphthous stomatitis is typically a recurrent round or oval sore or ulcer on the inside of the lips and cheeks or underneath the tongue and is not associated with HIV. Erythroplakia describes a red area or red spots on the lining of the mouth and is not associated with HIV. Nicotine stomatitis is a white patch in the mouth caused by extreme heat from smoking.

The nurse is assessing a client with an ulcer for signs and symptoms of hemorrhage. The nurse interprets which condition as a sign/symptom of possible hemorrhage? Hematemesis Hypertension Polyuria Bradycardia

Hematemesis The nurse interprets hematemesis as a sign/symptom of possible hemorrhage from the ulcer. Other signs that can indicate hemorrhage include tachycardia, hypotension, and oliguria/anuria.

The nurse is assessing a client with an ulcer for signs and symptoms of hemorrhage. The nurse interprets which condition as a sign/symptom of possible hemorrhage? Polyuria Hypertension Bradycardia Hematemesis

Hematemesis The nurse interprets hematemesis as a sign/symptom of possible hemorrhage from the ulcer. Other signs that can indicate hemorrhage include tachycardia, hypotension, and oliguria/anuria.

The nurse is obtaining a health history from a client describing urinary complications. Which assessment finding is most suggestive of a malignant tumor of the bladder? Incontinence Frequency Hematuria Dysuria

Hematuria The most common first symptom of a malignant tumor is hematuria. Most malignant tumors are vascular; thus, abnormal bleeding can be a first sign of abnormality. The client then has symptoms of incontinence (a later sign), dysuria and frequency.

To assess circulating oxygen concentration, the 2001 Kidney Disease Outcomes Quality Initiative: Management of Anemia Guidelines recommends the use of which diagnostic test? Arterial blood gases Hematocrit Hemoglobin Serum iron concentration

Hemoglobin Although hematocrit has always been the blood test of choice to assess for anemia, the 2001 Kidney Disease Outcomes Quality Initiative: Management of Anemia Guidelines recommend that anemia be quantified using hemoglobin rather than hematocrit measurements. Hemoglobin is recommended because it more accurately assesses circulating oxygen than does hematocrit. Serum iron concentration measures iron storage in the body. Arterial blood gases assess the adequacy of oxygenation, ventilation, and acid-base status.

When examining the abdomen of a client with reports of nausea and vomiting, what would the nurse do first? Palpation Percussion Auscultation Inspection

Inspection Explanation: When assessing the abdomen, the nurse would first inspect or observe the abdomen. This would be followed by auscultation, percussion, and lastly, palpation.

A client with severe peptic ulcer disease has undergone surgery and is several hours postoperative. During assessment, the nurse notes that the client has developed cool skin, tachycardia, labored breathing, and appears to be confused. Which complication has the client most likely developed? Penetration Perforation Hemorrhage Pyloric obstruction

Hemorrhage Signs of hemorrhage following surgery include cool skin, confusion, increased heart rate, labored breathing, and blood in the stool. Signs of penetration and perforation are severe abdominal pain, rigid and tender abdomen, vomiting, elevated temperature, and increased heart rate. Indicators of pyloric obstruction are nausea, vomiting, distended abdomen, and abdominal pain.

The nurse is performing a renal assessment on a client with prostate cancer. Which clinical manifestation suggests prostate cancer? Select all that apply. Palpitations Dyspnea Hesitancy Chills Nocturia

Hesitancy Nocturia Clinical manifestations of prostate cancer include urinary hesitancy and nocturia. Palpitations, chills, and dyspnea are not suggestive of prostate cancer.

A group of students is reviewing information about oral cancers in preparation for an examination. The students demonstrate a need for additional review when they identify which of the following as a risk factor? History of GERD Pipe smoking Excess alcohol use Prolonged exposure to sun

History of GERD Development of oral cancers is linked with smoking, chewing tobacco, and drinking alcohol in excess. Lip cancer is associated with pipe smoking and prolonged exposure to wind and sun. A history of GERD is associated with the development of esophageal cancer.

A client who is postoperative open RYGB bariatric surgery is scheduled for discharge and will have a Jackson-Pratt drain to care for while at home. Which teaching will the nurse include specific to this? Select all that apply. Recording drainage amount How to change the drain When to contact the health care provider How to empty the drain How to measure the drainage amount

How to empty the drain Recording drainage amount When to contact the health care provider How to measure the drainage amount A client who is discharged with a Jackson-Pratt drain must be taught on methods to measure, record, and empty the drain. Additionally, the nurse should instruct the client on when to contact the health care provider. The client will not change the drain, this is reserved for the health care provider only.

The nurse auscultates the abdomen to assess bowel sounds. She documents five to six sounds heard in less than 30 seconds. How does the nurse document the bowel sounds? Hypoactive Borborygmi Hyperactive Normal

Hyperactive Explanation: Bowel sounds are assessed using the diaphragm of the stethoscope for high-pitched and gurgling sounds (Gu, Lim, & Moser, 2010). The frequency and character of the sounds are usually heard as clicks and gurgles that occur irregularly and range from 5 to 35 per minute. The terms normal (sounds heard about every 5 to 20 seconds), hypoactive (one or two sounds in 2 minutes), hyperactive (5 to 6 sounds heard in less than 30 seconds), or absent (no sounds in 3 to 5 minutes) are frequently used in documentation, but these assessments are highly subjective (Li, Wang, & Ma, 2012).

A nurse working in a cardiac health care office notes increased risk of certain cardiac conditions as a result of obesity. Which conditions can be associated with obesity? Select all that apply. Heart failure Hypertension Heart murmur Coronary artery disease Myocardial infarction

Hypertension Coronary artery disease Heart failure Myocardial infarction Various cardiac diseases and conditions may be associated with obesity. These include: hypertension, heart failure, myocardial infarction, and coronary artery disease. Heart murmur is not directly associated with obesity.

An elderly client diagnosed with diarrhea is taking digoxin. Which electrolyte imbalance should the nurse be alert to? Hypernatremia Hyponatremia Hypokalemia Hyperkalemia

Hypokalemia The older client taking digitalis must be aware of how quickly dehydration and hypokalemia can occur with diarrhea. The nurse teaches the client to recognize the symptoms of hypokalemia because low levels of potassium intensify the action of digitalis, leading to digitalis toxicity.

A critical care nurse is closely monitoring a patient who has recently undergone surgical repair of a bleeding peptic ulcer. The nurse should prioritize assessments of which of the following signs and symptoms of a recurrence of hemorrhage? Bradypnea and pursed lip breathing Restlessness and cyanosis Peripheral and pulmonary edema Hypotension and tachycardia

Hypotension and tachycardia Rebleeding has multiple manifestations. However, an increase in heart rate and decrease in blood pressure are key signs of a hemorrhage that are present in nearly all patients who are experiencing rebleeding.

A critical care nurse is closely monitoring a patient who has recently undergone surgical repair of a bleeding peptic ulcer. The nurse should prioritize assessments of which of the following signs and symptoms of a recurrence of hemorrhage? Restlessness and cyanosis Peripheral and pulmonary edema Bradypnea and pursed lip breathing Hypotension and tachycardia

Hypotension and tachycardia Rebleeding has multiple manifestations. However, an increase in heart rate and decrease in blood pressure are key signs of a hemorrhage that are present in nearly all patients who are experiencing rebleeding.

Review the following four examples of ideal body weight (IBW), actual weight, and body mass index (BMI). Using three criteria for each example, select the body weight that indicates morbid obesity. IBW = 175 lbs; weight = 265 lbs; BMI = 29 kg/m2 IBW = 145 lbs; weight = 290 lbs; BMI = 31 kg/m2 IBW = 132 lbs; weight = 184 lbs; BMI = 28 kg/m2 IBW = 150 lbs; weight = 190 lbs; BMI = 26 kg/m2

IBW = 145 lbs; weight = 290 lbs; BMI = 31 kg/m2 The criteria for morbid obesity are a body weight that is twice IBW and a BMI that exceeds 30 kg/m2.

Which factor contributes to UTI in older adults? Low incidence of chronic illness Active lifestyle Sporadic use of antimicrobial agents Immunocompromise

Immunocompromise Factors that contribute to urinary tract infection in older adults include immunocompromise, high incidence of chronic illness, immobility, frequent use of antimicrobial agents, incomplete emptying of the bladder, and obstructed urine flow.

The nurse is conducting a community education program on UTIs. The nurse determines that the participants understand the teaching when they identify which factor as contributing to UTIs in older adults? Low incidence of chronic illness Sporadic use of antimicrobial agents Active lifestyle Immunocompromise

Immunocompromise Factors that contribute to UTIs in older adults include immunocompromise, cognitive impairment, high incidence of chronic illness, immobility, incomplete emptying of the bladder, obstructed flow of urine, and frequent use of antimicrobial agents.

The nurse is admitting a client with traumatic injuries who also has class III obesity. When planning this client's care, the nurse should address the client's heightened risk of what nursing diagnoses related to obesity? Select all that apply. Unilateral neglect Impaired skin integrity Deficient fluid volume Impaired gas exchange Bowel incontinence

Impaired skin integrity Impaired gas exchange Obesity creates risks for ineffective respiration and consequent impaired gas exchange due to changes in the structure and function of the respiratory system. As well, obesity is associated with risks to skin integrity due to the possibility of pressure injuries. There is no accompanying risk of bowel incontinence or fluid volume deficit that is accounted for by obesity. If neglect exists, it is likely to be bilateral, not unilateral.

The nurse is educating a patient with urolithiasis about preventive measures to avoid another occurrence. What should the patient be encouraged to do? Increase fluid intake so that the patient can excrete 2,500 to 4,000 mL every day, which will help prevent additional stone formation. Limit voiding to every 6 to 8 hours so that increased volume can increase hydrostatic pressure, which will help push stones along the urinary system. Add calcium supplements to the diet to replace losses to renal

Increase fluid intake so that the patient can excrete 2,500 to 4,000 mL every day, which will help prevent additional stone formation. A patient who has shown a tendency to form stones should drink enough fluid to excrete greater than 2,000 mL (preferably 3,000 to 4,000 mL) of urine every 24 hours (Meschi et al., 2011).

The nurse is assessing the abdomen of the client with an undiagnosed disorder. In which sequence would the nurse conduct the abdominal assessment? Use all options. Percussion Auscultation Inspection Palpation

Inspection Auscultation Percussion Palpation Explanation: The nurse is correct to assess the abdomen in a specific order to be able to judge the undisturbed status of the abdominal region. Begin with inspection of the abdomen using the nurse's assessment skills. Next, auscultate the abdomen before percussing and finally palpating.

An older client is experiencing an increasingly troublesome need to urinate several times through the night. The client's prostate is within normal limits, and the physician prescribes limiting fluid intake after the evening meal. What is another important intervention to keep the client safe? Increase fluid intake throughout the day. Decrease salt intake. Decrease overall fluid intake. Increase protein intake.

Increase fluid intake throughout the day. Older persons may need to drink more fluids throughout the day to allow for limiting their intake after the evening meal. Urine formation increases during the night, when leg elevation promotes blood return to the heart and kidneys, and may interrupt sleep patterns. Salt is secreted. Filtrate that is secreted as urine usually contains sodium and chloride. Protein molecules, except for periodic small amounts of globulins and albumin, also are reabsorbed.

The nurse is working on a general medical unit. A client is scheduled for an upper gastrointestinal series. Upon returning to the nursing unit, what does the nurse identify as the client goal? Decrease in nausea and vomiting Increase in the amount of fluids Recovery from the general anesthesia Ambulates independently

Increase in the amount of fluids Explanation: The client, returning from an upper gastrointestinal series, needs to increase fluids in an effort to eliminate the barium from the body through a bowel movement. General anesthesia is not used. The client typically does not have nausea and vomiting following the procedure. If the client is able to ambulate independently prior to the procedure, the client will be able to ambulate independently following.

Patients with urolithiasis need to be encouraged to: Limit their voiding to every 6 to 8 hours so that increased volume can increase hydrostatic pressure, which will help push stones along the urinary system. Supplement their diet with calcium needed to replace losses to renal calculi. Increase their fluid intake so that they can excrete 2.5 to 4 liters every day. Participate in strenuous exercises so that the tone of smooth muscle in the urinary tract can be strengthened to help propel calculi

Increase their fluid intake so that they can excrete 2.5 to 4 liters every day. Fluids need to be increased up to 4 L/day to help prevent additional stone formation.

Place the pathophysiological steps in order of how a client with obesity is at greater risk for venous thromboembolism in comparison to the general population. Impairment of peripheral blood flow Blood stasis Formation of a thrombus Increased adipose tissue

Increased adipose tissue Impairment of peripheral blood flow Blood stasis Formation of a thrombus In obesity, an increase in adipose tissue impairs the peripheral blood flow, leading to blood stasis and the formation of a thrombus.

Place the pathophysiological steps in order for the normal role of leptin as it relates to hunger and satiety. Increased leptin Inhibition of food intake Increased fat stores Increased satiety

Increased fat stores Increased leptin Increased satiety Inhibition of food intake Under normal circumstances, increased fat stores, increases leptin, which increases satiety and feeling full. This then inhibits food intake. In obesity, alterations in leptin are thought to play a role in the development of the disease.

Gastrin has which of the following effects on gastrointestinal (GI) motility? Increased motility of the stomach Relaxation of gastroesophageal sphincter Relaxation of the colon Contraction of the ileocecal sphincter

Increased motility of the stomach Explanation: Gastrin has the following effects on GI motility: increased motility of the stomach, excitation of the colon, relaxation of ileocecal sphincter, and contraction of the gastroesophageal sphincter.

A nurse is reviewing the laboratory test results of a client with renal disease. Which of the following would the nurse expect to find? Increased serum albumin Increased serum creatinine Decreased potassium Decreased blood urea nitrogen (BUN)

Increased serum creatinine In clients with renal disease, the serum creatinine level would be increased. The BUN also would be increased, serum albumin would be decreased, and potassium would likely be increased.

A client is being evaluated for esophageal cancer. What initial manifestation of esophageal cancer should the nurse assess? Sensation of a mass in throat Foul breath Hiccups Increasing difficulty in swallowing

Increasing difficulty in swallowing The client first becomes aware of intermittent and increasing difficulty in swallowing with esophageal cancer. As the tumor grows and the obstruction becomes nearly complete, even liquids cannot pass into the stomach. Other clinical manifestations may include the sensation of a mass in the throat, foul breath, and hiccups, but these are not the most common initial clinical manifestation with clients with esophageal cancer.

A client is recovering from an ileostomy that was performed to treat inflammatory bowel disease. During discharge teaching, what would the nurse stress the importance of? Wearing an appliance pouch only at bedtime Increasing fluid intake to prevent dehydration Taking only enteric-coated medications Consuming a low-protein, high-fiber diet

Increasing fluid intake to prevent dehydration Because stool forms in the large intestine, an ileostomy typically drains liquid waste. To avoid fluid loss through ileostomy drainage, the nurse should instruct the client to increase fluid intake. The nurse should teach the client to wear a collection appliance at all times because ileostomy drainage is incontinent, to avoid high-fiber foods because they may irritate the intestines, and to avoid enteric-coated medications because the body can't absorb them after an ileostomy.

A client has a radical neck dissection to treat cancer of the neck. The nurse develops the care plan and includes all the following diagnoses. The nurse identifies the highest priority diagnosis as Impaired tissue integrity related to surgical intervention Risk for infection related to surgical intervention Imbalanced nutrition: less than body requirements, related to treatment Ineffective airway clearance related to obstruction by mucus

Ineffective airway clearance related to obstruction by mucus All the nursing diagnoses are appropriate for a client who has a radical neck dissection. According to Maslow's hierarchy of needs, physiological needs take priority. Under physiological needs, airway, breathing, circulation (ABCs) take highest priority. Thus, ineffective airway clearance is the highest priority nursing diagnosis.

A client reports urinary frequency, urgency, and dysuria (painful urination). Which of the following would the nurse most likely suspect? Infection Obstruction of the lower urinary tract Acute renal failure Nephrotic syndrome

Infection Frequency, urgency, and dysuria are commonly associated with urinary tract infection. Hesitancy and enuresis may indicate an obstruction. Oliguria or anuria and proteinuria might suggest acute renal failure. Oliguria is defined as a urine output that is less than 400 mL/24 h or less than 17 mL/h in adults. Anuria is defined as urine output that is less than 100 mL/24 h or 0 mL/12 h. Nocturia (is a condition in which you wake up during the night because you have to urinate) is associated with nephrotic syndrome.

A nurse caring for a patient with a neurogenic bladder knows to assess for the major complication of: Infection Permanent distention Consistent pain Daily and painful spasms

Infection Infection is caused by an increased urinary bacterial count that results from incomplete and delayed emptying of the bladder.

Crohn's disease is a condition of malabsorption caused by which pathophysiological process? Gastric resection Inflammation of all layers of intestinal mucosa Infectious disease Disaccharidase deficiency

Inflammation of all layers of intestinal mucosa Crohn's disease, also known as regional enteritis, can occur anywhere along the gastrointestinal tract but most commonly at the distal ileum and in the colon. Infectious disease causes problems such as small-bowel bacterial overgrowth, leading to malabsorption. Disaccharidase deficiency leads to lactose intolerance. Postoperative malabsorption occurs after gastric or intestinal resection.

A nursing student is caring for a client with gastritis. Which of the following would the student recognize as a common cause of gastritis? Choose all that apply. Irritating foods Overuse of aspirin Participation in highly competitive sports DASH diet Ingestion of strong acids

Ingestion of strong acids Irritating foods Overuse of aspirin Acute gastritis is often caused by dietary indiscretion-a person eats food that is irritating, too highly seasoned, or contaminated with disease-causing microorganisms. Other causes of acute gastritis include overuse of aspirin and other nonsteroidal anti-inflammatory drugs (NSAIDs), excessive alcohol intake, bile reflux, and radiation therapy. A more severe form of acute gastritis is caused by the ingestion of strong acid or alkali, which may cause the mucosa to become gangrenous or to perforate. A DASH diet is an acronym for Dietary Approaches to Stop Hypertension, which would not cause gastritis. Participation in competitive sports also would not cause gastritis.

A client has a cheesy white plaque in the mouth. The plaque looks like milk curds and can be rubbed off. What is the nurse's best intervention? Provide saline rinses prior to meals. Instruct the client to swish prescribed nystatin solution for 1 minute. Remove the plaque from the mouth by rubbing with gauze. Encourage the client to ingest a soft or bland diet.

Instruct the client to swish prescribed nystatin solution for 1 minute. A cheesy white plaque in the mouth that looks like milk curds and can be rubbed off is candidiasis. The most effective treatment is antifungal medication such as nystatin (Mycostatin). When used as a suspension, the client is to swish vigorously for at least 1 minute and then swallow. Other measures such as providing saline rinses or ingesting a soft or bland diet are comfort measures. The nurse does not remove the plaques; doing so will cause erythema and potential bleeding.

The nurse observes that the client's urine is orange. Which additional assessment would be important for this client? Bleeding Intake of multiple vitamin preparations Intake of medication such as phenazopyridine hydrochloride Infection

Intake of medication such as phenazopyridine hydrochloride Urine that is orange may be caused by intake of phenazopyridine hydrochloride or other medications. Orange to amber-colored urine may also indicate concentrated urine due to dehydration or fever. Urine that is pink to red may indicate lower urinary tract bleeding. Urine that is bright yellow is an anticipated abnormal finding in the client taking a multiple vitamin preparation. Yellow to milky white urine may indicate infection, pyuria, or, in the female client, the use of vaginal creams.

A patient visited a nurse practitioner because he had diarrhea for 2 weeks. He described his stool as large and greasy. The nurse knows that this description is consistent with a diagnosis of: Inflammatory colitis. A small bowel disorder. Intestinal malabsorption. A disorder of the large bowel.

Intestinal malabsorption. Watery stools are characteristic of disorders of the small bowel, whereas loose, semisolid stools are associated more often with disorders of the large bowel. Large, greasy stools suggest intestinal malabsorption, and the presence of mucus and pus in the stools suggests inflammatory enteritis or colitis.

A client is diagnosed with megaloblastic anemia caused by vitamin B12 deficiency. The health care provider begins the client on cyanocobalamin (Betalin-12), 100 mcg I.M. daily. Which substance influences vitamin B12 absorption? Intrinsic factor Hydrochloric acid Liver enzyme Histamine

Intrinsic factor Explanation: Vitamin B12 absorption depends on intrinsic factor, which is secreted by parietal cells in the stomach. The vitamin binds with intrinsic factor and is absorbed in the ileum. Hydrochloric acid, histamine, and liver enzymes don't influence vitamin B12 absorption.

A nurse geneticist is researching the gut microbiome and its relationship to disease. What is true regarding the microbiome? It is less diverse than human genome. It has over 100 times more genes than the human genome. It has over 10 times more genes than the human genome. Its function has yet to be discovered.

It has over 100 times more genes than the human genome. The collective genome of the microbiota, or the gut microbiome, has more than 100 times more genes than in the human genome. Its function and relationship to disease has long been studied.

To obtain information about the chief report and medical history of an older client, the nurse asks the client about any medication history. Why is obtaining a medication history important? It may reflect the client's childhood and family illnesses. It may indicate multiple medications taken by the client. It may indicate drugs that should not be prescribed to the client. It may indicate the client's general health.

It may indicate multiple medications taken by the client. The nurse should obtain information about a client's medication history because older clients, in particular, may be taking multiple medications that may affect their renal function. The medication history in general indicates the probable risk factors of renal or urologic disorders. The medication history of an older client is not used to obtain information about the client's general health, childhood and family illnesses, or drugs that are contraindicated for use by the client.

A nurse working with clients with obesity understands that the hypothalamus plays an important role in hunger and satiety. Which statement best describes the role of the hypothalamus in hunger and satiety? It signals the release of neuropeptide y, which leads to feelings of satiety. It signals the release of ghrelin, which increases feelings of hunger. It signals the GI system to slow gastric motility. It signals higher neural pathways that lead to eating behaviors.

It signals higher neural pathways that lead to eating behaviors. The hypothalamus signals higher neural pathways that lead to eating behaviors. The hypothalamus does not signal the release of ghrelin or neuropeptide y, nor does it signal the GI system to slow gastric motility.

A client comes to the emergency department complaining of sudden onset of sharp, severe pain in the lumbar region that radiates around the side and toward the bladder. The client also reports nausea and vomiting and appears pale, diaphoretic, and anxious. The physician tentatively diagnoses renal calculi and orders flat-plate abdominal X-rays. Renal calculi can form anywhere in the urinary tract. What is their most common formation site? Urethra Ureter Kidney Bladder

Kidney The most common site of renal calculi formation is the kidney. Calculi may travel down the urinary tract with or without causing damage and lodge anywhere along the tract or may stay within the kidney. The ureter, bladder, and urethra are less common sites of renal calculi formation.

The nurse reviews a client's history and notes that the client has a history of hyperparathyroidism. The nurse would identify that this client most likely would be at risk for which of the following? Neurogenic bladder Chronic renal failure Kidney stones Fistula

Kidney stones A client with hyperparathyroidism is at risk for kidney stones. The client with diabetes mellitus is a risk factor for developing chronic renal failure and neurogenic bladder. A client with radiation to the pelvis is at risk for urinary tract fistula.

A patient is having an MAG3 renogram and is informed that radioactive material will be injected to determine kidney function. What should the nurse instruct the patient to do during the procedure? Drink contrast material at various intervals during the procedure. Take deep breaths and hold them at various times throughout the procedure. Turn from side to side to get a variety of views during the procedure. Lie still on the table for approximately 35 minutes.

Lie still on the table for approximately 35 minutes. This relatively new scan is used to further evaluate kidney function in some centers. The patient is given an injection containing a small amount of radioactive material, which will show how the kidneys are functioning. The patient needs to lie still for about 35 minutes while special cameras take images (Albala, Gomelia, Morey, et al., 2010).

A nurse is caring for a client with a BMI of 35 kg/m2 who is wanting to lose weight. What is the initial recommendation the nurse will expect from the client's health care provider? Lifestyle modification Pharmacological management Nonsurgical interventions Surgical interventions

Lifestyle modification All answer choices represents the various treatment for obesity; however, lifestyle modification is the initial recommendation for weight loss.

A nurse cares for a client with obesity who has type 2 diabetes. Which medication does the nurse recognize may assist in weight loss and is also approved to treat type 2 diabetes? Lorcaserin Orlistat Benzphetamine Liraglutide

Liraglutide Liraglutide (Saxenda), a GLP-1 receptor agonist, is used for both the treatment of obesity and type 2 diabetes. The other medications are used for the treatment of obesity only.

After 20 seconds of auscultating for bowel sounds on a client recovering from abdominal surgery, the nurse hears nothing. What should the nurse do based on the assessment findings? Listen longer for the sounds. Return in 1 hour and listen again to confirm findings. Call the health care provider to report absent bowel sounds. Document that the client is constipated.

Listen longer for the sounds. Explanation: Auscultation is used to determine the character, location, and frequency of bowel sounds. The frequency and character of sounds are usually heard as clicks and gurgles that occur irregularly and range from 5 to 35 per minutes. Normal sounds are heard about every 5 to 20 seconds, whereas hypoactive sounds can be one or two sounds in 2 minutes. Postoperatively, it is common for sounds to be reduced; therefore, the nurse needs to listen at least 3 to 5 minutes to verify absent or no bowel sounds.

For a client with salivary calculi, which procedure uses shock waves to disintegrate the stone? Biopsy Radiation Chemotherapy Lithotripsy

Lithotripsy Lithotripsy uses shock waves to disintegrate stones. It may be used instead of surgical extraction for parotid stones and smaller submandibular stones. Radiation, chemotherapy, and biopsy do not use shock waves to disintegrate a stone.

The nurse is caring for a geriatric client and notices polypharmacy. Which diagnostic studies are anticipated? Urinalysis Blood chemistry Complete blood count Liver function studies

Liver function studies Explanation: The liver metabolizes and biotransforms the medications ingested. Geriatric clients who experience polypharmacy or multiple medications have an elevated risk of liver impairment. Routine liver function studies monitor the status of the liver and its ability to metabolize.

The nurse performs a physical examination on a client diagnosed with acute pyelonephritis to assist in determining which of the following? Structural defects in the kidneys Location of discomfort Abnormalities in urine Elevated calcium levels

Location of discomfort The physical examination of a client with pyelonephritis helps the nurse determine the location of discomfort and signs of fluid retention, such as peripheral edema or shortness of breath. Observing and documenting the characteristics of the client's urine helps the nurse detect abnormalities in the urine. Laboratory blood tests reveal elevated calcium levels, whereas radiography and ultrasonography depict structural defects in the kidneys.

A nurse cares for a client who is post op bariatric surgery. Which position will the nurse place the client in order to best promote comfort? High Fowler's Low Fowler's Lateral Upright

Low Fowler's Positioning the client in low Fowler's position best promotes comfort in the client who is post op bariatric surgery. In addition to decreasing incisional pain, this position also promotes gastric emptying.

Diet modifications for patient diagnosed with chronic inflammatory bowel disease include which of the following? Iron restriction Low protein Low residue Calorie restriction

Low residue Oral fluids and a low-residue, high-protein, high-calorie diet with supplemental vitamin therapy and iron replacement are prescribed to meet the nutritional needs, reduce inflammation, and control pain and diarrhea.

A client undergoes surgery to remove a malignant tumor, followed by a urinary diversion procedure. Which postoperative procedure should the nurse perform? Suggest a visit to a local ostomy group Determine the client's ability to manage stoma care Maintain skin and stoma integrity Show photographs and drawings of the placement of the stoma

Maintain skin and stoma integrity The most important postoperative nursing management is to maintain skin and stoma integrity to avoid further complications, such as skin infections and urinary odor. Determining the client's ability to manage stoma care, showing photographs, and suggesting a visit to a local ostomy group would be a part of the preoperative procedure.

A client undergoes surgery to remove a malignant tumor, followed by a urinary diversion procedure. The nurse's postoperative plan of care should include which action? Suggest a visit to a local ostomy group. Determine the client's ability to manage stoma care. Show pictures and drawings of placement of the stoma. Maintain skin and stomal integrity.

Maintain skin and stomal integrity. The most important postoperative nursing management is to maintain skin and stomal integrity to avoid further complications, such as skin infections and urinary odor. Determining the client's ability to manage stoma care, showing photographs, and suggesting a visit to a local ostomy group would be a part of the preoperative procedure.

A 75-year-old male patient presents at the emergency department with symptoms of a small bowel obstruction. An emergency room nurse is obtaining assessment data from this patient. What assessment finding is characteristic of a small bowel obstruction? Nausea and vomiting Decrease in urine production Mucosal edema Mucus in the stool

Nausea and vomiting Nausea and vomiting are symptoms of a small bowel obstruction. Decrease in urine production and mucosal edema are not symptoms of a bowel obstruction. The patient may defecate mucus, but this is not accompanied by stool.

A client who reports increasing difficulty swallowing, weight loss, and fatigue is diagnosed with esophageal cancer. Because this client has difficulty swallowing, what should the nurse assign highest priority to? Helping the client cope with body image changes Preventing injury Maintaining a patent airway Ensuring adequate nutrition

Maintaining a patent airway Rapid growth of cancer cells in the esophagus may put pressure on the adjacent trachea, jeopardizing the airway. Therefore, maintaining a patent airway is the highest care priority for a client with esophageal cancer. Helping the client cope with body image changes, ensuring adequate nutrition, and preventing injury are appropriate for a client with this disease, but are less crucial than maintaining airway patency.

A nurse is planning care for a client who will be arriving to the unit postoperatively from bariatric surgery. In an effort to decrease the risk of venous thromboembolism (VTE), which health care provider orders does the nurse anticipate? Mechanical compression and prophylactic anticoagulation Prophylactic anticoagulation only Mechanical compression only Early ambulation only

Mechanical compression and prophylactic anticoagulation Both mechanical compression (intermittent pneumatic compression devices) and prophylactic anticoagulation with low molecular weight heparin agents are prescribed in the client who is postoperative bariatric surgery. Early ambulation is encouraged; however, it is not the only intervention.

The nurse is conducting a history and assessment related to a client's incontinence. Which element should the nurse include in the assessment before beginning a bladder training program? Occupational history History of allergies Smoking habits Medication usage

Medication usage It is essential to assess the client's physical and environmental conditions before beginning a bladder training program, because the patient may not be able to reach the bathroom in time. During the bladder training program, a change in environment may be an effective suggestion for the client. It is not so essential to assess the client's history of allergy, occupation, and smoking habits before beginning a bladder training program.

Rebleeding may occur from a peptic ulcer and often warrants surgical interventions. Signs of bleeding include which of the following? Mental confusion Bradypnea Hypertension Bradycardia

Mental confusion Signs of bleeding include tachycardia, tachypnea, hypotension, mental confusion, thirst, and oliguria.

Vomiting results in which of the following acid-base imbalances? Metabolic alkalosis Respiratory acidosis Metabolic acidosis Respiratory alkalosis

Metabolic alkalosis Vomiting results in loss of hydrochloric acid (HCl) and potassium from the stomach, leading to a reduction of chlorides and potassium in the blood and to metabolic alkalosis.

Which of the following is considered a bulk-forming laxative? Dulcolax Mineral oil Milk of Magnesia Metamucil

Metamucil Metamucil is a bulk-forming laxative. Milk of Magnesia is classified as a saline agent. Mineral oil is a lubricant. Dulcolax is a stimulant.

A client with obesity is diagnosed with type 2 diabetes. In order to promote weight loss in the client and aid in glucose management, which medication will the nurse anticipate the health care provider ordering? Glyburide Metformin Glipizide Pioglitazone

Metformin Metformin (Glucophage) is a diabetes medication that also promotes weight loss. The other medications are diabetes medications; however, these promote weight gain, not weight loss.

The nurse is conducting a community education class on gastritis. The nurse includes that chronic gastritis caused by Helicobacter pylori is implicated in which disease/condition? Systemic infection Pernicious anemia Peptic ulcers Colostomy

Peptic ulcers Chronic gastritis caused by Helicobacter pylori is implicated in the development of peptic ulcers. Chronic gastritis is sometimes associated with autoimmune disease, such as pernicious anemia, but not as a cause of the anemia. Chronic gastritis is not implicated in system infections and/or colostomies.

A client is scheduled for a renal angiography. Which of the following would be appropriate before the test? Assess the client's mental changes. Monitor the client for signs of electrolyte and water imbalance. Monitor the client for an allergy to iodine contrast material. Evaluate the client for periorbital edema.

Monitor the client for an allergy to iodine contrast material. A renal angiography procedure will be contraindicated if the client is allergic to iodine contrast material. Therefore, it is important for the nurse to monitor the client for an allergy to iodine contrast material. The nurse monitors the client for the signs of electrolyte and water imbalance, mental changes, and periorbital edema at any time regardless of the test being done.

A patient has had surgery to create an ileal conduit for urinary diversion. What is a priority intervention by the nurse in the postoperative phase of care? Monitor urine output hourly and report output less than 30 mL/hr. Clean the stoma with soap and water after the patient voids. Turn the patient every 2 hours around the clock. Administer pain medication every 2 hours.

Monitor urine output hourly and report output less than 30 mL/hr. In the immediate postoperative period, urine volumes are monitored hourly. Throughout the patient's hospitalization, the nurse monitors closely for complications, reports signs and symptoms of them promptly, and intervenes quickly to prevent their progression. If urinary drainage stops or decreases to less than 30 mL/hour, or if the client complains of back pain, the nurse needs to notify the physician immediately.

A nurse is giving a client barium swallow test. What is the most important assessment a nurse would make to ensure that a client does not retain any barium after a barium swallow? Monitoring the stool passage and its color. Observing the color of urine. Placing any stool passed in a specific preservative. Monitoring the volume of urine.

Monitoring the stool passage and its color. Explanation: Monitoring stool passage and its color will ensure that the client remains barium free following a barium swallow test. The white or clay color of the stool would indicate barium retention. The stool should be placed in a special preservative if the client undergoes a stool analysis. Observing the color and volume of urine will not ensure that the client is barium free because barium is not eliminated through urine but through stool.

Which is a true statement regarding gastric cancer? Most clients are asymptomatic during the early stage of the disease. Most cases are discovered before metastasis. Women have a higher incidence of gastric cancer. The prognosis for gastric cancer is good.

Most clients are asymptomatic during the early stage of the disease. Most clients are asymptomatic during the early stage of the disease. Men have a higher incidence of gastric cancer. The prognosis is poor because the diagnosis is usually made late because most clients are asymptomatic during the early stage. Most cases of gastric cancer are discovered only after local invasion has advanced or metastases are present.

The wall of the bladder has four layers. Which of the following layers contains a membrane that prevents reabsorption of urine stored in the bladder? Mucosal Detrusor Adventitia Connective tissue

Mucosal Beneath the detrusor is a submucosal layer of loose connective tissue that serves as an interface between the detrusor and the innermost layer, a mucosal lining. This inner layer contains specialized transitional cell epithelium, a membrane that is impermeable to water and prevents reabsorption of urine stored in the bladder.

Celiac sprue is an example of which category of malabsorption? Postoperative malabsorption Mucosal disorders causing generalized malabsorption Infectious diseases causing generalized malabsorption Luminal problems causing malabsorption

Mucosal disorders causing generalized malabsorption In addition to celiac sprue, regional enteritis and radiation enteritis are examples of mucosal disorders. Examples of infectious diseases causing generalized malabsorption include small-bowel bacterial overgrowth, tropical sprue, and Whipple disease. Examples of luminal problems causing malabsorption include bile acid deficiency, Zollinger-Ellison syndrome, and pancreatic insufficiency. Postoperative gastric or intestinal resection can result in development of malabsorption syndromes.

The nurse in the ED admits a client with suspected gastric outlet obstruction. The client's symptoms include nausea and vomiting. The nurse anticipates that the physician will issue which order? Stool specimen Nasogastric tube insertion Oral contrast Pelvic x-ray

Nasogastric tube insertion The nurse anticipates an order for nasogastric tube insertion to decompress the stomach. Pelvic x-ray, oral contrast, and stool specimens are not indicated at this time.

A nurse has been asked to speak to a local women's group about preventing cystitis. Which of the following would the nurse include in the presentation? Suggestion to take tub baths instead of showers Need to urinate after engaging in sexual intercourse Importance of urinating every 4 to 6 hours while awake Need to wear underwear made from synthetic material

Need to urinate after engaging in sexual intercourse Measures to prevent cystitis include voiding after sexual intercourse, wearing cotton underwear, urinating every 2 to 3 hours while awake, and taking showers instead of tub baths.

The nurse is reviewing the laboratory test results of a client with Crohn's disease. Which of the following would the nurse most likely find? Decreased white blood cell count Negative stool cultures Decreased erythrocyte sedimentation rate Increased albumin levels

Negative stool cultures Stool cultures fail to reveal an etiologic microorganism or parasite, but occult blood and white blood cells (WBCs) often are found in the stool. Results of blood studies indicate anemia from chronic blood loss and nutritional deficiencies. The WBC count and erythrocyte sedimentation rate may be elevated, confirming an inflammatory disorder. Serum protein and albumin levels may be low because of malnutrition.

The nurse is admitting a client with a diagnosis of diverticulitis and assesses that the client has a board-like abdomen, no bowel sounds, and reports of severe abdominal pain. What is the nurse's first action? Start an IV with lactated Ringer's solution. Notify the health care provider. Administer a retention enema. Administer an opioid analgesic.

Notify the health care provider. Abdominal pain, a rigid board-like abdomen, loss of bowel sounds, and signs and symptoms of shock occur with peritonitis. Peritonitis is typically a life-threatening emergency that requires prompt surgical intervention, and typically involves postoperative critical care monitoring due to the risk of sepsis, organ failure, and subsequent infections; thus, the nurse should notify the health care provider.

A client undergoes total gastrectomy. Several hours after surgery, the nurse notes that the client's nasogastric (NG) tube has stopped draining. How should the nurse respond? Notify the health care provider. Reposition the tube. Increase the suction level. Irrigate the tube.

Notify the health care provider. The nurse should notify the health care provider because an NG tube that fails to drain during the postoperative period may be clogged, which could increase pressure on the suture site because fluid isn't draining adequately. Repositioning or irrigating an NG tube in a client who has undergone gastric surgery can disrupt the anastomosis. Increasing the level of suction may cause trauma to GI mucosa or the suture line.

The nurse working in the recovery room is caring for a client who had a radical neck dissection. The nurse notices that the client makes a coarse, high-pitched sound upon inspiration. Which intervention by the nurse is appropriate? Document the presence of stridor Lower the head of the bed Notify the physician Administer a breathing treatment

Notify the physician The presence of stridor, a coarse, high-pitched sound upon inspiration, in the immediate postoperative period following radical neck dissection, indicates obstruction of the airway, and the nurse must report it immediately to the physician.

A client has been taking a 10-day course of antibiotics for pneumonia. The client has been having white patches that look like milk curds in the mouth. What treatment will the nurse educate the client about? Cephalexin Nystatin Acyclovir Fluocinolone acetonide oral base gel

Nystatin Candidiasis is a fungal infection that results in a cheesy white plaque in the mouth that looks like milk curds. It commonly occurs in antibiotic therapy. Antifungal medications such as nystatin (Mycostatin), amphotericin B, clotrimazole, or ketoconazole may be prescribed.

Peptic ulcer disease occurs more frequently in people with which blood type? O B A AB

O People with blood type O are more susceptible to peptic ulcers than those with blood type A, B, or AB.

When preparing a client for surgery to treat appendicitis, the nurse formulates a nursing diagnosis of Risk for infection related to inflammation, perforation, and surgery. What is the rationale for choosing this nursing diagnosis? Infection of the appendix diminishes necrotic arterial blood flow and increases venous drainage. The appendix may develop gangrene and rupture, especially in a middle-aged client. Obstruction of the appendix may increase venous drainage and cause the appendix to rupt

Obstruction of the appendix reduces arterial flow, leading to ischemia, inflammation, and rupture of the appendix. A client with appendicitis is at Risk for infection related to inflammation, perforation, and surgery because obstruction of the appendix causes mucus fluid to build up, increasing pressure in the appendix and compressing venous outflow drainage. The pressure continues to rise with venous obstruction; arterial blood flow then decreases, leading to ischemia from lack of perfusion. Inflammation and bacterial growth follow, and swelling continues to raise pressure within the appendix, resulting in gangrene and rupture. Elderly, not middle-aged, clients are especially susceptible to appendix rupture.

A creatinine level has been ordered. The nurse prepares to: Obtain a blood specimen. Collect the client's urine for 24 hours. Obtain a clean catch urine. Straight cath for a specimen.

Obtain a blood specimen. A creatinine level is determined from a blood sample. It is used to assess renal function.

A male client, scheduled for a renal angiography, expresses his fear and anxiety to the nurse about the use of intravenous contrast medium substances in the test. Which of the following would be most appropriate for the nurse to do to help him overcome his apprehension? Arrange for a radioactive expert to have a talk with the client. Offer assurance about the safety of contrast media substances. Discuss the client's anxiety with the physician. Distract the client's attention from the test.

Offer assurance about the safety of contrast media substances. Because the client is anxious about the use of intravenous contrast media for a renal angiography, the nurse should offer him assurance about the safety of these substances which are iodine based. The nurse can do this by confirming that the substances are safe and ordinarily pose no danger to the client or others. The test would be contraindicated if the client had an allergy to iodine or seafood. It is not necessary to discuss the client's anxiety with the physician or ask an expert to talk with the client. More important than the technical details, the client requires assurance and comforting words about the test experience that will help him gain confidence.

A nurse practitioner prescribes drug therapy for a patient with peptic ulcer disease. Choose the drug that can be used for 4 weeks and has a 90% chance of healing the ulcer. Cimetidine Famotidine Omeprazole Nizatidine

Omeprazole Omeprazole (Prilosec) is a proton pump inhibitor that, if used according to the health care provider's directions, will result in healing in 90% of patients. The other drugs are H2 receptor antagonists that need to be used for 6 weeks.

Which medication classification represents a proton (gastric acid) pump inhibitor? Famotidine Omeprazole Metronidazole Sucralfate

Omeprazole Omeprazole decreases gastric acid by slowing the hydrogen-potassium adenosine triphosphatase pump on the surface of the parietal cells. Sucralfate is a cytoprotective drug. Famotidine is a histamine-2 receptor antagonist. Metronidazole is an antibiotic, specifically an amebicide.

Urine specific gravity is a measurement of the kidney's ability to concentrate and excrete urine. The specific gravity measures urine concentration by measuring the density of urine and comparing it with the density of distilled water. Which is an example of how urine concentration is affected? On a hot day, a person who is perspiring profusely and taking little fluid has low urine output with a high specific gravity. When the kidneys are diseased, the ability to concentrate urine may be impair

On a hot day, a person who is perspiring profusely and taking little fluid has low urine output with a high specific gravity. On a hot day, a person who is perspiring profusely and taking little fluid has low urine output with a high specific gravity. A person who has a high fluid intake and who is not losing excessive water from perspiration, diarrhea, or vomiting has copious urine output with a low specific gravity. When the kidneys are diseased, the ability to concentrate urine may be impaired and the specific gravity remains relatively constant.

A client with obesity reports pain in the joints. Which musculoskeletal condition related to obesity does the nurse suspect the client has? Osteoarthritis Inflammatory arthritis Necrotizing arthritis Rheumatoid arthritis

Osteoarthritis Osteoarthritis is an obesity-related musculoskeletal condition. Rheumatoid arthritis, inflammatory arthritis, and necrotizing arthritis are not obesity-related conditions.

A client is in the hospital for the treatment of peptic ulcer disease. The client reports vomiting and a sudden severe pain in the abdomen. The nurse then assesses a board-like abdomen. What does the nurse suspect these symptoms indicate? A reaction to the medication given for the ulcer Gastric penetration Ineffective treatment for the peptic ulcer Perforation of the peptic ulcer

Perforation of the peptic ulcer Signs and symptoms of perforation include the following: Sudden, severe upper abdominal pain (persisting and increasing in intensity), which may be referred to the shoulders, especially the right shoulder, because of irritation of the phrenic nerve in the diaphragm; vomiting; collapse (fainting); extremely tender and rigid (board-like) abdomen; and hypotension and tachycardia, indicating shock.

A nurse researcher studies the pathophysiology and etiology of obesity. What does the nurse discover is true regarding the "thrifty gene" theory of obesity? A single gene mutation is responsible for the epidemic. Over time, we have become efficient in food storage and deposition of fat stores. Over time, we have become less efficient in hunting and gathering of food. Multiple mutations of genes over time have lead to the epidemic.

Over time, we have become efficient in food storage and deposition of fat stores. According to the "thrifty gene" theory, hunting for scarce food sources during prehistoric times consumed a lot of energy, and food sources were not abundant. Storing fat to provide energy sources during times of food scarcity was a physiologic adaptive response to these environmental challenges and over time, we became more efficient in food storage and fat deposition.

Which of the following is considered an early symptom of gastric cancer? Weight loss Dyspepsia Pain relieved by antacids Bloating after meals

Pain relieved by antacids Symptoms of early disease, such as pain relieved by antacids, resemble those of benign ulcers and are seldom definitive. Symptoms of progressive disease include weight loss, bloating after meals, and dyspepsia.

A nurse is reviewing the history and physical examination of a client with a suspected malignant tumor of the bladder. Which finding would the nurse identify as the most common initial symptom? Fever Urinary retention Frequency Painless hematuria

Painless hematuria The most common first symptom of a malignant tumor of the bladder is painless hematuria. Additional early symptoms include UTI with symptoms such as fever, dysuria, urgency, and frequency. Later symptoms are related to metastases and include pelvic pain, urinary retention (if the tumor blocks the bladder outlet), and urinary frequency from the tumor occupying bladder space.

A 64-year-old man is seeing his urologist for an annual check-up, post prostatectomy. The health care provider is concerned with the symptom he finds because it is considered diagnostic for bladder cancer. Which of the following signs/symptoms is diagnostic for bladder cancer? Deep flank and abdominal pain Muscle spasm and abdominal rigidity over the flank Painless, gross hematuria Decreasing kidney function associated with fever and hematuria

Painless, gross hematuria Although flank pain may occur, the painless, gross hematuria is characteristic of bladder cancer.

The nurse is working with clients with digestive tract disorders. Which of the following organs does the nurse realize has effects as an exocrine gland and an endocrine gland? Gallbladder Liver Pancreas Stomach

Pancreas Explanation: The pancreas is both an exocrine gland, one that releases secretions into a duct or channel, and an endocrine gland, one that releases substances directly into the bloodstream. The other organs have a variety of functions but do not have a combination function such as the pancreas.

A 66-year-old African-American client has recently visited a physician to confirm a diagnosis of gastric cancer. The client has a history of tobacco use and was diagnosed 10 years ago with pernicious anemia. He and his family are shocked about the possibility of cancer because he was asymptomatic prior to recent complaints of pain and multiple gastrointestinal symptoms. On the basis of knowledge of disease progression, the nurse assumes that organs adjacent to the stomach are also affected. Whic

Pancreas Duodenum Liver Most gastric cancers are adenocarcinomas; they can occur anywhere in the stomach. The tumor infiltrates the surrounding mucosa, penetrating the wall of the stomach and adjacent organs and structures. The liver, pancreas, esophagus, and duodenum are often already affected at the time of diagnosis. Metastasis through lymph to the peritoneal cavity occurs later in the disease.

The nurse is assisting the health care provider with a gastric acid stimulation test for a client. What medication should the nurse prepare to administer subcutaneously to stimulate gastric secretions? Atropine Glycopyrronium bromide Pentagastrin Acetylcysteine

Pentagastrin Explanation: The gastric acid stimulation test usually is performed in conjunction with gastric analysis. Histamine or pentagastrin is administered subcutaneously to stimulate gastric secretions.

Which of the following is an enzyme secreted by the gastric mucosa? Trypsin Bile Ptyalin Pepsin

Pepsin Explanation: Pepsin is secreted by the gastric mucosa. Trypsin is secreted by the pancreas. The salivary glands secrete ptyalin. The liver and gallbladder secrete bile.

A client is in the hospital for the treatment of peptic ulcer disease. The client reports vomiting and a sudden severe pain in the abdomen. The nurse then assesses a board-like abdomen. What does the nurse suspect these symptoms indicate? Ineffective treatment for the peptic ulcer Gastric penetration A reaction to the medication given for the ulcer Perforation of the peptic ulcer

Perforation of the peptic ulcer Signs and symptoms of perforation include the following: Sudden, severe upper abdominal pain (persisting and increasing in intensity), which may be referred to the shoulders, especially the right shoulder, because of irritation of the phrenic nerve in the diaphragm; vomiting; collapse (fainting); extremely tender and rigid (board-like) abdomen; and hypotension and tachycardia, indicating shock.

The nurse is providing an education program for the nursing assistants in a long-term care facility in order to decrease the number of UTIs in the female population. What interventions should the nurse introduce in the program? Select all that apply. Encourage patients to wear briefs. For those patients who are incontinent, insert indwelling catheters. Perform hand hygiene prior to patient care. Provide careful perineal care. Assist the patients with frequent toileting.

Perform hand hygiene prior to patient care. Assist the patients with frequent toileting. Provide careful perineal care. In institutionalized older patients, such as those in long-term care facilities, infecting pathogens are often resistant to many antibiotics. Diligent hand hygiene, careful perineal care, and frequent toileting may decrease the incidence of UTIs.

If an indwelling catheter is necessary, which nursing intervention should be implemented to prevent infection? Use sterile technique to disconnect the catheter from the tubing to obtain urine specimens Perform meticulous perineal care daily with soap and water Use clean technique during insertion Place the catheter bag on the client's abdomen when moving the client

Perform meticulous perineal care daily with soap and water Cleanliness of the area will reduce potential for infection. Strict aseptic technique must be used when inserting a urinary bladder catheter. The nurse must maintain a closed system and use the catheter's port to obtain specimens. The catheter bag must never be placed on the client's abdomen unless it is clamped because it may cause urine to flow back from the tubing into the bladder.

A client with a diagnosis of acute appendicitis is awaiting surgical intervention. The nurse listens to bowel sounds and hears none and observes that the abdomen is rigid and board-like. What complication does the nurse determine may be occurring at this time? Peritonitis Constipation Accumulation of gas Paralytic ileus

Peritonitis Lack of bowel motility typically accompanies peritonitis. The abdomen feels rigid and board-like as it distends with gas and intestinal contents. Bowel sounds typically are absent. The diagnosis of acute appendicitis correlates with the symptoms of rupture of the appendix and peritonitis. A paralytic ileus and gas alone do not produce these symptoms.

The nurse is monitoring a client's postoperative course after an appendectomy. The nurse's assessment reveals that the client has vomited, has abdominal tenderness and rigidity, and has tachycardia. The nurse reports to the physician that the client has signs/symptoms of which complication? Pelvic abscess Peritonitis Ileus Hemorrhage

Peritonitis The nurse should report to the physician that the client has signs/symptoms of peritonitis. Signs/symptoms of a pelvic abscess include anorexia, chills, fever, diaphoresis, and diarrhea. Signs/symptoms of an ileus include absent bowel sounds, nausea, and abdominal distention. Signs/symptoms of hemorrhage include tachycardia, hypotension, anxiety, and bleeding.

A client undergoing a diagnostic examination for gastrointestinal disorder was given polyethylene glycol/electrolyte solution as a part of the test preparation. Which of the following measures should the nurse take once the solution is administered? Instruct the client to have low-residue meals. Allow the client to ingest fat-free meal. Provide saline gargles to the client. Permit the client to drink only clear liquids.

Permit the client to drink only clear liquids. Explanation: After polyethylene glycol/electrolyte solution is administered, the client should have clear liquids because this ensures watery stools, which are necessary for procedures like a barium enema. Allowing the client to ingest a fat-free meal is used in preparation for oral cholecystography. Instructing the client to have low-residue meals is a pretest procedure for barium enema. A client is offered saline gargles after esophagogastroduodenoscopy.

A nurse cares for a client with obesity who reports taking "a medication of weight loss" but cannot remember the name of it. The client also reports nervousness and feeling "jittery". Which medication is the client most likely taking? Phentermine Lorcaserin Orlistat Naltrexone/bupropion

Phentermine Phentermine is a sympathomimetic amine that stimulates central noradrenergic receptors, causing appetite suppression. Feeling jittery and nervousness is associated with this type of medication. The other answer choices represent treatment options for obesity; however, these do not cause the client's symptoms.

An 82-year-old client experiences urinary incontinence. Which factor should the nurse assess before beginning a bladder training program for this client? Occupational history History of allergies Smoking habits Physical and environmental conditions

Physical and environmental conditions It is essential to assess the client's physical and environmental conditions before beginning a bladder training program, because the client may not be able to reach the bathroom in time. During the bladder training program, a change in environment may be an effective suggestion for the client. It is not so essential to assess the client's history of allergy, occupation, and smoking habits before beginning a bladder training program.

Which characteristic is seen with a healthy stoma? No bleeding when cleansing the stoma Pink color Painful Dry in appearance

Pink color Characteristics of a normal stoma include a pink and moist appearance. It is insensitive to pain because it has no nerve endings. The area is vascular and may bleed when cleaned.

The client has returned to the floor following a radical neck dissection. Anesthesia has worn off. What is the nurse's priority action? Administer morphine for report of pain. Provide feeding through the gastrostomy tube. Empty the Jackson-Pratt device (portable drainage device). Place the client in the Fowler's position.

Place the client in the Fowler's position. All the options are activities the nurse may do; however, the nurse has to prioritize according to Maslow's hierarchy of needs. Physiological needs are addressed first. Under physiological needs, ABCs (airway, breathing, circulation) take priority. Placing the client in the Fowler's position facilitates breathing and promotes comfort.

Which diagnostic produces images of the body by detecting the radiation emitted from radioactive substances? Magnetic resonance imaging (MRI) Computed tomography (CT) Positron emission tomography (PET) Fibroscopy

Positron emission tomography (PET) Explanation: PET produces images of the body by detecting the radiation emitted from radioactive substances. CT provides cross-sectional images of abdominal organs and structures. MRI uses magnetic fields and radio waves to produce an image of the area being studied. Fibroscopy of the upper GI tract allows direct visualization of the esophageal, gastric, and duodenal mucosa through a lighted endoscope.

Retention of which electrolyte is the most life-threatening effect of renal failure? Phosphorous Calcium Potassium Sodium

Potassium Retention of potassium is the most life-threatening effect of renal failure.

The nurse is caring for a client who is scheduled for a percutaneous liver biopsy. Which diagnostic test is obtained prior? Prothrombin time (PT) Erythrocyte sedimentation rate (ESR) Blood chemistry Complete blood count (CBC)

Prothrombin time (PT) Explanation: The client must have coagulation studies before the procedure such as a PT or PTT because a major complication after a liver biopsy is bleeding. Clients at risk for serious bleeding may receive precautionary vitamin K. A complete blood count and blood chemistry may be completed for baseline values. Typically, an ESR is not associated with the procedure.

A patient has been diagnosed with a hiatal hernia. The nurse explains the diagnosis to the patient and his family by telling them that a hernia is a (an): Protrusion of the upper stomach into the lower portion of the thorax. Twisting of the duodenum through an opening in the diaphragm. Involution of the esophagus, which causes a severe stricture. Extension of the esophagus through an opening in the diaphragm.

Protrusion of the upper stomach into the lower portion of the thorax. It is important for the patient and his family to understand the altered association between the esophagus and the stomach. The diaphragm opening, through which the esophagus passes, becomes enlarged and part of the upper stomach moves up into the lower portion of the thorax. The abnormality is not an involuntary, protruding, or twisted segment.

A client is scheduled for a renal arteriogram. When the nurse checks the chart for allergies to shellfish or iodine, she finds no allergies recorded. The client is unable to provide the information. During the procedure, the nurse should be alert for which finding that may indicate an allergic reaction to the dye used during the arteriogram? Unusually smooth skin Pruritus Hypoventilation Increased alertness

Pruritus (itching) The nurse should be alert for pruritus and urticaria (hives), which may indicate a mild anaphylactic reaction to the arteriogram dye. Decreased (not increased) alertness may occur as well as dyspnea- shortness of breath (not hypoventilation). Unusually smooth skin isn't a sign of anaphylaxis.

Which term refers to inflammation of the renal pelvis? Pyelonephritis Cystitis Interstitial nephritis Urethritis

Pyelonephritis Pyelonephritis is an upper urinary tract inflammation, which may be acute or chronic. Cystitis is inflammation of the urinary bladder. Urethritis is inflammation of the urethra. Interstitial nephritis is inflammation of the kidney.

A patient with a UTI is having burning and pain when urinating. What urinary analgesic is prescribed for relief of these symptoms? Pyridium Bactrim Septra Levaquin

Pyridium The urinary analgesic agent phenazopyridine (Pyridium) is used specifically for relief of burning, pain, and other symptoms associated with UTI.

A client who is postoperative from bariatric surgery reports foul-smelling, fatty stools. What is the nurse's understanding of the primary reason for this finding? Excessive fat intake Rapid gastric dumping Decreased motility Decreased gastric size

Rapid gastric dumping Rapid gastric dumping may lead to steatorrhea, excessive fat in the feces. The primary cause of this finding is rapid gastric dumping. Excessive fat intake can make the problem worse; however, this is not the primary cause of the symptoms. Steatorrhea results from increased motility, not decreased and the size of the stomach does not contribute to this finding.

When interviewing a client with internal hemorrhoids, what would the nurse expect the client to report? Itching Soreness Rectal bleeding Pain

Rectal bleeding Internal hemorrhoids cause bleeding but are less likely to cause pain, unless they protrude through the anus. External hemorrhoids may cause few symptoms, or they can produce pain, itching, and soreness of the anal area.

Which of the following is the most common symptom of a polyp? Rectal bleeding Anorexia Abdominal pain Diarrhea

Rectal bleeding The most common symptom is rectal bleeding. Lower abdominal pain may also occur. Diarrhea and anorexia are clinical manifestations of ulcerative colitis.

Following ingestion of carrots or beets, the nurse would expect which alteration in stool color? Yellow Black Milky white Red

Red Explanation: Carrots or beets will tend to change the stool color to red. Black stools are associated with iron, licorice, and charcoal. Senna is associated with yellow stools. A milky white stool is associated with administration of barium.

Sympathomimetics have which of the following effects on the body? Relaxation of bladder wall Constriction of bronchioles Decrease of heart rate Constriction of pupils

Relaxation of bladder wall Sympathomimetics mimic the sympathetic nervous system, causing increased heart rate and contractility, dilation of bronchioles and pupils, and bladder wall relaxation.

A woman comes to her health care provider's office with signs and symptoms of kidney stones. Which of the following should be the primary medical management goal? Relieve the pain. Determine the stone type. Prevent nephron destruction. Relieve any obstruction.

Relieve the pain. The immediate objective is to relieve pain, which can be incapacitating depending on the location of the stone.

A 59-year-old woman with a recent history of heartburn, regurgitation, and occasional dysphagia has been diagnosed with a sliding hiatal hernia following an upper GI series. The nurse is providing patient education about the management of this health problem. What should the nurse suggest as a management strategy to this patient? Minimizing her intake of highly spiced foods and dairy products Drinking one to two glasses of water before and after each meal Remaining upright for at least 1 hour f

Remaining upright for at least 1 hour following each meal Management for a sliding hernia includes frequent, small feedings that can pass easily through the esophagus. The patient is advised not to recline for 1 hour after eating, to prevent reflux or movement of the hernia, and to elevate the head of the bed on 4- to 8-inch (10- to 20-cm) blocks to prevent the hernia from sliding upward. Fluid intake is encouraged, but this should be ingested throughout a meal, not just before and after the meal. It is not necessary to refrain from drinking alcohol, spicy foods, or dairy products.

Which of the following is the most successful treatment for gastric cancer? Radiation Chemotherapy Palliation Removal of the tumor

Removal of the tumor There is no successful treatment for gastric carcinoma except removal of the tumor. If the tumor can be removed while it is still localized to the stomach, the patient may be cured. If the tumor has spread beyond the area that can be excised, cure is less likely.

The nurse recognizes that a referral for genetic counseling is inappropriate for the client with: Wilms' tumor Polycystic kidney disease Alport syndrome Renal calculi

Renal calculi Wilms' tumor, polycystic disease, and Alport are conditions that have a genetic influence. Renal calculi are not influenced by genetic factors.

A nurse caring for a client with obesity recognizes the client is at risk for renal complications related to obesity. Which disease or condition is associated with obesity? Glomerulonephritis Rhabdomyolysis Nephrolithiasis Renal cancer

Renal cancer Renal cancer is associated with obesity. Rhabdomyolysis, glomerulonephritis, and nephrolithiasis are all renal conditions; however, these are not directly associated with obesity.

The nephrons are the functional units of the kidney, responsible for the initial formation of urine. The nurse knows that damage to the area of the kidney where the nephrons are located will affect urine formation. Identify that area. Renal pelvis Renal papilla Renal medulla Renal cortex

Renal cortex The majority of nephrons (80% to 85%) are located in the renal cortex. The remaining 15% to 20% are located deeper in the cortex.

A client with obesity is interested in trying orlistat for weight loss. Which disease or condition in the client's medical history alert the nurse of potential complications if the client uses this medication? Renal insufficiency Chronic obstructive pulmonary disease Anemia Diabetes mellitus

Renal insufficiency Clients with a history of renal sufficiency or liver disease should use caution while taking this medication as it has been linked to increase rates of cholelithiasis and liver failure. The other conditions do not pose an increase risk with this medication.

A nurse is describing the renal system to a client with a kidney disorder. Which structure would the nurse identify as emptying into the ureters? Nephron Renal pelvis Glomerulus Parenchyma

Renal pelvis The renal pelvis empties into the ureter which carries urine to the bladder for storage. The nephron consists of the glomerulus, afferent arteriole, efferent arteriole, Bowman's capsule, distal and proximal convoluted tubules, the loop of Henle, and collecting tubule. The nephron is located in the cortex and carries out the functions of the kidney. The parenchyma is made up of the cortex and medulla.

Which of the following hormones is secreted by the juxtaglomerular apparatus? Renin Aldosterone Calcitonin Antidiuretic hormone (ADH)

Renin Renin is a hormone directly involved in the control of arterial blood pressure; it is essential for proper functioning of the glomerulus. ADH, also known as vasopressin, plays a key role in the regulation of extracellular fluid by excreting or retaining water. Calcitonin regulates calcium and phosphorous metabolism.

The nurse is aware, when caring for patients with renal disease, that which substance made in the glomeruli directly controls blood pressure? Albumin Renin Vasopressin Cortisol

Renin Renin is directly involved in the control of arterial blood pressure. It is also essential for the proper functioning of the glomerulus and management of the body's renin-angiotensin system (RAS).

A nurse works in a bariatric clinic and cares for client with obesity who will or have undergone bariatric surgery. What is the nurse's understanding of how the procedure works? Restricts the client's ability to digest fat. Restricts the client's ability to eat. Impairs gastric motility. Impairs caloric absorption.

Restricts the client's ability to eat. Bariatric surgical procedures work by restricting a patient's ability to eat (restrictive procedure), interfering with ingested nutrient absorption (malabsorptive procedures), or both. Bariatric procedures do not impair caloric absorption; rather, nutrients are impaired by malabsorption.

A client is admitted with a diagnosis of acute appendicitis. When assessing the abdomen, the nurse would expect to find rebound tenderness at which location? Right upper quadrant Right lower quadrant Left lower quadrant Left upper quadrant

Right lower quadrant The pain of acute appendicitis localizes in the right lower quadrant (RLQ) at McBurney's point, an area midway between the umbilicus and the right iliac crest. Often, the pain is worse when manual pressure near the region is suddenly released, a condition called rebound tenderness.

Following percutaneous nephrolithotomy, the client is at greatest risk for which nursing diagnosis? Risk for infection Risk for altered urinary elimination Risk for deficient knowledge: self-catherization Risk for fluid volume excess

Risk for infection Percutaneous nephrolithotomy is an invasive procedure for the removal of renal calculi. The client would be at risk for infection.

The client asks the nurse about the functions of the kidney. Which should the nurse include when responding to the client? Select all that apply. Secretion of insulin Vitamin B production Regulation of blood pressure Secretion of prostaglandins Vitamin D synthesis

Secretion of prostaglandins Regulation of blood pressure Vitamin D synthesis Functions of the kidney include secretion of prostaglandins, regulation of blood pressure, and synthesis of aldosterone and vitamin D. The pancreas secretes insulin. The body does not produce Vitamin B.

The nurse is instructing the client who was newly diagnosed with peptic ulcers. Which of the following diagnostic studies would the nurse anticipate reviewing with the client? A sigmoidoscopy A complete blood count including differential Gastric analysis Serum antibodies for H. pylori

Serum antibodies for H. pylori Explanation: Helicobacter pylori, a bacterium, is believed to be responsible for the majority of peptic ulcers. Blood tests are used to determine whether there are antibodies to H. pylori in the blood. A complete blood count with differential can indicate bleeding and infection associated with a bleeding ulcer. A sigmoidoscopy assesses the lower gastrointestinal tract. Gastric analysis is more common in analyzing gastric fluid in determining problems with the secretory activity of the gastric mucosa.

Which term describes an inflammation of the salivary glands? Pyosis Parotitis Stomatitis Sialadenitis

Sialadenitis Sialadenitis is inflammation of the salivary glands. Parotitis is inflammation of the parotid glands. Stomatitis is inflammation of the oral mucosa. Pyosis is pus.

Which term is used to describe stone formation in a salivary gland, usually the submandibular gland? Parotitis Sialolithiasis Stomatitis Sialadenitis

Sialolithiasis Salivary stones are formed mainly from calcium phosphate. Parotitis refers to inflammation of the parotid gland. Sialadenitis refers to inflammation of the salivary glands. Stomatitis refers to inflammation of the oral mucosa.

The nurse is answering questions regarding fecal matter for a client who is scheduled for a colon resection. The client is asking questions regarding the composition of the fecal matter and when it becomes a formed mass. The nurse is most correct to state at which location? Duodenum Ileum Cecum Sigmoid colon

Sigmoid colon Explanation: Water is reabsorbed by means of diffusion across the intestinal membrane as the contents move through the colon. By the time the mixture reaches the descending and sigmoid colon, the portion of the bowel adjacent to the rectum, it is a formed mass. The ileum and duodenum are located in the small intestine. The cecum is located at the beginning of the large intestine.

A client is recovering from gastric surgery. Toward what goal should the nurse progress the client's enteral intake? Six small meals daily with 120 mL fluid between meals Six small meals and 120 mL fluid daily Three meals and three snacks and 120 mL fluid daily Three meals and 120 ml fluid daily

Six small meals daily with 120 mL fluid between meals After the return of bowel sounds and removal of the nasogastric tube, the nurse may give fluids, followed by food in small portions. Foods are gradually added until the client can eat six small meals a day and drink 120 mL of fluid between meals.

The nurse is reviewing the client's urinalysis results. The finding that is most suggestive of dehydration of the client is: Creatinine 0.7 mg/dL Bright yellow urine Protein 15 mg/dL Specific gravity 1.035

Specific gravity 1.035 Specific gravity is reflective of hydration status. A concentrated specific gravity, such as 1.035, is suggestive of dehydration. Bright yellow urine suggests ingestion of multiple vitamins. Proteinuria can be benign or be caused by conditions which alter kidney function. Creatinine measures the ability of the kidney to filter the blood. A level of 0.7 is within normal limits.

An older client is diagnosed with parotitis. What bacterial infection does the nurse suspect caused the client's parotitis? Streptococcus viridans Staphylococcus aureus Pneumococcus Pseudomonas

Staphylococcus aureus The elderly and debilitated clients experience decreased salivary flow from general dehydration or medications. The bacterial infection is usually caused by Staphylococcus aureus. The infecting organism travels from the mouth through the salivary gland.

A nurse cares for a client who is post op from bariatric surgery. Once able, the nurse encourages oral intake for what primary purpose? Assess for gastric perforation Stimulate GI peristalsis Assess for intact swallowing Stimulate digestive hormones

Stimulate GI peristalsis Early oral hydration stimulates GI peristalsis. The nurse would not give a client oral hydration to assess for intact swallowing as this may lead to aspiration. There is no reason to assume a client would have gastric perforation and this would not be appropriate. Digestive hormones are stimulated once peristalsis begins; however, this is not the primary purpose of early oral hydration.

After teaching a group of students about the various organs of the upper gastrointestinal tract and possible disorders, the instructor determines that the teaching was successful when the students identify which of the following structures as possibly being affected? Liver Large Intestine Stomach Ileum

Stomach Explanation: The upper gastrointestinal (GI) tract begins at the mouth and ends at the jejunum. Therefore, the stomach would be a component of the upper GI tract. The lower GI tract begins at the ileum and ends at the anus. The liver is considered an accessory structure.

Select the assessment finding that the nurse should immediately report, post radical neck dissection. Temperature of 99°F Pain Stridor Localized wound tenderness

Stridor Stridor is the presence of coarse, high-pitched sounds on inspiration. The nurse would auscultate frequently over the trachea. This finding must be immediately reported because it indicates airway obstruction.

The nurse is preparing the procedure room for a client who will undergo an intravenous pyelogram. Which item(s) should the nurse include? Padded tongue blades Suction equipment Dressings and tape Antihypertensive agents

Suction equipment The contrast agent injected into the client for an intravenous pyelogram is allergenic and nephrotoxic. Emergency supplies and equipment should be readily available in case the client experiences an anaphylactic reaction, including airway and suction equipment, oxygen, epinephrine, corticosteroids, and vasopressors.

Which is the procedure of choice for men with recurrent or complicated UTIs? IV urogram CT Transrectal ultrasonography MRI

Transrectal ultrasonography A transrectal ultrasonography is the procedure of choice for men with recurrent or complicated UTIs.

The nurse is preparing to examine the abdomen of a client who reports a change in bowel pattern. The nurse would place the client in which position? Left Sim's lateral Lithotomy Knee-chest Supine with knees flexed

Supine with knees flexed Explanation: When examining the abdomen, the client lies supine with knees flexed. This position assists in relaxing the abdominal muscles. The lithotomy position commonly is used for a female pelvic examination and to examine the rectum. The knee-chest position can be used for a variety of examinations, most commonly the anus and rectum. The left Sim's lateral position may be used to assess the rectum or vagina and to administer an enema.

Which is an effect of aging on upper and lower urinary tract function? Susceptibility to develop hypernatremia Increased blood flow to the kidney Increased glomerular filtration rate Acid-base balance

Susceptibility to develop hypernatremia The elderly are more susceptible to developing hypernatremia. These clients typically have a decreased glomerular filtration rate, decreased blood flow to the kidney, and acid-base imbalances.

The nurse is educating a female patient with a UTI on the pharmacologic regimen for treatment. What is important for the nurse to instruct the patient to do? Take the antibiotic as well as an antifungal for the yeast infection she will probably have. Be sure to take the medication with grapefruit juice. Take the antibiotic for 3 days as prescribed. Understand that if the infection reoccurs, the dose will be higher next time.

Take the antibiotic for 3 days as prescribed. The trend is toward a shortened course of antibiotic therapy for uncomplicated UTIs, because most cases are cured after 3 days of treatment. Regardless of the regimen prescribed, the patient is instructed to take all doses prescribed, even if relief of symptoms occurs promptly. Although brief pharmacologic treatment of UTIs for 3 days is usually adequate in women, infection recurs in about 20% of women treated for uncomplicated UTIs.

A client is admitted to the hospital for diagnostic testing to rule out colorectal cancer. Which intervention should the nurse include on the plan of care? Prepare the client for a gastrostomy tube placement. Test all stools for occult blood. Administer morphine (Duramorph PF) routinely, as ordered. Administer topical ointment to the rectal area to decrease bleeding.

Test all stools for occult blood. Blood in the stools is one of the warning signs of colorectal cancer. The nurse should plan on checking all stools for both frank and occult blood. The blood in the stool is coming from the colon or rectum; administering an ointment wouldn't help decrease the bleeding. Preparing a client for a gastrostomy tube isn't appropriate when diagnosing colorectal cancer. Colorectal cancer is usually painless; administering opioid pain medication isn't needed

A client with dysphagia is being prepared for discharge. Which outcome indicates that the client is ready for discharge? The client doesn't exhibit rectal tenesmus. The client has normal gastric structures. The client reports diminished duodenal inflammation. The client is free from esophagitis and achalasia.

The client is free from esophagitis and achalasia. Dysphagia may be the reason why a client with esophagitis or achalasia seeks treatment. Therefore, when the client is free of esophagitis or achalasia, he is ready for discharge. Dysphagia isn't associated with rectal tenesmus, duodenal inflammation, or abnormal gastric structures.

A nurse enters the room of a client who has returned to the unit after having a radical neck dissection. Which assessment finding requires immediate intervention? Serosanguineous drainage on the dressing Foley catheter bag containing 500 ml of amber urine A piggyback infusion of levofloxacin The client lying in a lateral position, with the head of bed flat

The client lying in a lateral position, with the head of bed flat A client who has had neck surgery is at risk for neck swelling. To prevent respiratory complications, the head of the bed should be at least at a 30-degree angle. This position gives the lungs room to expand and decreases swelling by promoting venous and lymphatic drainage. This position also minimizes the risk of aspiration. Serosanguineous drainage on the dressing, a Foley bag containing amber urine, and levofloxacin infusing aren't causes for concern.

A client realizes that regular use of laxatives has greatly improved bowel patterns. However, the nurse cautions this client against the prolonged use of laxatives for which reason? The client's natural bowel function may become sluggish. The client may develop inflammatory bowel disease. The client may lose his or her appetite. The client may develop arthritis or arthralgia.

The client's natural bowel function may become sluggish. It is essential for the nurse to caution the client against the prolonged use of laxatives because it decreases muscle tone in the large intestine. Prolonged use of laxatives may cause the client's natural bowel function to become sluggish. Laxatives do not increase the risk of developing inflammatory bowel disease, arthritis, or arthralgia, nor do they cause a loss in appetite.

Which is a true statement regarding regional enteritis (Crohn's disease)? It is characterized by pain in the lower left abdominal quadrant. The clusters of ulcers take on a cobblestone appearance. The lesions are in continuous contact with one another. It has a progressive disease pattern.

The clusters of ulcers take on a cobblestone appearance. The clusters of ulcers take on a cobblestone appearance. It is characterized by remissions and exacerbations. The pain is located in the lower right quadrant. The lesions are not in continuous contact with one another and are separated by normal tissue.

The nurse is caring for a client who has presented to the walk-in clinic. The client verbalizes pain on urination, feelings of fatigue, and diffuse back pain. When completing a head-to-toe assessment, at which specific location would the nurse assess the client's kidneys for tenderness? Above the symphysis pubis The costovertebral angle The upper abdominal quadrants on the left and right side Around the umbilicus

The costovertebral angle The nurse is correct to assess the kidneys for tenderness at the costovertebral angle. The other options are incorrect.

Which is an accurate statement regarding gastric cancer? Most gastric cancer-related deaths occur in people younger than 40 years. Females have a higher incidence of gastric cancers than males. A diet high in smoked foods and low in fruits and vegetables may decrease the risk of gastric cancer. The incidence of stomach cancer continues to decrease in the United States.

The incidence of stomach cancer continues to decrease in the United States. While the incidence in the United States continues to decrease, gastric cancer still accounts for 10,700 deaths annually. While gastric cancer deaths occasionally occur in younger people, most occur in people older than 40 years of age. Males have a higher incidence of gastric cancers than females. More accurately, a diet high in smoked foods and low in fruits and vegetables may increase the risk of gastric cancer.

A client with a genitourinary problem is being examined in the emergency department. When palpating the client's kidneys, the nurse should keep in mind which anatomic fact? The kidneys are situated just above the adrenal glands. The kidneys lie between the 10th and 12th thoracic vertebrae. The left kidney usually is slightly higher than the right one. The average kidney is approximately 5 cm (2 in.) long and 2 to 3 cm (0.8 to 1.2 in.) wide.

The left kidney usually is slightly higher than the right one. The left kidney usually is slightly higher than the right one. An adrenal gland lies atop each kidney. The average kidney measures approximately 11 cm (4??) long, 5 to 5.8 cm (2? to 2¼?) wide, and 2.5 cm (1?) thick. The kidneys are located retroperitoneally, in the posterior aspect of the abdomen, on either side of the vertebral column. They lie between the 12th thoracic and 3rd lumbar vertebrae.

The nurse is assisting in the transport of a client with an indwelling catheter to the diagnostic studies unit. Which action made by the nursing assistant would require instruction? The nursing assistant holds the drainage bag while the client moves to the wheelchair. The nursing assistant keeps the catheter and drainage bag together when moving the client. The nursing assistant places the drainage bag on the lower area of the wheelchair for transport. The nursing assistant places the drainage

The nursing assistant places the drainage bag on the client's abdomen for transport. The nurse would instruct the nursing assistant to maintain the drainage bag lower than the genital region to avoid a backflow of urine into the bladder. The nursing assistant is correct to move the catheter and drainage bag with the client to not put tension on the catheter, place the drainage bag on the lower area of the wheelchair, and hold the drainage bag while the client is in the process of moving.

The nurse is educating a patient about preparation for an IV urography. What should the nurse be sure to include in the preparation instructions? A liquid restriction for 8 to 10 hours before the test is required The patient will have enemas until the urine is clear. The patient may have liquids before the test. The patient is restricted from eating or drinking from midnight until after the test.

The patient may have liquids before the test. IV urography may be used as the initial assessment of many suspected urologic conditions, especially lesions in the kidneys and ureters. The patient preparation is the same as for excretory urography, except fluids are not restricted.

Which nursing instruction is correct to provide the client following a barium enema? An enema will be used to clear the bowel. The client will maintain a low residue diet. The stools may be a white or clay colored. Sips of fluid may be increased if tolerated.

The stools may be a white or clay colored. Explanation: It is important to instruct the client that it is normal to have a white- or clay-colored stool following the barium enema. The client should report the color of the stool to the nurse. A progression of clay colored stools to brown-colored stools should be noted. The client is prescribed a low-residue diet before the procedure. An increased fluid intake is offered to eliminate the barium from the bowel. The client is encouraged to move the bowel independently.

A client is to have an upper GI procedure with barium ingestion and abdominal ultrasonography. While scheduling these diagnostic tests, the nurse must consider which factor? Both tests need to be done before breakfast. The client may eat a light meal before either test. The ultrasonography should be scheduled before the GI procedure. The upper GI should be scheduled before the ultrasonography.

The ultrasonography should be scheduled before the GI procedure. Explanation: Both an upper GI procedure with barium ingestion and an ultrasonography may be completed on the same day. The ultrasonography test should be completed first, because the barium solution could interfere with the transmission of the sound waves. The ultrasonography test uses sound waves that are passed into internal body structures, and the echoes are recorded as they strike tissues. Fluid in the abdomen prevents transmission of ultrasound.

A nurse is caring for a newly admitted patient with a suspected gastrointestinal (GI) bleed. The nurse assesses the patient's stool after a bowel movement and notes it to be a tarry-black color. The nurse recognizes that the bleeding is likely occurring where? The lower GI tract The esophagus The anal area The upper GI tract

The upper GI tract Explanation: Blood shed in sufficient quantities in the upper GI tract will produce a tarry-black color (melena). Blood entering the lower portion of the GI tract or passing rapidly through it will appear bright or dark red. Lower rectal or anal bleeding is suspected if there is streaking of blood on the surface of the stool or if blood is noted on toilet tissue.

A nurse epidemiologist examines the overall decrease in life expectancy related to obesity. What finding is true? There is a 21-28 year decrease in overall life expectancy for those with obesity. There is a 25-30 year decrease in overall life expectancy for those with obesity. There is a 6-20 year decrease in overall life expectancy for those with obesity. There is a 2-4 year decrease in overall life expectancy for those with obesity.

There is a 6-20 year decrease in overall life expectancy for those with obesity. Overall, there is a 6-20 year decrease in overall life expectancy for those with obesity.

A nurse cares for a client who is post op bariatric surgery and the nurse offers the client a sugar-free beverage. What is the primary purpose of offering a sugar-free beverage? These ease gastric distention. These are less likely to cause dumping syndrome. These ease nausea. These are less likely to raise the blood sugar.

These are less likely to cause dumping syndrome. The primary purpose of offering a sugar-free beverage is that they are less likely to cause dumping syndrome in the client who is post op from bariatric surgery. Sugar-free beverages are less likely than sugary beverages to raise the blood sugar; however, this is not the primary purpose of offering the sugar-free beverage. Sugar-free beverages do not necessarily ease nausea or gastric distention.

A client informs the nurse that he is taking a stimulant laxative in order to be able to have a bowel movement daily. What should the nurse inform the client about taking a stimulant laxative? The laxative is safe to take with other medication the client is taking. The client should take a fiber supplement along with the stimulant laxative. They can be habit forming and will require increasing doses to be effective. As long as the client is drinking 8 glasses of water per day, he can continue t

They can be habit forming and will require increasing doses to be effective. The nurse should discourage self-treatment with daily or frequent enemas or laxatives. Chronic use of such products causes natural bowel function to be sluggish. In addition, laxatives continuing stimulants can be habit forming, requiring continued use in increasing doses. Although the nurse should encourage the client to have adequate fluid intake, laxative use should not be encouraged. The laxative may interact with other medications the client is taking and may cause a decrease in absorption. A fiber supplement may be taken alone but should not be taken with a stimulant laxative.

The nurse determines which is a true statement regarding older clients, considering the age-related effects on the gastrointestinal (GI) system? They have no awareness of the filling reflex. They tend to have higher physiologic reserves to compensate for fluid loss. They tend to have increased muscle tone and mass. They usually have less control of the rectal sphincter.

They usually have less control of the rectal sphincter. Explanation: Older adults tend to have fewer physiologic reserves to compensate for fluid loss. The older adult usually has less control of the rectal sphincter than a younger adult because of age-related changes in innervations, a diminished awareness of the filling reflex, and decreased muscle tone.

A nurse examines the socioeconomic impact of obesity among Americans. Which statements does the nurse understand is true? Select all that apply. Income is not related to the prevalence of disease. Education is not related to the prevalence of disease. Those with less education are impacted at a greater prevalence of disease. Those with less income are impacted at a greater prevalence of disease. Those who own their own homes have a decreased prevalence of disease.

Those with less education are impacted at a greater prevalence of disease. Those with less income are impacted at a greater prevalence of disease. The socioeconomic disparities of obesity among Americans is great. In general, those who are less educated and earn less income are more likely to have obesity. Home ownership does not decrease the prevalence of obesity.

The presence of mucus and pus in the stools suggests which condition? Small-bowel disease Intestinal malabsorption Ulcerative colitis Disorders of the colon

Ulcerative colitis The presence of mucus and pus in the stools suggests ulcerative colitis. Watery stools are characteristic of small-bowel disease. Loose, semisolid stools are associated more often with disorders of the colon. Voluminous, greasy stools suggest intestinal malabsorption.

A client has a new order for metoclopramide. What extrapyramidal side effect should the nurse assess for in the client? Anxiety or irritability Hyperactivity Uncontrolled rhythmic movements of the face or limbs Dry mouth not relieved by sugar-free hard candy

Uncontrolled rhythmic movements of the face or limbs Metoclopramide is a prokinetic agent that accelerates gastric emptying. Because metoclopramide can have extrapyramidal side effects that are increased in certain neuromuscular disorders, such as Parkinson's disease, it should be used only if no other option exists, and the client should be monitored closely for uncontrolled rhythmic movements of the face or limbs. Metoclopramide side effects are headache, confusion, and drowsiness. Anxiety, hyperactivity, and a dry mouth are not common side effects.

A client with GERD develops esophagitis. Which diagnostic test would the nurse expect the physician to order to confirm the diagnosis? Stool testing for occult blood Barium swallow 24-hour esophageal pH monitoring Upper endoscopy with biopsy

Upper endoscopy with biopsy Upper endoscopy with biopsy confirms esophagitis. Barium-swallow would reveal inflammation or stricture formation from chronic esophagitis. Tests of stool may show positive findings of blood. Ambulatory 24-hour esophageal pH monitoring allows for observation of the frequency of reflux episodes and their associated symptoms.

The client presents with nausea and vomiting, absent bowel sounds, and colicky flank pain. The nurse interprets these findings as consistent with: Interstitial cystitis Ureteral colic Acute prostatitis Urethritis

Ureteral colic These clinical manifestations are consistent with ureteral colic.

A group of students is reviewing for a test on the urinary and renal system. The students demonstrate understanding of the information when they identify which of the following as part of the upper urinary tract? Urethra Bladder Ureters Pelvic floor muscles

Ureters The upper urinary tract is composed of the kidneys, renal pelvis, and ureters. The lower urinary tract consists of the bladder, urethra, and pelvic floor muscles.

A client comes to the clinic for a follow-up visit. During the interview, the client states, "Sometimes when I have to urinate, I can't control it and do not reach the bathroom in time." The nurse suspects that the client is experiencing which type of incontinence? Functional Stress Overflow Urge

Urge Urge incontinence occurs when the client experiences the sensation to void but cannot control voiding in time to reach a toilet. Stress incontinence occurs when the client has an involuntary loss of urine that results from a sudden increase in intra-abdominal pressure. Overflow incontinence occurs when the client experiences an involuntary loss of urine related to an over distended bladder; the client voids small amounts frequently and dribbles. Functional incontinence occurs when the client has function of the lower urinary tract but cannot identify the need to void or ambulate to the toilet.

The nurse advises a patient with renal stones to avoid eating shellfish, asparagus, and organ meats. She emphasizes these foods because she knows that his renal stones are composed of which of the following substances? Cystine Struvite Uric acid Calcium

Uric acid Uric acid stones are found in patients with gout and myeloproliferative disorders. Therefore, a diet low in purines is recommended.

The most frequent reason for admission to skilled care facilities includes which of the following? Congestive heart failure Urinary incontinence Myocardial infarction Stroke

Urinary incontinence Urinary incontinence is the most common reason for admission to skilled nursing facilities.

Which statement describing urinary incontinence in an older adult client is true? Urinary incontinence isn't a disease. Urinary incontinence is a normal part of aging. Urinary incontinence in the elderly population can't be treated. Urinary incontinence is a disease.

Urinary incontinence isn't a disease. Urinary incontinence isn't a normal part of aging nor is it a disease. It may be caused by confusion, dehydration, fecal impaction, restricted mobility, or other causes. Certain medications, including diuretics, hypnotics, sedatives, anticholinergics, and antihypertensives, may trigger urinary incontinence. Most clients with urinary incontinence can be treated; some can be cured.

A group of students are reviewing information about disorders of the bladder and urethra. The students demonstrate understanding of the material when they identify which of the following as a voiding dysfunction? Urinary retention Cystitis Urethral stricture Bladder stones

Urinary retention Urinary retention and urinary incontinence are voiding dysfunctions, temporary or permanent alterations in the ability to urinate normally. Cystitis is an infectious disorder. Bladder stones and urethral stricture are obstructive disorders.

Which of the following is the most common site of a nosocomial infection? Gastrointestinal tract Skin Respiratory tract Urinary tract

Urinary tract The urinary tract is the most common site of nosocomial infection, accounting for greater than 3% of the total number reported by hospitals each year.

The nurse observes the color of the client's urine, which appears pale blue-green. The nurse obtains a drug history from the client based on the understanding that drugs used by the client may affect which of the following? Amount of urine produced Size of the urinary bladder Urine specific gravity Urinary tract tests

Urinary tract tests It is important to inquire about drugs because some drugs may affect the outcome of urinary tract tests as well as the color and odor of the urine. Dietary intake may affect urine characteristics as well as urinary tract disorders and their management. Drugs do not directly affect the size of the urinary bladder or the amount of urine produced.

The nurse is caring for a client who is describing urinary symptoms of needing to go to the bathroom with little notice. When the nurse is documenting these symptoms, which medical term will the nurse document? Urinary incontinence Urinary frequency Urinary urgency Urinary stasis

Urinary urgency The nurse would document urinary urgency. Urinary frequency is urinating more frequently than normal often times due to inadequate emptying of the bladder. Urinary incontinence is the involuntary loss of urine. Urinary stasis is a stoppage or diminution of flow.

The nurse is reviewing the client's lab results. Which lab result requires follow up by the nurse? Select all that apply. BUN 28 mg/dL Urine: RBC 20 Urine specific gravity 1.020 Serum creatinine 0.8 mg/dL Urine: WBC 1

Urine: RBC 20 BUN 28 mg/dL Hematuria (> 3RBCs) and an elevated BUN are both suggestive of a problem within the genitourinary tract. A serum creatinine of 0.8 mg/dL and a urine specific gravity of 1.020 are within normal limits. A rare white blood cell is not clinically significant.

Which of the following interventions are appropriate for clients with gastritis? Select all that apply. Discourage cigarette smoking. Provide general education about how to prevent recurrences. Use a calm approach to reduce anxiety. Give the client food and fluids every 4 hours. Notify the physician of indicators of hemorrhagic gastritis.

Use a calm approach to reduce anxiety. Discourage cigarette smoking. Notify the physician of indicators of hemorrhagic gastritis. The nurse should use a calm approach when answering questions and providing teaching. He or she should discuss smoking cessation and monitor for any indicators of hemorrhagic gastritis. The client will take nothing by mouth for up to a few days until symptoms subside. The nurse needs to develop an individualized teaching plan for the client that includes information about stress management, diet, and medications.

A nurse is providing care for a client recovering from gastric bypass surgery. During assessment, the client exhibits pallor, perspiration, palpitations, headache, and feelings of warmth, dizziness, and drowsiness. The client reports eating 90 minutes ago. What will the nurse suspect? Vasomotor symptoms associated with dumping syndrome Dehiscence of the surgical wound A normal reaction to surgery Peritonitis

Vasomotor symptoms associated with dumping syndrome Early manifestations of dumping syndrome occur 15 to 30 minutes after eating. Signs and symptoms include vertigo, tachycardia, syncope, sweating, pallor, palpitations, diarrhea, nausea, and the desire to lie down. Dehiscence of the surgical wound is characterized by pain and a pulling or popping feeling at the surgical site. Peritonitis presents with a rigid, board-like abdomen, tenderness, and fever. The client's signs and symptoms aren't a normal reaction to surgery.

Which of the following is used to identify vesicoureteral reflux? Renal angiography Voiding cystourethrography Bladder ultrasonography IV urography

Voiding cystourethrography A voiding cystourethrography is used as a diagnostic tool to identify vesicoureteral reflux. An IV urography may be used as the initial assessment of various suspected urologic problems, especially lesions in the kidneys and ureters, and it provides an approximate estimate of renal function. Renal angiography is used to evaluate renal blood flow, to differentiate renal cysts from tumors, to evaluate hypertension, and preoperatively for renal transplantation.

Which of the following assessment findings would be most important for indicating dumping syndrome in a postgastrectomy client? Persistent loose stools, chills, hiccups after eating Abdominal distention, elevated temperature, weakness before eating Constipation, rectal bleeding following bowel movements Weakness, diaphoresis, diarrhea 90 minutes after eating

Weakness, diaphoresis, diarrhea 90 minutes after eating Dumping syndrome produces weakness, dizziness, sweating, palpitations, abdominal cramping, and diarrhea from the rapid emptying of the chyme after eating. Elevated temperature and chills can be a significant finding for infection and should be reported. Constipation with rectal bleeding is not indicative of dumping syndrome.

The nurse is assigned to care for a patient in the oliguric phase of kidney failure. When does the nurse understand that oliguria is said to be present? When the urine output is between 300 and 500 mL/h When the urine output is about 100 mL/h When the urine output is between 500 and 1,000 mL/h When the urine output is less than 30 mL/h

When the urine output is less than 30 mL/h Oliguria is defined as urine output <0.5 mL/kg/h

A client is admitted to the emergency department with reports right lower quadrant pain. Blood specimens are drawn and sent to the laboratory. Which laboratory finding should be reported to the health care provider immediately? Hematocrit 42% White blood cell (WBC) count 22.8/mm3 Serum potassium 4.2 mEq/L Serum sodium 135 mEq/L

White blood cell (WBC) count 22.8/mm3 The nurse should report the elevated WBC count. This finding, which is a sign of infection, indicates that the client's appendix might have ruptured. Hematocrit of 42%, serum potassium of 4.2 mEq/L, and serum sodium of 135 mEq/L are within normal limits. Alterations in these levels don't indicate appendicitis.

While completing an abdominal assessment, the nurse will use which landmark as the upper boundary for auscultating bowel sounds? Umbilicus Xiphoid process Symphysis pubis T12 to L3 vertebrae

Xiphoid process Explanation: Understanding the division of the abdomen into four quadrants or nine regions helps the nurse to complete thorough assessment. The xiphoid process in the epigastric region is the upper boundary for auscultating bowel sounds.

A nurse is caring for a client diagnosed with acute renal failure. The nurse notes on the intake and output record that the total urine output for the previous 24 hours was 35 mL. Urine output that's less than 50 ml in 24 hours is known as: hematuria. oliguria. polyuria. anuria.

anuria. Urine output less than 50 ml in 24 hours is called anuria. Urine output of less than 400 ml in 24 hours is called oliguria. Polyuria is excessive urination. Hematuria is the presence of blood in the urine.

In a diagnosis of a lower urinary tract infection, which structures could be affected? Select all that apply. bladder urethra kidney ureter

bladder urethra The lower urinary tract consists of the bladder, urethra, and pelvic floor muscles.

A client in a short-procedure unit is recovering from renal angiography in which a femoral puncture site was used. When providing postprocedure care, the nurse should: keep the client's knee on the affected side bent for 6 hours. check the client's pedal pulses frequently. remove the dressing on the puncture site after vital signs stabilize. apply pressure to the puncture site for 30 minutes.

check the client's pedal pulses frequently. After renal angiography involving a femoral puncture site, the nurse should check the client's pedal pulses frequently to detect reduced circulation to the feet caused by vascular injury. The nurse also should monitor vital signs for evidence of internal hemorrhage and should observe the puncture site frequently for fresh bleeding. The client should be kept on bed rest for several hours so the puncture site can seal completely. Keeping the client's knee bent is unnecessary. By the time the client returns to the short-procedure unit, manual pressure over the puncture site is no longer needed because a pressure dressing is in place. The nurse should leave this dressing in place for several hours — and only remove it if instructed to do so.

The nurse prepares a client for a barium enema. The nurse should place the client on which diet prior to the procedure? soft diet 1 day prior nothing by mouth (NPO) 2 days prior high-fiber diet 1 to 2 days prior clear liquids day before

clear liquids day before Explanation: The nurse should place the client on clear liquids the evening before the procedure, a low-residue diet 1 to 2 days before the test, and NPO at midnight in preparation for the barium enema.

An older adult's most recent laboratory findings indicate a decrease in creatinine clearance. When performing an assessment related to potential causes, the nurse should: assess the client's usual intake of sodium. palpate the client's bladder before and after voiding. confirm all of the medications and supplements normally taken. confirm which beverages the client normally consumes.

confirm all of the medications and supplements normally taken. Adverse effects of medications are a common cause of decreased renal function in older adults. Quantity, rather than type, of beverages is relevant. Sodium intake does not normally cause decreased renal function. Bladder palpation can be used to confirm urinary retention, but this does not normally affect renal function as much as medications.

An older adult's most recent laboratory findings indicate a decrease in creatinine clearance. When performing an assessment related to potential causes, the nurse should: palpate the client's bladder before and after voiding. assess the client's usual intake of sodium. confirm which beverages the client normally consumes. confirm all of the medications and supplements normally taken.

confirm all of the medications and supplements normally taken. Adverse effects of medications are a common cause of decreased renal function in older adults. Quantity, rather than type, of beverages is relevant. Sodium intake does not normally cause decreased renal function. Bladder palpation can be used to confirm urinary retention, but this does not normally affect renal function as much as medications.

A client has a 10-year history of Crohn's disease and is seeing the physician due to increased diarrhea and fatigue. What is the recommended dietary approach to treat Crohn's disease? low-fiber diet dietary approach varies. lactose-rich foods high-fiber diet

dietary approach varies. The dietary approach varies. A high-fiber diet may be indicated when it is desirable to add bulk to loose stools. A low-fiber diet may be indicated in cases of severe inflammation or stricture. A high-calorie and high-protein diet helps replace nutritional losses from chronic diarrhea. The client may need nutritional supplements, depending on the area of the bowel affected. When the small intestine is inflamed, some clients experience lactose intolerance, requiring avoidance of lactose-rich foods.

The nurse is preparing a client for a nuclear scan of the kidneys. Following the procedure, the nurse instructs the client to notify the health care team if bloody urine is noted. carefully handle urine because it is radioactive. maintain bed rest for 2 hours. drink liberal amounts of fluids.

drink liberal amounts of fluids. After the procedure is completed, the client is encouraged to drink fluids to promote excretion of the radioisotope by the kidneys. The remaining instructions are not associated with a nuclear scan.

When gastric analysis testing reveals excess secretion of gastric acid, the nurse recognizes which medical diagnoses is supported? chronic atrophic gastritis gastric cancer duodenal ulcer pernicious anemia

duodenal ulcer Explanation: Clients with duodenal ulcers usually secrete an excess amount of hydrochloric acid. Clients with chronic atrophic gastritis secrete little or no acid. Clients with gastric cancer secrete little or no acid. Clients with pernicious anemia secrete no acid under basal conditions or after stimulation.

Cystic fibrosis, a genetic disorder characterized by pulmonary and pancreatic dysfunction, usually appears in young children but can also affect adults. If the pancreas was functioning correctly, where would the bile and pancreatic enzymes enter the GI system? duodenum ileum jejunum cecum

duodenum Explanation: The duodenum, which is approximately 10 inches long, is the first region of the small intestine and the site where bile and pancreatic enzymes enter.

The most common symptom of esophageal disease is nausea. vomiting. dysphagia. odynophagia.

dysphagia. Dysphagia may vary from an uncomfortable feeling that a bolus of food is caught in the upper esophagus to acute pain upon swallowing. Nausea is the most common symptom of gastrointestinal problems in general. Vomiting is a nonspecific symptom that may have a variety of causes. Odynophagia refers specifically to acute pain upon swallowing.

Although the primary function of the urinary system is the transport of urine, the kidneys perform several functions. Which is NOT a function of the kidneys? stimulating RBC production excreting nitrogen waste products regulating blood pressure excreting protein

excreting protein Although the kidneys excrete excess water and nitrogen-based waste products of protein metabolism, persistent renal excretion of protein is not the function of kidneys, which are in the state of homeostasis. The kidneys assist in maintenance of acid-base and electrolyte balance; produce the enzyme renin, which helps regulate blood pressure; and produce the hormone erythropoietin.

A client reports severe pain and bleeding while having a bowel movement. Upon inspection, the health care provider notes a linear tear in the anal canal tissue. The client is diagnosed with a: fissure. fistula. pilonidal cyst. hemorrhoid.

fissure An anal fissure (fissure in ano) is a linear tear in the anal canal tissue. An anal fistula (fistula in ano) is a tract that forms in the anal canal. Hemorrhoids are dilated veins outside or inside the anal sphincter. A pilonidal sinus is an infection in the hair follicles in the sacrococcygeal area above the anus.

When the bladder contains 400 to 500 mL of urine, this is referred to as renal clearance. anuria. specific gravity. functional capacity.

functional capacity. A marked sense of fullness and discomfort, with a strong desire to void, usually occurs when the bladder contains 400 to 500 mL of urine, referred to as the "functional capacity." Anuria is a total urine output less than 50 mL in 24 hours. Specific gravity reflects the weight of particles dissolved in the urine. Renal clearance refers to the ability of the kidneys to clear solutes from the plasma.

A client is experiencing some secretion abnormalities, for which diagnostics are being performed. Which substance is typically reabsorbed and not secreted in urine? glucose creatinine potassium chloride

glucose Amino acids and glucose typically are reabsorbed and not excreted in the urine. The filtrate that is secreted as urine usually contains water, sodium, chloride, bicarbonate, potassium, urea, creatinine, and uric acid.

The major carbohydrate that tissue cells use as fuel is glucose. chyme. proteins. fats.

glucose. Explanation: Glucose is the major carbohydrate that tissue cells use as fuel. Proteins are a source of energy after they are broken down into amino acids and peptides. Chyme stays in the small intestine for 3 to 6 hours, allowing for continued breakdown and absorption of nutrients. Ingested fats become monoglycerides and fatty acids by the process of emulsification.

A client is scheduled to undergo surgical creation of an ileal conduit. The primary nurse educates the client about surgery and the postoperative period. The nurse informs the client that many members of the health care team (including a mental health practitioner) will see him. A mental health practitioner should be involved in the client's care to: help the client cope with the anxiety associated with changes in body image. evaluate the client's need for mental health intervention. assess whe

help the client cope with the anxiety associated with changes in body image. Many clients who undergo surgery for creation of an ileal conduit experience anxiety associated with changes in body image. The mental health practitioner can help the client cope with these feelings of anxiety. Mental health practitioners don't evaluate whether the client is a surgical candidate. None of the evidence suggests that urinary diversion surgery, such as creation of an ileal conduit, places the client at risk for suicide. Although evaluating the need for mental health intervention is always important, this client displays no behavioral changes that suggest intervention is necessary at this time.

A client describes being constipated, but also experiencing abdominal cramping, pain, and urgent diarrhea. These symptoms occur more often when the client is nearing a deadline or is under emotional stress. What would be recommended to treat these symptoms? Select all that apply. cholinergic low-residue diet psyllium high-fiber diet

high-fiber diet psyllium Dietary changes reduce flatulence and abdominal discomfort. A high-fiber diet (30 to 40 g/day) or a bulk-forming agent, such as products containing psyllium, is prescribed to regulate bowel elimination. The fiber draws water into constipated stool and adds bulk to watery stool. An anticholinergic, such as dicyclomine (Bentyl), has an antispasmodic effect if taken before meals.

Diet therapy for clients diagnosed with irritable bowel syndrome (IBS) includes: high-fiber diet. caffeinated products. spicy foods. fluids with meals.

high-fiber diet. A high-fiber diet is prescribed to help control diarrhea and constipation. Foods that are possible irritants, such as caffeine, spicy foods, lactose, beans, fried foods, corn, wheat, and alcohol, should be avoided. Fluids should not be taken with meals because they cause abdominal distention.

The nurse recognizes that which risk factor does NOT predispose a client to the development of kidney stones? immobilization hypoparathyroidism hyperparathyroidism gout

hypoparathyroidism Hypoparathyroidism is not a risk factor for the development of kidney stones. Immobilization, gout, and hyperparathyroidism are risk factors.

Which response is a parasympathetic response in the GI tract? increased peristalsis decreased motility decreased gastric secretion blood vessel constriction

increased peristalsis Explanation: Increased peristalsis is a parasympathetic response in the GI tract. Decreased gastric secretion, blood vessel constriction, and decreased motility are sympathetic responses in the GI tract.

A client is prescribed amitriptyline (an antidepressant) for incontinence. The nurse understands that this drug is an effective treatment because it: increases bladder neck resistance. decreases involuntary bladder contractions. reduces bladder spasticity. increases contraction of the detrusor muscle.

increases bladder neck resistance. Some tricyclic antidepressant medications (amitriptyline, nortriptyline, and amoxapine) are useful in treating incontinence because they decrease bladder contractions and increase bladder neck resistance. Anticholinergic drugs such as oxybutynin chloride (Ditropan), reduce bladder spasticity and involuntary bladder contractions. Bethanechol (Urecholine) helps to increase contraction of the detrusor muscle, which assists with emptying of the bladder.

The nurse prepares to administer the lavage solution to a client having a colonoscopy completed. The nurse stops and notifies the physician when noting that the client has which condition? congestive heart failure chronic obstructive pulmonary disease inflammatory bowel disease pulmonary hypertension

inflammatory bowel disease Explanation: The nurse stops administering the lavage solution and notifies the physician when the nurse notes that the client has inflammatory bowel disease. Another contraindication for use of lavage solution is intestinal obstruction. Chronic obstructive pulmonary disease (COPD), congestive heart failure, and pulmonary hypertension are not contraindications to administration of lavage solution in preparation for a colonoscopy.

Bladder retraining following removal of an indwelling catheter begins with performing straight catheterization after 4 hours. advising the client to avoid urinating for at least 6 hours. instructing the client to follow a 2- to 3-hour timed voiding schedule. encouraging the client to void immediately.

instructing the client to follow a 2- to 3-hour timed voiding schedule. Immediately after the removal of the indwelling catheter, the client is placed on a timed voiding schedule, usually 2 to 3 hours, not 6 hours. At the given time interval, the client is instructed to void. Immediate voiding is not usually encouraged. If bladder ultrasound shows 100 mL or more of urine remaining in the bladder after voiding, straight catheterization may be performed to ensure complete bladder emptying.

A resident at a long-term care facility lost the ability to swallow following a stroke 4 years ago. The client receives nutrition via a PEG tube, has adapted well to the tube feedings, and remains physically and socially active. Occasionally, the client develops constipation that requires administration of a laxative to restore regular bowel function. What is the most likely cause of this client's constipation? increased fiber lack of exercise lack of free water intake lack of solid food

lack of free water intake A client who cannot swallow food cannot drink enough water to meet daily needs. Inadequate fluid intake is a common cause of constipation.

A client is scheduled to undergo rhinoplasty in the morning, and reports medications used on a daily basis, which the nurse records on the client's chart. Which daily medications have the potential to result in constipation? NSAIDs multivitamin without iron acetaminophen laxative

laxative Constipation may also result from chronic use of laxatives ("cathartic colon")because such use can cause a loss of normal colonic motility and intestinal tone. Laxatives also dull the gastrocolic reflex.

A client with enteritis reports frequent diarrhea. What assessment should the nurse should anticipate? metabolic acidosis respiratory acidosis respiratory alkalosis metabolic alkalosis

metabolic acidosis Diarrhea causes a bicarbonate deficit. With loss of the relative alkalinity of the lower GI tract, the relative acidity of the upper GI tract predominates, leading to metabolic acidosis. Loss of acid, which occurs with severe vomiting, may lead to metabolic alkalosis. Diarrhea doesn't lead to respiratory acid-base imbalances, such as respiratory acidosis and respiratory alkalosis.

When bowel sounds are heard about every 15 seconds, the nurse would record that the bowel sounds are hypoactive. absent. normal. sluggish.

normal. Explanation: Normal bowel sounds are heard every 5 to 20 seconds. Hypoactive bowel sound is the description given to auscultation of one to two bowel sounds in 2 minutes. Sluggish is not a term a nurse would use to accurately describe bowel sounds. The nurse records that bowel sounds are absent when no sound is heard in 3 to 5 minutes.

The term used to describe total urine output less than 0.5 mL/kg/hour is nocturia. dysuria. anuria. oliguria.

oliguria. Oliguria is associated with acute and chronic renal failure. Anuria is used to describe total urine output less than 50 mL in 24 hours. Nocturia refers to awakening at night to urinate. Dysuria refers to painful or difficult urination.

A client has a suspected bladder tumor. What is the most common first symptom of a malignant tumor of the bladder? fever dysuria urgency painless hematuria

painless hematuria The most common first symptom of a malignant tumor of the bladder is painless hematuria. Additional early symptoms include UTI with symptoms such as fever, dysuria, urgency, and frequency.

The nurse is caring for a client who reports orange urine. The nurse suspects which factor as the cause of the urine discoloration? metronidazole phenazopyridine hydrochloride phenytoin infection

phenazopyridine hydrochloride Orange to amber-colored urine is caused by concentrated urine due to dehydration, fever, bile, excess bilirubin or carotene, and the medications phenazopyridine hydrochloride and nitrofurantoin. Infection would cause yellow to milky white urine. Phenytoin would cause the urine to become pink to red. Metronidazole would cause the urine to become brown to black.

A client tells the nurse that the stool was colored yellow. The nurse assesses the client for recent foods ingested. pilonidal cyst. occult blood. ingestion of bismuth.

recent foods ingested. Explanation: The nurse should assess for recent foods that the client ingested, as ingestion of senna can cause the stool to turn yellow. Ingestion of bismuth can turn the stool black and, when occult blood is present, the stool can appear to be tarry black.

A client is undergoing diagnostics due to a significant drop in renal output. The physician has scheduled an angiography. This test will reveal details about: renal circulation. urine production. kidney structure. kidney function.

renal circulation. A renal angiography (renal arteriography) provides details of the arterial supply to the kidneys, specifically the location and number of renal arteries (multiple vessels to the kidney are not unusual) and the patency of each renal artery.

A nursing assessment of a client with peritonitis reveals hypotension, tachycardia, and signs and symptoms of dehydration. What else would the nurse expect to find? jaundice and vomiting rectal bleeding and a change in bowel habits severe abdominal pain with direct palpation or rebound tenderness tenderness and pain in the right upper abdominal quadrant

severe abdominal pain with direct palpation or rebound tenderness Peritonitis decreases intestinal motility and causes intestinal distention. A classic sign of peritonitis is a sudden, diffuse, severe abdominal pain that intensifies in the area of the underlying causative disorder (i.e., appendicitis, diverticulitis, ulcerative colitis, a strangulated obstruction). The client may also have rebound tenderness. Tenderness and pain in the right upper abdominal quadrant suggest cholecystitis. Jaundice and vomiting are signs of cirrhosis of the liver. Rectal bleeding or a change in bowel habits may indicate colorectal cancer.

Which procedure is performed to examine and visualize the lumen of the small bowel? peritoneoscopy small bowel enteroscopy panendoscopy colonoscopy

small bowel enteroscopy Explanation: Small bowel enteroscopy is the endoscopic examination and visualization of the lumen of the small bowel. Colonoscopy is the examination of the entire large intestine with a flexible fiberoptic colonoscope. Panendoscopy is the examination of both the upper and lower GI tracts. Peritoneoscopy is the examination of GI structures through an endoscope inserted percutaneously through a small incision in the abdominal wall.

A nurse is teaching a client with malabsorption syndrome about the disorder and its treatment. The client asks which part of the GI tract absorbs food. What is the nurse's best response? stomach rectum small intestine large intestine

small intestine Explanation: The small intestine absorbs products of digestion, completes food digestion, and secretes hormones that help control the secretion of bile, pancreatic juice, and intestinal secretions. The stomach stores, mixes, and liquefies the food bolus into chyme and controls food passage into the duodenum; it doesn't absorb products of digestion. Although the large intestine completes the absorption of water, chloride, and sodium, it plays no part in absorbing food. The rectum is the portion of the large intestine that forms and expels feces from the body; its functions don't include absorption.

An older adult client in a long term care facility is concerned about bowel regularity. During a client education session, the nurse reinforces the medically acceptable definition of "regularity." What is the actual measurement of "regular"? one bowel movement every other day one bowel movement daily two bowel movements daily stool consistency and client comfort

stool consistency and client comfort Normal bowel patterns range from three bowel movements per day to three bowel movements per week. In differentiating normal from abnormal, the consistency of stools and the comfort with which a person passes them are more reliable indicators than is the frequency of bowel elimination.

The nurse is assisting the physician in a percutaneous liver biopsy. In assisting with positioning, the nurse should assist the client into a: high Fowler's position. dorsal recumbent position. lithotomy position. supine position.

supine position. Explanation: The nurse is correct to instruct the client to assume the supine position. Also, the nurse places a rolled towel beneath the right lower ribs.

A client presents to the ED reporting left flank pain and lower abdominal pain. The pain is severe, sharp, stabbing, and colicky in nature. The client has also experienced nausea and emesis. The nurse suspects the client is experiencing: cystitis. Urethral infection. pyelonephritis. ureteral stones.

ureteral stones. The findings are constant with ureteral stones, edema or stricture, or a blood clot. The other answers do not apply.

The nurse is assessing a client for constipation. To identify the cause of constipation, the nurse should begin by reviewing the client's: current medications. activity levels. alcohol consumption. usual pattern of elimination.

usual pattern of elimination. Constipation has many possible reasons; assessing the client's usual pattern of elimination is the first step in identifying the cause.

The nurse recognizes which change of the GI system is an age-related change? weakened gag reflex increased mucus secretion increased motility hypertrophy of the small intestine

weakened gag reflex Explanation: A weakened gag reflex is an age-related change of the GI system. There is decreased motility, atrophy of the small intestine, and decreased mucus secretion.


Conjuntos de estudio relacionados

SHRMSCP reviewed in class practice questions

View Set

Chapter exam end questions; ch 5; Life underwriting

View Set

Apologia General Science Module 16 Study Guide

View Set

Section 4: industrial revolution 4.1-4.4

View Set

PSYC231: Cognitive Psychology (Chapter 8 - Everyday Memory and Memory Errors)

View Set

Keys to your future Choosing a path & Balancing Daily Life

View Set

controllable properties of light

View Set

Foods--Fruits Ch. 15--true/false

View Set